Вы находитесь на странице: 1из 89

ING FROM ...

ED BY PARALLELLESIM

3. The increased popularity and availability of televisions has led to the decline of regional dialects, language
variations which originate from diverse ethnic and cultural heritages and perpetuated by geographic isolation.
A. which originate from diverse ethnic and cultural heritages and perpetuated
B. that originated from diverse ethnic and cultural heritages and perpetuated
C. originated from diverse ethnic and cultural heritages and perpetuated
D. originating from diverse ethnic and cultural heritages and perpetuated
E. originating from diverse ethnic and cultural heritages and perpetuating
3. The increased popularity and availability of televisions has led to the decline of regional dialects, language
variations which originate from diverse ethnic and cultural heritages and perpetuated by geographic isolation.
A. which originate from diverse ethnic and cultural heritages and perpetuated
B. that originated from diverse ethnic and cultural heritages and perpetuated
C. originated from diverse ethnic and cultural heritages and perpetuated
D. originating from diverse ethnic and cultural heritages and perpetuated
E. originating from diverse ethnic and cultural heritages and perpetuating
This is a positive D.
This problem is similar to the Time Zones problem (Q. 4)
Well - between B and D, D is actually parallel because of the correct usage in D of participles (originating and
perpetuated) even though they are structurally dissimilar.
if we were to use the structure of B -

4
variations that originated from should be paralleled by and that were perpetuated by.... then it would have been
parallel.
Alternatively D uses
Variations ORIGINATING from (originating acts as an adjective modifying Variations) and perpetuated by (again
perpetuated acts as an adjective modifying variatons).
4. The growth of the railroads led to the abolition of local times, which was determined by when the sun reached
the observers meridian and differing from city to city, and to the establishment of regional times.
A. which was determined by when the sun reached the observers meridian and differing
B. which was determined by when the sun reached the observers meridian and which differed
C. which were determined by when the sun reached the observers meridian and differing
D. determined by when the sun reached the observers meridian and differed
E. determined by when the sun reached the observers meridian and differing
The only difference between choices D and E is differed/differing, and, as you've no doubt realized, the key issue
is making the choice that's most parallel
to 'determined' (from the first half of the modifier).
Here's the key: 'Determined' isn't a verb here. It's a PARTICIPLE: a verb form that functions as an adjective.
The local times were WEIRD (adjective).
The local times were DETERMINED (participle acting as adjective) by ...
So you need to pick the second half that's also 'participle acting as adjective'. Try them both:
D) The local times were DIFFERED: uh oh, that doesn't work.
E) The local times were DIFFERING from city to city. That works. It's a 'present participle' rather than a 'past
participle', but it's the only one of the
two that's a participle at all.
In D and E, you can tell determined = participle, not verb, because (a) it immediately follows a comma and (b)
there's no subject you could match up with
determined in a normal subject-verb configuration. Even if you match determined with the noun "times" (to which
the clause is referring) "times, determined
by" is not proper subject-verb config.
4. The growth of the railroads led to the abolition of local times, which was determined by when the sun reached
the observers meridian and differing from city to city, and to the establishment of regional times.
A. which was determined by when the sun reached the observers meridian and differing
B. which was determined by when the sun reached the observers meridian and which differed
C. which were determined by when the sun reached the observers meridian and differing
D. determined by when the sun reached the observers meridian and differed
E. determined by when the sun reached the observers meridian and differing

272. As the honeybees stinger is heavily barbed, staying where it is inserted, this results in
the act of stinging causing the bee to sustain a fatal injury.

A. 'this' cannot be used to refer to 'staying ... inserted' - Incorrect

B. Incomplete sentence. Both 'As the ... inserted' and 'with .. injury' are only modifiers. There is no subject.

C. Implies that the stinger results in a fatal injury and the action. Incorrect

D. Same problem as in C
E. Correct

Question from GMATPrep 2:

As the honeybees stinger is heavily barbed, staying where it is inserted, this results in the act of stinging causing the bee to sustain a fatal injury.

A. As the honeybees stinger is heavily barbed, staying where it is inserted, this results in the act of stinging causing
B. As the heavily barbed stinger of the honeybee stays where it is inserted, with the result that the act of stinging causes
C. The honeybees stinger, heavily barbed and staying where it is inserted, results in the fact that the act of stinging causes
D. The heavily barbed stinger of the honeybee stays where it is inserted, and results in the act of stinging causing
E. The honeybees stinger is heavily barbed and stays where it is inserted, with the result that the act of stinging causes

OA is E. My queries to the instructors are:

1)Is E correct because of parallelism? i.e. is//stays//causes?


2) In A, B and D, is 'results in..' unidiomatic?
3) What is this sentence trying to convey really? I think I spent about 2.5 mins on this question trying to get to the gist of it. How should one split the
answer choices?

Thanks,
-BM-
RonPurewal
ManhattanGMAT Staff

Posts: 16352
Joined: Tue Aug 14, 2007 8:23 am

Re: As the honeybees stinger is heavily barbed


Wed Jun 24, 2009 7:08 am

bluementor wrote:Question from GMATPrep 2:

As the honeybees stinger is heavily barbed, staying where it is inserted, this results in the act of stinging causingthe bee to sustain a fatal injury.

A. As the honeybees stinger is heavily barbed, staying where it is inserted, this results in the act of stinging causing
B. As the heavily barbed stinger of the honeybee stays where it is inserted, with the result that the act of stinging causes
C. The honeybees stinger, heavily barbed and staying where it is inserted, results in the fact that the act of stinging causes
D. The heavily barbed stinger of the honeybee stays where it is inserted, and results in the act of stinging causing
E. The honeybees stinger is heavily barbed and stays where it is inserted, with the result that the act of stinging causes

OA is E. My queries to the instructors are:

1)Is E correct because of parallelism? i.e. is//stays//causes?


2) In A, B and D, is 'results in..' unidiomatic?
3) What is this sentence trying to convey really? I think I spent about 2.5 mins on this question trying to get to the gist of it. How should one split the
answer choices?

Thanks,
-BM-

first off, the construction (preposition) + NOUN + VERBing is WRONG, unless the preposition refers directly to the NOUN. (that isn't
usually the case, so, if you're in doubt, you should strike choices with this sort of construction.)

for instance:
i've never heard of bees stinging dogs
WRONG. this is not an issue of whether you've heard of bees themselves; it's an issue of whether you've heard of their stinging dogs.

...results in the act of stinging causing...


WRONG. this doesn't result in the act of stinging itself; it results in what is caused by the act of stinging.

i have a picture of my cousin playing hockey.


CORRECT. this time, the picture is actually of my cousin, so we're good.

therefore, (a) and (d) are wrong because of "...results in the act of stinging causing...".

--

the pronoun "this" in (a) doesn't refer to any particular noun. this consideration also kills (a).

--

you can't say "the stinger results in...".


"results in..." can only be used when it's LITERALLY TRUE. for instance, you could say that the attempt resulted in failure, since the attempt ITSELF
ended in failure.
if you understand this literal meaning, then it goes without saying that you can't use this sort of construction for physical objects.
TAKEAWAY:
you can only say "X results in Y" when X is an ACTION. if X is an OBJECT, you can NEVER say that X "results" in anything.
this kills choices (c) and (d), in which "stinger" is the subject of the verb "results".

--

choice (b) misuses the connector "as".


the connector "as" connects two complete sentences BY ITSELF. if "as" is used to connect two complete sentences, it should NOT
be used in conjunction with any other connector words.

in choice (b), "as" and "with" are used together. the use of either of these prohibits the use of the other, so this choice is wrong.

--

not sure what you're asking in #3. it looks like at least 2 completely different questions.

the MEANING of the sentence is that the bee's stinger stays where it's placed, and so the bee dies because it can't get away after the sting.

as far as HOW TO SPLIT, you'll notice that i've given several different angles from which to approach this one.
there aren't really genuine "splits" - i.e., easily identifiable points of divergence between clearly parallel items - so you just have to try to identify any of
the multitude of errors present.

269. A South American bird that forages for winged termites and other small insects while
swinging upside down from the foliage of tall trees, the graveteiro belongs to the
ovenbird family, a group of New World tropical birds that includes more than 230
species and that are represented in virtually every kind of habitat

A. 'that are' cannot be used to refer to singular form 'group'

B. Correct

C. 'include' cannot be used to refer to singular form 'group'

D. Same as A

269. A South American bird that forages for winged termites and other small insects while swinging
upside down from the foliage of tall trees, the graveteiro belongs to the ovenbird family, a group
of New World tropical birds that includes more than 230 species and that are represented in
virtually every kind of habitat.
A. graveteiro belongs to the ovenbird family, a group of New World tropical birds that includes
more than 230 species and that are
B. graveteiro belongs to the ovenbird family, a group of New World tropical birds that includes
more than 230 species and is
C. graveteiro belongs to the ovenbird family, a group of New World tropical birds that include more
than 230 species and is
D. graveteiro, which belongs to the ovenbird family, a group of New World tropical birds that
includes more than 230 species and that are
E. graveteiro, which belongs to the ovenbird family, a group of New World tropical birds that
includes more than 230 species and is

265. Prospecting for gold during the California gold rush was a relatively easy task, because of
erosion, prehistoric glacier movement, and ancient, gold-bearing riverbeds thrust to the surface
by volcanic activity put gold literally within reach for anybody with a pan or shovel.
A. because of erosion, prehistoric glacier movement, and ancient, gold-bearing riverbeds thrust to
the surface by volcanic activity put gold literally within reach for
B. because of erosion, prehistoric glacier movement, and volcanic activity that thrust ancient, goldbearing
riverbeds to the surface, and putting gold literally within reach of
C. owing to erosion, prehistoric glacier movement, and volcanic activity that had thrust ancient,
gold-bearing riverbeds to the surface, and putting gold literally within reach of
D. since erosion, prehistoric glacier movement, and volcanic activity that thrust ancient, goldbearing
riverbeds to the surface, putting gold literally within reach for
E. since erosion, prehistoric glacier movement, and ancient, gold-bearing riverbeds thrust to the
surface by volcanic activity put gold literally within reach of

265. Prospecting for gold during the California gold rush was a relatively easy task,
because of erosion, prehistoric glacier movement, and ancient, gold-bearing riverbeds
thrust to the surface by volcanic activity put gold literally within reach for anybody
with a pan or shovel.

A. because the word "of" is a preposition, and you cannot follow a preposition with a clause. You CAN, of course, follow a preposition
with a noun, or even a noun list, but here these nouns are acting as the subject of the verb "put." This makes a clause;

B. "putting gold literally within reach" is not parallel to "thrust."

C. Use of past perfect 'had thrust' is incorrect

D. This choice violates the correct usage of "since." "Putting" following the comma here is functioning as a modifier, not a verb.
"Since," which is used here interchangeably with "because," should be followed by a clause. Incorrect

E. Correct

251. On Earth, among the surest indications of sunspot cycles are believed to be the rate
that trees grow, as seen in the rings visible in the cross sections of their trunks.

A. singular 'is believed' is required as only one indication is mentioned.

B. Same as A

C. as seen in the rings...' is a modifier that must be placed next to the thing that it modifies, which is 'the rate at which trees grow'.
the way choice c is currently written, it says that indications of sunspot cycles can be seen directly in the rings - and it also implies
that sunspot cycles (instead of trees) have rings.

D. Implies that sunspot cycles are believed to be the growth rate. incorrect

E. Correct

251. On Earth, among the surest indications of sunspot cycles are believed to be the rate that trees
grow, as seen in the rings visible in the cross sections of their trunks.
A. On Earth, among the surest indications of sunspot cycles are believed to be the rate that trees
grow
B. On Earth, among the surest indications of sunspot cycles are, it is believed, the rate of tree
growth
C. On Earth, the rate at which trees grow is believed to be among the surest indications of sunspot
cycles
D. Among the surest indications on Earth of sunspot cycles, believed to be the tree growth rate
E. Among the surest indications on Earth of sunspot cycles is believed to be the rate at which trees
grow

Fri Aug 23, 2013 9:18 am

Quote
As per my analysis, there is a 3/2 split seen at the beginning.
If we are saying on Earth(Do we need any other planet?)
A is wrong as we all agree
B Is wordy and "it is believed" does not fit in
C (Contention point) it indicates Sunspots as seen in trees? That is not correct, it should be trees
D As "iamcste" has mentioned as seen should be closer to trees, but in D it incorrectly modifies rate.
E is best among the given options.

SC is getting mad, madder maddest slowly. But, it's nice to learn new things

@vietmoi999" Welcome to GMAT SC!(Pun intended)


SAID BUSINESS SCHOOL, OXFORD - MBA CLASS OF 2008

248. Marconis conception of the radio was as a substitute for the telephone, a tool for private
conversation; instead, it is precisely the opposite, a tool for communicating with a large, public
audience.
A. Marconis conception of the radio was as a substitute for the telephone, a tool for private
conversation; instead, it is
B. Marconi conceived of the radio as a substitute for the telephone, a tool for private conversation,
but which is
C. Marconi conceived of the radio as a tool for private conversation that could substitute for the
telephone; instead, it has become
D. Marconi conceived of the radio to be a tool for private conversation, a substitute for the
telephone, which has become
E. Marconi conceived of the radio to be a substitute for the telephone, a tool for private
conversation, other than what it is,
248. Marconis conception of the radio was as a substitute for the telephone, a tool for
private conversation; instead, it is precisely the opposite, a tool for communicating
with a large, public audience

A. the verb "conceived" is preferred to the noun "conception."

B. 'which' seems to modify telephone. Incorrect

C. Correct

D. 'conceived of X as Y' is the correct idiom

E. Same as D

246. The globalization of financial-services companies has been a boon to money launders,
because of allowing money placed in a bank in a less regulated jurisdiction to be
transferred to a branch in a more regulated one

A. 'it allows' is the correct idiom.

B. Same as A

C. 'allows that' is unidiomatic.

D. 'have been placed' wrong tense to refer to money

E. Correct

246. The globalization of financial-services companies has been a boon to money launders, because of
allowing money placed in a bank in a less regulated jurisdiction to be transferred to a branch in a
more regulated one.
A. of allowing money placed in a bank in a less regulated jurisdiction to be transferred
B. of allowing the transfer of money placed in a bank in a less regulated jurisdiction
C. it allows that money placed in a bank in a less regulated jurisdiction is transferred
D. it allows the transfer of money have been placed in a bank in a less regulated jurisdiction
E. it allows money placed in a bank in a less regulated jurisdiction to be transferred

244. Scholars who once thought Native American literatures were solely oral narratives
recorded by missionaries or anthropologists now understand this body of work to
consist of both oral literatures and the written works of Native American authors, who
have been publishing since 1772

A. Correct

B. 'and' changes the meaning of the sentence

C. Same as C

D. The sentence implies that 'The native american literate' now understand. Incorrect

E. Same as D

Ok, finally got the light at the end...


D has a major problem with the meaning...Native american literatures become (subject) in D...so see the full meaning...
Native american literatures, which some.....now understand...
So D conveys the meaning that literatures now understand and not the scholars.
Good one!!
244. Scholars who once thought Native American literatures were solely oral narratives recorded by
missionaries or anthropologists now understand this body of work to consist of both oral
literatures and the written works of Native American authors, who have been publishing since
1772.
A. Scholars who once thought Native American literatures were solely oral narratives
B. Scholars thinking of Native American literatures once solely as oral narratives, and
C. Scholars who once had thought of Native American literatures solely as oral narratives and
D. Native American literatures, which some scholars once thought were solely oral narratives
E. Native American literatures, which some scholars once, thinking they were solely oral narratives

243. There are several ways to build solid walls using just mud or clay, but the most
extensively used method has been the forming of bricks out of mud or clay, and, after
some preliminary air drying or sun drying, they are laid in the wall in mud mortar
A. 'the forming' is not parallel to 'are laid'

B. 'forming ' not parallel to 'to lay'

C. 'having bricks' not parallel to 'they were'

D. Correct

E. 'method has been' is present continuous. Therefore using the bricks 'were formed' - simple past - is incorrect

243. There are several ways to build solid walls using just mud or clay, but the most extensively used
method has been the forming of bricks out of mud or clay, and, after some preliminary air drying
or sun drying, they are laid in the wall in mud mortar.
A. the forming of bricks out of mud or clay, and, after some preliminary air drying or sun drying,
they are laid
B. forming the mud or clay into bricks, and, after some preliminary air drying or sun drying, to lay
them
C. having bricks formed from mud or clay, and, after some preliminary air drying or sun drying,
they were laid
D. to form the mud or clay into bricks, and, after some preliminary air drying or sun drying, to lay
them
E. that bricks were formed from mud or clay, which, after some preliminary air drying or sun
drying, were laid

242. Several financial officers of the company spoke on condition that they not be named
in the press reports.

A. Correct IMO A

Subjunctive
<subjunctive>+that+noun

There is uncertainty that what press will do

B. Wordy

C. 'are not used' incorrect tense

D. 'condition that' is the correct idiom

E. Same as D

242. Several financial officers of the company spoke on condition that they not be named in the press
reports.
A. that they not be named
B. that their names will not be used
C. that their names are not used
D. of not having their names
E. of not naming them

264. Emily Dickinsons letters to Susan Huntington Dickinson were written over a period
beginning a few years before Susans marriage to Emilys brother and ending shortly
before Emilys death in 1886, outnumbering her letters to anyone else.

A. the phrase in question, "outnumbering ...", is NOT, in any way whatsoever, a "(direct/indirect) result" of the time period over
which the letters were written. these are two completely independent and unrelated observations about the letters, and so they
can't be placed into the sort of construction that appears in choice (a).

B. Since these events happened in the past - the use of present tense 'begins' is incorrect

C. 'beginning' is not parallel to 'that ends'

D. This construction implies that the 'period' is 'outnumbering' her letters. (as outnumbering also seems to modify period) Incorrect

E. Correct

264. Emily Dickinsons letters to Susan Huntington Dickinson were written over a period beginning a
few years before Susans marriage to Emilys brother and ending shortly before Emilys death in
1886, outnumbering her letters to anyone else.
A. Dickinson were written over a period beginning a few years before Susans marriage to Emilys
brother and ending shortly before Emilys death in 1886, outnumbering
B. Dickinson were written over a period that begins a few years before Susans marriage to Emilys
brother and ended shortly before Emilys death in 1886, outnumber
C. Dickinson, written over a period beginning a few years before Susans marriage to Emilys brother
and that ends shortly before Emilys death in 1886and outnumbering
D. Dickinson, which were written over a period beginning a few years before Susans marriage to
Emilys brother, ending shortly before Emilys death in 1886, and outnumbering
E. Dickinson, which were written over a period beginning a few years before Susans marriage to
Emilys brother and ending shortly before Emilys death in 1886, outnumber

262. Some scientists contend that many species of dinosaur had a metabolism more like a
warm-blooded mammals than a cold-blooded reptiles.

A. Correct

B. Metabolism is singular; therefore 'were' is incorrect

C. 'those of' is unnecessary as the sentence already has the possessive form 'warm blooded mammal's'

D. 'Contend that' is the correct form

E. Same as D

262. Some scientists contend that many species of dinosaur had a metabolism more like a warmblooded
mammals than a cold-blooded reptiles.
A. that many species of dinosaur had a metabolism more like
B. that the metabolism of many dinosaur species were more like
C. that the metabolisms of many species of dinosaurs were more like those of
D. there were many species of dinosaurs that had metabolisms more like those of
E. there were many dinosaur species that had a metabolism more like that of

261. Approved April 24, 1800, the act of Congress that made provision for the removal of the
government of the United States to the new federal city, Washington, D.C., also established the
Library of Congress.
A. Approved April 24, 1800, the act of Congress that made provision for the removal of the
government of the United States to the new federal city, Washington, D.C., also established
B. The act of Congress, which was approved April 24, 1800, making provision for the removal of
the government of the United States to the new federal city, Washington, D.C., also established
C. The act of Congress approved April 24, 1800, which made provision for the removal of the
government of the United States to the new federal city, Washington, D.C., and established
D. Approved April 24, 1800, making provision for the removal of the government of the United
States to the new federal city, Washington, D.C., the act of Congress also established
E. Approved April 24, 1800, the act of Congress made provision for the removal of the government
of the United States to the new federal city, Washington, D.C., also establishing

261. Approved April 24, 1800, the act of Congress that made provision for the removal of
the government of the United States to the new federal city, Washington, D.C., also
established the Library of Congress

A. Correct

B. This construction implies that the " Library of Congress " while 'making the provision'. Incorrect

C. without the comma - the meaning changes to 'The act of congress approved April 24' - the date- which does not make any sense

D. 'Approved ...' needs to modify 'The act of Congress' and not 'making'.

E. The big problem with choice e is that it strips 'the act of congress' of a necessary modifier - so that, instead of saying 'the act of
congress that did X', it just says 'the act of congress' - as if congress has only ever performed one act in its history.

259. In 1914 a total of 469,000 cars and trucks were produced in the United States, but in 1929 almost
twice the numbers of tucks alone came off the assembly lines.
A. the numbers of tucks alone
B. that number of trucks alone
C. the number of trucks by themselves
D. as many trucks themselves
E. as many trucks by themselves
259. In 1914 a total of 469,000 cars and trucks were produced in the United States, but in
1929 almost twice the numbers of trucks alone came off the assembly lines

A. plural 'numbers' is incorrect. Since we are comparing the number in 1929 to that of 1914 - we need to use 'twice that number'

B. Correct

C. Since we are comparing the number in 1929 to that of 1914 - we need to use 'twice that number'

D. choice says "twice AS MANY trucks", which means "twice as many trucks as were produced in 1914". The problem is that the
sentence doesn't tell us how many trucks were produced in 1914 - the only figure given is a combined figure for cars and trucks - so
this statement doesn't make any sense in context.

Also, "trucks themselves" doesn't make sense. You don't use "x itself" unless you are trying to emphasize some element of the
inherent nature of x (as opposed to something associated with x, or with some part of x).

E. changes the meaning - 'by themselves came off the assembly line' says the trucks came off the line by themselves.

259. In 1914 a total of 469,000 cars and trucks were produced in the United States, but in 1929 almost
twice the numbers of tucks alone came off the assembly lines.
A. the numbers of tucks alone
B. that number of trucks alone
C. the number of trucks by themselves
D. as many trucks themselves
E. as many trucks by themselves

256. When working with overseas clients, an understanding of cultural norms is at least as important
as grasping the pivotal business issues for the global manager.
A. When working with overseas clients, an understanding of cultural norms is at least as important
as grasping the pivotal business issues for the global manager.
B. When they work with overseas clients, understanding cultural norms is at least of equal
importance to the global manager as grasping the pivotal business issues.
C. For global managers working with overseas clients, understanding cultural norms is at least as
important as grasping the pivotal business issues.
D. For global managers working with overseas clients, an understanding of cultural norms is at least
as important to them as that they grasp the pivotal business issues.
E. Global managers working with overseas clients find an understanding of cultural norms to be
equally important as grasping the pivotal business issues.

256. When working with overseas clients, an understanding of cultural norms is at least as
important as grasping the pivotal business issues for the global manager.

A. 'When working ..' needs to modify global manager

B. Same as A

C. Correct

D. 'them' is ambiguous

E. 'as important as' is the correct idiom

236. Imported into Massachusetts form Europe in 1869, the gypsy moth was used by a
French
scientist in an attempt at developing a strong strain of silk-producing insects, crossing
gypsy
moths with adult silkworms.
A. Imported into Massachusetts from Europe in 1869, the gypsy moth was used by a French
scientist in an attempt at developing a strong strain of silk-producing insects, crossing gypsy
moths with adult silkworms.
B. Imported into Massachusetts form Europe in 1869, a French scientist was attempting to develop a
strong strain of silk-producing insects by crossing gypsy moths with adult silkworms.
C. To cross gypsy moths with adult silkworms, in attempting the development of a strong strain of
silk-producing insects, a French scientist in 1869 imported the gypsy moth into Massachusetts
from Europe.
D. The gypsy moth was imported into Massachusetts from Europe in 1869 by a French scientist
attempting to develop a strong strain of silk-producing insects by crossing gypsy moths with
adult silkworms.
E. In an attempt at the development of a strong strain of silk-producing insects, a French scientist,
importing the gypsy moth from Europe into Massachusetts in 1869 in order to cross gypsy moths
and adult silkworms.
236. Imported into Massachusetts form Europe in 1869, the gypsy moth was used by a
French scientist in an attempt at developing a strong strain of silk-producing insects,
crossing gypsy moths with adult silkworms

A. Correct idiom is 'to develop'. Also, the modifier 'crossing ..silkworms' is used unclear.

B. Incorrectly says that the scientist was imported

C. The moth was imported in '1869' . This option says the scientist used it in 1869 which is incorrect

D. Correct

E. This choice says the scientist imported the moth- which is not said in the original sentence. Incorrect

I would rather go with option A ... can't "attemt at" be used at all ????

Some help here please!

the biggest problem with choice (a) is actually not an idiom issue -- it's the gross misuse of the COMMA + -ING modifier.
see:
the gypsy moth was used by a French scientist in an attempt at developing a strong strain of silk-producing insects, crossing gypsy moths
with...

when you use COMMA -ING modifiers:


* they modify the action of the preceding clause, and
* the SUBJECT of the preceding clause should be the agent of the -ING action

the second of these rules, if applied to choice (a), tells us that the gypsy moth itself (the subject of the preceding clause) is the one who is "crossing
gypsy moths...". that is most certainly not true, so choice (a) is ungrammatical.
Pueden hacerle preguntas a Ron en castellano

232. Despite the growing number of people who purchase plane tickets online, airline
executives are convinced that, just as one-third of bank customers still prefer human
tellers to automatic teller machines, many travelers will still use travel agents.

A. Correct

B. future tense needs 'will' not 'would'. Incorrect placement of 'that'. distorts meaning

C. notice that "prefer ... compared to" is redundant. You should just say "prefer ... to", as the word "prefer" already encodes the idea
of making a comparison.

D. future tense needs 'will' not 'would'

E. Same as D

232. Despite the growing number of people who purchase plane tickets online, airline executives are
convinced that, just as one-third of bank customers still prefer human tellers to automatic teller
machines, many travelers will still use travel agents.
A. growing number of people who purchase plane tickets online, airline executives are convinced
that, just as one-third of bank customers still prefer human tellers to automatic teller machines,
many travelers will
B. growing number of people who purchase plane tickets online, airline executives are convinced,
just as one-third of bank customers still prefer human tellers to automatic teller machines, that
many travelers would
C. growing number of people purchasing plane tickets online, airline executives are convinced, just
as one-third of bank customers still prefer human tellers as compared to automatic teller
machines, many travelers will
D. fact that the number of people purchasing plane tickets online is growing, airline executives are
convinced, just as one-third of bank customers still prefer human tellers as compared to
automatic teller machines, that many travelers would
E. fact that the number of people who purchase plane tickets online are growing, airline executives
are convinced that, just as one-third of bank customers still prefer human tellers compared with
automatic teller machines, many travelers would

230. Analyzing campaign expenditures, the media has had as a focus the high costs and
low ethics of
campaign finance, but they have generally overlooked the cost of actually administering
elections, which includes facilities, transport, printing, staffing, and technology.
A. Analyzing campaign expenditures, the media has had as a focus
B. Analyses of campaign expenditures by the media has been focus on
C. In analyzing campaign expenditures, the media have focused on
D. Media analyses of campaign expenditures have had as a focus
E. In their analysis of campaign expenditures, the media has been focusing on

230. Analyzing campaign expenditures, the media has had as a focus the high costs and
low ethics of campaign finance, but they have generally overlooked the cost of
actually administering elections, which includes facilities, transport, printing, staffing,
and technology.

A. Media is plural and requires 'have'

B. Analyses is plural ; therefore 'have' is needed

C. Correct

D. No parallelism - 'and they' - here they needs to refer to media; however in this sentence only media's analyses is present not the
media

E. Same as A

227. For protection from the summer sun, the Mojave lived in open-sided, flat-topped dwellings known
as shades, each a roof of poles and arrowweed supported by posts set in a rectangle.
A. each a roof of poles and arrowweed
B. each a roof of poles and arrowweed that are being
C. with each being a roof of poles and arrowweed
D. with roofs of poles and arrowweed to be
E. with roofs of poles and arrowweed that are

227. For protection from the summer sun, the Mojave lived in open-sided, flat-topped
dwellings known as shades, each a roof of poles and arrow weed supported by posts set
in a rectangle

A. Correct

B. 'roof' is singular; 'are supported' is incorrect

C. Wordy. 'being' not preferred

D. 'to be supported' is incorrect usage. Meaning ambiguity - could mean each shade has multiple roofs

E. Meaning ambiguity - could mean each shade has multiple roofs

226. According to a survey of graduating medical students conducted by the Association of American
Medical Colleges, minority graduates are nearly four times more likely than are other graduates
in planning to practice in socioeconomically deprived areas.
A. minority graduates are nearly four times more likely than are other graduates in planning to
practice
B. minority graduates are nearly four times more likely than other graduates who plan on practicing
C. minority graduates are nearly four times as likely as other graduates to plan on practicing
D. it is nearly four times more likely that minority graduates rather than other graduates will plan to
practice
E. it is nearly four times as likely for minority graduates than other graduates to plan to practice

226. According to a survey of graduating medical students conducted by the Association


of American Medical Colleges, minority graduates are nearly four times more likely
than are other graduates in planning to practice in socioeconomically deprived areas.

A. Correct idiom is 'as likely as'. Also, compares minority graduates to a likelihood

B. Correct idiom is 'as likely as'. Changes meaning - comparison between 'minority graduates' and 'other graduates who do X'

C. Correct

D. Correct idiom is 'as likely as'

E. Correct idiom is 'as likely as'


224. Over 75 percent of the energy produced in France derives from nuclear power, while in Germany
it is just over 33 percent.
A. while in Germany it is just over 33 percent
B. compared to Germany, which uses just over 33 percent
C. whereas nuclear power accounts for just over 33 percent of the energy produced in Germany
D. whereas just over 33 percent of the energy comes from nuclear power in Germany
E. compared with the energy from nuclear power in Germany, where it is just over 33 percent

224. Over 75 percent of the energy produced in France derives from nuclear power, while
in Germany it is just over 33 percent

A. 'it' has no clear antecedent

B. Changes meaning. The sentence speaks of production not usage.

C. Correct

D. Implies that 33% of the energy in France comes from Germany

E. Compares a percent and energy . Incorrect

223. The United States minted about 857 million silver-colored Susan B.
Anthony dollars between
1979 and 1981, but the coin proved unpopular because it looked and felt too
much like a quarter.
A. The United States minted about 857 million silver-colored Susan B. Anthony dollars between
1979 and 1981, but the coin
B. About 857 million silver-colored Susan B. Anthony dollars were minted as coins in the United
States between 1979 and 1981 but
C. About 857 million silver-colored Susan B. Anthony dollars that were minted between 1979 and
1981 in the United States
D. About 857 million silver-colored Susan B. Anthony dollars that the United States minted
between 1979 and 1981
E. Between 1979 and 1981 the United States minted about 857 million silver-colored Susan B.
Anthony dollars, which

223. The United States minted about 857 million silver-colored Susan B. Anthony
dollars between 1979 and 1981, but the coin proved unpopular because it looked and
felt too much like a quarter

A. Correct

B. 'It' in the second half in this option does not have a clear antecedent.

C. 'it' cannot refer to 857 million coins

D. Same as C

E. 'it' has no clear singular antecedent

V IF + WERE ... WOULD or IF + past tense ... WOULD or Were ... + would. Only C gets it right.

217. Unlike most other mergers in the utility industry, which have been driven by the need
to save money and extend companies service areas, the merger of the nations
leading gas and electric company is intended to create a huge marketing network for
the utilities in question with states opening their utility markets to competition.

A. "with states opening..." is incorrect. when you have an -ING WITHOUT COMMA modifier, like this one, it's a RESTRICTIVE
("essential") modifier. this means that we're only talking about those states that are "opening their utility markets to competition",
the implication being that only some states are doing so. the intended meaning is better conveyed by "as states open..."

213. A mixture of poems and short fiction, Jean Toomers Cane has been called one of the three best
novels ever written by Black Americansthe others being Richard Wright, author of Native Son,
and Ralph Ellison, author of Invisible Man.
A. Black Americansthe others being Richard Wright, author of Native Son, and Ralph Ellison,
author of Invisible Man
B. Black Americansincluding Native Son by Richard Wright and Invisible Man by Ralph Ellison
C. a Black Americanincluding Richard Wright, author of Native Son, and Ralph Ellison, author
of Invisible Man
D. a Black Americanthe others being Richard Wright, author of Native Son, and Ralph Ellison,
author of Invisible Man
E. a Black Americanthe others being Richard Wrights Native Son and Ralph Ellisons Invisible
Man

213. A mixture of poems and short fiction, Jean Toomers Cane has been called one of the
three best novels ever written by Black Americansthe others being Richard Wright,
author of Native Son, and Ralph Ellison, author of Invisible Man

A. The book was written by one person. Therefore 'a black american' is needed.

B. Same as A

C. 'including...' gives the other books. therefore must be novels and not authors.

D. Same as C

E. Correct

212. When drive-ins were at the height of their popularity in the late 1950s, some 4,000
existed in the United States, but today there are less than one-quarter that many.

A. less is used for amounts not numbers

B. Correct

C. amount cannot be used to refer to a number 4000.

D. Same as C

E. 'it' has no clear antecedent

212. When drive-ins were at the height of their popularity in the late 1950s, some 4,000 existed in the
United States, but today there are less than one-quarter that many.
A. there are less than one-quarter that many
B. there are fewer than one-quarter as many
C. there are fewer than one-quarter of that amount
D. the number is less than one-quarter the amount
E. it is less than one-quarter of that amount

in general, you won't need the second half of that construction if you have already mentioned the data elsewhere in the sentence.
the earlier part of this sentence already mentions the # of drive-ins in the u.s. in the 1950's, so it is not stated again. since it is not stated as part of an
actual parallel construction, you don't need the second "as".

examples:
there were once 20 shirts on this shelf, but, now, barely half as many are left. --> i already mentioned the data (i.e., 20 shirts) earlier in the sentence,
so it's not mentioned again.

there are barely half as many shirts on this shelf [i]as there were last week.[/i] --> i didn't mention this comparison point earlier, so i'm mentioning it
now

this is a little bit too much analysis. you can rule these choices out in a single stroke, because they contain "amount".

recall that "amount" is a construction that can only be used to refer to UNCOUNTABLE quantities -- i.e., continuous quantities / mass nouns, which are
not divided into distinctly countable units.
for instance, you can have an "amount" of water, furniture, etc., but you CANNOT have an "amount" of people, items, etc.

this sentence is very clearly speaking about a countable quantity -- the number (not "amount") of drive-in theaters. since that's countable, any choice
with "amount" can be eliminated immediately.
Pueden hacerle preguntas a Ron en castellano

210. At the annual stockholders meeting, investors heard a presentation on the numerous
challenges facing the company, including among them the threat from a rivals
multibillion-dollar patent-infringement suit and the declining sales for the companys
powerful microprocessor chip.

A. The correct idioms are 'threat of' and 'sales of'.


B. 'which' seems to refer to company. Incorrect

C. Same as A

D. Correct

E. Same as A

209. While studying the genetic makeup of corn, a new class of mutant genes was discovered by
Barbara McClintock, a discovery which led to greater understanding of cell differentiation.
A. a new class of mutant genes was discovered by Barbara McClintock, a discovery which led
B. a new class of mutant genes in corn were discovered by Barbara McClintock, leading
C. Barbara McClintock discovered a new class of mutant genes, and it led
D. Barbara McClintock discovered a new class of mutant genes, a discovery that led
E. Barbara McClintock, who discovered a new class of mutant genes, leading

208. Because of a similarity to dance, synchronized swimmingexhibition swimming in which the


movements of one or more swimmers synchronize to a musical accompanimentis sometimes
called water ballet, especially in theatrical situations.
A. of one or more swimmers synchronize to
B. of one swimmer or more is synchronized with
C. of one or more swimmers are synchronized with
D. by one swimmer or more is synchronized to
E. by one or more swimmers synchronize to

208. Because of a similarity to dance, synchronized swimmingexhibition swimming in


which the movements of one or more swimmers synchronize to a musical
accompanimentis sometimes called water ballet, especially in theatrical situations.

A. Verb 'are synchronized' is required. Also, correct idiom is 'synchronized with'

B. 'Movements' is plural and requires 'are'

C. Correct

D. Same as B

E. Same as A

chindia wrote:
Thanks Mitch.

Is it an absolute phrase?
There is a distinction.

An APPOSITIVE serves to explain or define the preceding NOUN:


Mary waved to Mr. Smith, her favorite teacher in the whole school.
Here, the appositive her favorite teacher serves to define Mr. Smith, the preceding noun.

An ABSOLUTE PHRASE serves as an adverb, providing context for the entire preceding CLAUSE:
Mary entered the room, her eyes shining brightly.
Here, the absolute phrase her eyes shining brightly does not refer solely to the preceding noun.
Rather, it modifies the entire preceding clause, explaining HOW Mary entered the room.

_________________
GMATGuruNY wrote:
agemroy wrote:
A March 2000 Census Bureau survey showed that Mexico accounted for more than a quarter of all foreign-born residents of the United States the
largest share for any country to contribute since 1890, when about 30 percent of the countrys foreign-born population was from Germany.
1. the largest share for any country to contribute
2. the largest share that any country has contributed
3. which makes it the largest share for any country to contribute
4. having the largest share to be contributed by any country
5. having the largest share to have been contributed by any country

OA is A
I received a PM asking me to comment.

In C, which incorrectly refers to the United States. Eliminate C.

In D and E, any country since 1890 implies -- incorrectly -- that the sentence is discussing countries that have come into existence since 1890. The
intended meaning of the sentence is to discuss how the different countries have contributed since 1890. Eliminate D and E.
In A, to contribute since 1890 makes no sense. Since requires the present perfect tense. Eliminate A.

The correct answer is B.

If the OA is A, ignore this question.


Hi Mitch,
The right answer 'the largest share that any country has contributed' makes it to be placed beside United States.

Though it is the best of answer choices here, doesn't it represent an example of misplaced modifier?
205. Not only did the systematic clearing of forests in the United States create farmland (especially in
the Northeast) and gave consumers relatively inexpensive houses and furniture, but it also caused
erosion and very quickly deforested whole regions.
A. Not only did the systematic clearing of forests in the United States create farmland (especially in
the Northeast) and gave consumers relatively inexpensive houses and furniture, but it also
B. Not only did the systematic clearing of forests in the United States create farmland (especially in
the Northeast), which gave consumers relatively inexpensive houses and furniture, but also
C. The systematic clearing of forests in the United States, creating farmland (especially in the
Northeast) and giving consumers relatively inexpensive houses and furniture, but also
D. The systematic clearing of forests in the United States created farmland (especially in the
Northeast) and gave consumers relatively inexpensive houses and furniture, but it also
E. The systematic clearing of forests in the United States not only created farmland (especially in
the Northeast), giving consumers relatively inexpensive houses and furniture, but it

205. Not only did the systematic clearing of forests in the United States create farmland
(especially in the Northeast) and gave consumers relatively inexpensive houses and
furniture, but it also caused erosion and very quickly deforested whole regions

A. 'create' is not parallel to 'gave'.

B. Incorrect parallelism in Not only .. but also. 'but also .. ' part does not have a subject

C. removing the non essential clause (causing ..furniture) makes the sentence -.. 'United States but also .. ' - this is an incorrect
construction

D. Correct

E. Not only .. but .. parallelism is missing. But it is incorrect. Also, Not only .. but also do not show the contradiction intended

204. According to a recent study of consumer spending on prescription medications,


increases in the sales of the 50 drugs that were advertised most heavily accounts for
almost half of the $20.8 billion increase in drug spending last year, the remainder of
which came from sales of the 9,850 prescription medicines that companies did not
advertise or advertised very little.

A. present tense 'accounts' cannot be used for something that happened last year. Also, 'which' cannot be used to refer to increase

B. 'what' has an ambiguous reference - increases or sales

C. Correct

D. Incorrect use of while - while implies a contradiction D is wrong with the incorrect usage of comma and changing the main verb " account"
into "accounting for"

E. 'which' does not have a clear reference.

204. According to a recent study of consumer spending on prescription medications, increases in the
sales of the 50 drugs that were advertised most heavily accounts for almost half of the $20.8
billion increase in drug spending last year, the remainder of which came from sales of the 9,850
prescription medicines that companies did not advertise or advertised very little.
A. heavily accounts for almost half of the $20.8 billion increase in drug spending last year, the
remainder of which came
B. heavily were what accounted for almost half of the $20.8 billion increase in drug spending last
year; the remainder of the increase coming
C. heavily accounted for almost half of the $20.8 billion increase in drug spending last year, the
remainder of the increase coming
D. heavily, accounting for almost half of the $20.8 billion increase in drug spending last year, while
the remainder of the increase came
E. heavily, which accounted for almost half of the $20.8 billion increase in drug spending last year,
with the remainder of it coming

191. Like the grassy fields and old pastures that the upland sandpiper needs for feeding and
nesting when it returns in May after wintering in the Argentine Pampas, the sandpipers
vanishing in the northeastern United States is a result of residential and industrial
development and of changes in farming practices

A. Use of gerund 'vanishing' is incorrect. the verb 'is vanishing' is required

B. Correct

C. 'Like X , Y ' is the correct comparison idiom. The comma needs to be followed by 'the bird'

D. & E. Same as C

191. Like the grassy fields and old pastures that the upland sandpiper needs for feeding and nesting when
it returns in May after wintering in the Argentine Pampas, the sandpipers vanishing in the
northeastern United States is a result of residential and industrial development and of changes in
farming practices.
A. the sandpipers vanishing in the northeastern United States is a result of residential and industrial
development and of changes in
B. the bird itself is vanishing in the northeastern United States as a result of residential and industrial
development and of changes in
C. that the birds themselves are vanishing in the northeastern United States is due to residential and
industrial development and changes to
D. in the northeastern United States, sandpipers vanishing is due to residential and industrial
development and to changes in
E. in the northeastern United States, the sandpipers vanishing, a result of residential and industrial
development and changing

192. The city has proposed a number of water treatment and conservation projects the cost of which raises
water bills high enough so that even environmentalists are beginning to raise alarms.
A. the cost of which raises water bills high enough so that
B. at a cost raising water bills so high that
C. at a cost which raises water bills high enough so
D. whose cost will raise water bills so high that
E. whose cost will raise water bills high enough so that

192. The city has proposed a number of water treatment and conservation projects the cost of
which raises water bills high enough so that even environmentalists are beginning to raise
alarms

A. 'high enough to' or 'so high that' are the correct idioms required. This options mixes both

B. gerund 'raising' needs to be replaced by a verb

C. Same as A

D. Correct

E. Same as A

194. The Organization of Petroleum Exporting Countries (OPEC) had long been expected to announce a
reduction in output to bolster sagging oil prices, but officials of the organization just recently
announced that the group will pare daily production by 1.5 million barrels by the beginning of next
year, but only if non-OPEC nations, including Norway, Mexico, and Russia, were to trim output by a
total of 500,000 barrels a day.
A. year, but only if non-OPEC nations, including Norway, Mexico, and Russia, were to trim output
B. year, but only if the output of non-OPEC nations, which includes Norway, Mexico, and Russia, is
trimmed
C. year only if the output of non-OPEC nations, including Norway, Mexico, and Russia, would be
trimmed
D. year only if non-OPEC nations, which includes Norway, Mexico, and Russia, were trimming output
E. year only if non-OPEC nations, including Norway, Mexico, and Russia, trim output

If something is still a future event, then use the future.

Tim told me yesterday that he would finish the book by this morning.
(...was the future then, but not now; "this morning" has come and gone.)
Tim predicted yesterday thay I will die at the age of 101.
(I'm not 101 yet. Still future.)

194. The Organization of Petroleum Exporting Countries (OPEC) had long been expected to
announce a reduction in output to bolster sagging oil prices, but officials of the
organization just recently announced that the group will pare daily production by 1.5
million barrels by the beginning of next year, but only if non-OPEC nations, including
Norway, Mexico, and Russia, were to trim output by a total of 500,000 barrels a day.

A. 'but ' is unnecessary and redundant. The sentence is a run on.

B. Same as A

C. Implies that Norway, Mexico etc are outputs of non-OPEC nations.

D. nations is plural; therefore 'includes' is incorrect

E. Correct

195. Unlike the original National Museum of Science and Technology in Italy, where the models are
encased in glass or operated only by staff members, the Virtual Leonardo Project, an online version
of the museum, encourages visitors to touch each exhibit, which thereby activates the animated
functions of the piece.
A. exhibit, which thereby activates
B. exhibit, in turn an activation of
C. exhibit, and it will activate
D. exhibit and thereby activate
E. exhibit which, as a result, activates

195. Unlike the original National Museum of Science and Technology in Italy, where the
models are encased in glass or operated only by staff members, the Virtual Leonardo
Project, an online version of the museum, encourages visitors to touch each exhibit,
which thereby activates the animated functions of the piece.

A. 'which' cannot be used to refer the action of touching

B. 'an activation of' is unidiomatic and also leaves the sentence incomplete

C. 'it' cannot refer to an action

D. Correct

E. implies that the exhibit activates the function - incorrect

197. The health benefits of tea have been the subject of much research; in addition to its
possibilities for preventing and inhibiting some forms of cancer, the brewed leaves of
Camellia sinensis may also play a role in reducing the risk of heart disease and stroke

A. 'its possibilities' is the wrong usage. 'Possibly' is required

B. Same as A

C. 'forms of cancer' is plural ; therefore 'prevents' is incorrect

D. It can prevent or inhibit. not both

E. Correct

197. The health benefits of tea have been the subject of much research; in addition to its possibilities for
preventing and inhibiting some forms of cancer, the brewed leaves of Camellia sinensis may also
play a role in reducing the risk of heart disease and stroke.
A. in addition to its possibilities for preventing and inhibiting
B. in addition to its possibilities to prevent or inhibit
C. besides the possibility that it prevents and inhibits
D. besides the possible preventing and inhibiting of
E. besides possibly preventing or inhibiting
189. The state has proposed new rules that would set minimum staffing levels for nurses, rules intended to
ensure that at least one nurse is assigned for every four patients put through triage in a hospital
emergency room.
A. rules intended to ensure that at least one nurse is assigned for every four patients put through triage in
a hospital emergency room
B. rules with the intent of ensuring one nurse at least to be assigned for every four patients to be put
through triage in a hospital emergency room
C. rules intending to ensure at least one nurse is assigned for every four patients in a hospital emergency
room put through triage
D. with the intent of ensuring that at least one nurse should be assigned for every four patients in a
hospital emergency room that are put through triage
E. and this is intended to ensure one nurse at least to be assigned for every four patients put through
triage in a hospital emergency room

188. The first commercially successful drama to depict Black family life sympathetically and
the first play by a Black woman to be produced on Broadway, it was Lorraine
Hansberrys A Raisin in the Sun that won the New York Drama Critics Circle Award in
1959, and was later made into both a film and a musical

A. 'The first ...' must modify the name of the play

B. Same as A

C. Same as A

D. Correct

E. 'and' parallelism missing. 'won' needs to be parallel to 'was made'

189. The state has proposed new rules that would set minimum staffing levels for nurses, rules
intended to ensure that at least one nurse is assigned for every four patients put through
triage in a hospital emergency room

A. Correct

B. Implies incorrectly that the rules had an intention

C. Same as B

D. 'with the intent ..' seems to modify nurses

E. 'this' cannot be used to refer to the proposal of rules

188. The first commercially successful drama to depict Black family life sympathetically and the first play
by a Black woman to be produced on Broadway, it was Lorraine Hansberrys A Raisin in the Sun that
won the New York Drama Critics Circle Award in 1959, and was later made into both a film and a
musical.
A. it was Lorraine Hansberrys A Raisin in the Sun that won the New York Drama Critics Circle Award
in 1959, and was later made
B. in 1959 A Raisin in the Sun, by Lorraine Hansberry, won the New York Drama Critics Circle Award
and was later made
C. Lorraine Hansberry won the New York Drama Critics Circle Award for A Raisin in the Sun in 1959,
and it was later made
D. Lorraine Hansberrys A Raisin in the Sun won the New York Drama Critics Circle Award in 1959
and was later made
E. A Raisin in the Sun, by Lorraine Hansberry, won the New York Drama Critics Circle Award in
1959, and later made it

The fact of some fraternal twins resembling each other greatly and others looking quite dissimilar highlights an interesting and often overlooked feature
of fraternal-twin pairs, namely they vary considerably on a spectrum of genetic relatedness.
A. The fact of some fraternal twins resembling each other greatly and others looking quite dissimilar highlights an interesting and often overlooked
feature of fraternal-twin pairs, namely they vary considerably
B. That some fraternal twins resemble each other greatly while others look quite dissimilar highlights an interesting and often overlooked feature of
fraternal-twin pairs, namely that they vary considerably
C. With some fraternal twins resembling each other greatly and others looking quite dissimilar, it highlights an interesting and often overlooked feature
of fraternal-twin pairs, namely considerable variation
D. With some fraternal twins resembling each other greatly and others looking quite dissimilar, it is a fact that highlights an interesting and often
overlooked feature of fraternal-twin pairs, namely a considerable variation
E. Because some fraternal twins resemble each other greatly and others look quite dissimilar, this fact highlights an interesting and often overlooked
feature of fraternal-twin pairs, namely they vary considerably

Can someone help to explain step-by-step method to get the right answer?

See Pic below for proof of question from GMATPrep.

here are some splits.

* "namely" must be used either before a noun or before a noun phrase / clause.
you can't use it directly before an independent clause.
because of this principle, you can't say "namely they vary...". however, "namely that they vary..." and "namely considerable variation" are both fine.
this kills (a) and (e).

* "the fact of X doing Y" is universally incorrect.


this kills (a).

* the sentence openers starting with "with" are incorrect. to work properly, they'd have to be followed by a subject that's actually "with" those things.
example: with five all-state players in its backfield, ballard high looks to shut down opposing offenses completely --> this makes sense, because
ballard high actually has five all-state players in its backfield, thus justifying the use of "with". there's no corresponding sense of possession / belonging
here.
this kills (c) and (d).

* the structure of choice (e) indicates that "this fact" is something other than the cited fact about fraternal twins' resemblance.
that kills (e).

* "it" is a pronoun with no antecedent in (c). (by contrast, in (d), "it" is fine because it's part of the special construction "it is a fact that...", which
behaves in the same manner as constructions such as "it is surprising that...").

RonPurewal wrote:
* "the fact of X doing Y" is universally incorrect.
this kills (a).

Hi Ron,

Can you please tell more about this rule ?

Does it mean that "the fact" is always wrong in GMAT ?

if no, then can you please tell under what circumstances "the fact" is wrong and under what circumstances it is correct ?

Thanks
Mohit

you should use "the fact that...", followed by a normal independent clause.

i.e.,
i was surprised by the fact that no one has ever won this game.

(note that "no one has ever won this game" would be an independent clause by itself)

herogmat wrote:Ron, can you give an example about the wrong usage of 'this' ?

anything in which "this" is used as a noun is incorrect. if you see "this is..."; "this shows that"; "this VERB" in general, then it's wrong.

the only acceptable use of "this" in formal written english is as an adjective: this fact, this predator, this notion, etc.
Authoritative parents are more likely than permissive parents to have children who as adolescents are self-confident, high in self-esteem, and
responsibly independent.

A.Authoritative parents are more likely than permissive parents to have children who as adolescents are self-confident, high in self-esteem, and
responsibly independent.
Only left
Though, I would prefer 'that' in place of 'who' in this case, if 'that' were given as an option.
B.Authoritative parents who are more likely than permissive parents to have adolescent children that are self-confident, high in self-esteem, and
responsibly independent.
incomplete sentence
Authoritative parents (who are...) ...no verb!!
C.Children of authoritative parents, rather than permissive parents, are the more likely to be self-confident, have a high self-esteem, and to be
responsibly independent as adolescents.
wrong comparison
'the more' seems inappropriate
D.Children whose parents are authoritative rather than being permissive, are more likely to have self-confidence, a high self-esteem, and be
responsibly independent when they are an adolescent.
not parallel
E.Rather than permissive parents, the children of authoritative parents are the more likely to have self-confidence, a high self-esteem, and to be
responsibly independent as an adolescent.
wrong comparison
'the more' seems inappropriate here. Opinions?

no, no mistake.

there are two kinds of parallel signals: ONE-PART (such as "and", "or", "but"), and TWO-PART (such as "not only ... but also", "both ... and").

when you have PARALLELISM WITH A ONE-PART SIGNAL, the only words that are "locked in" are the ones directly FOLLOWING the signal.
as long as you can find the corresponding structure in the other part, then the parallelism is fine.

examples:
i worked in nevada and florida.
i worked in nevada and in florida.

BOTH OF THESE ARE FINE.

reasons:
in the first, the part that's "locked in" by the signal and is just florida. therefore, the parallel construction would be just nevada.
since that construction is there, the sentence is parallel:
i worked in
nevada
and
florida.

in the second, the part that's "locked in" by the signal and is in florida. therefore, the parallel construction would be just in nevada.
since that construction is there, the sentence is parallel:
i worked
in nevada
and
in florida.

--

for completely analogous reasons, this sentence would be fine either with or without your second "that":

an increase that
would amount to roughly five miles per gallon
and
would represent...

an increase
that would amount to roughly five miles per gallon
and
that would represent...
Pueden hacerle preguntas a

181. The results of two recent unrelated studies support the idea that dolphins may share certain
cognitive abilities with humans and great apes; the studies indicate dolphins as capable of
recognizing themselves in mirrorsan ability that is often considered a sign of self-awareness
and to grasp spontaneously the mood or intention of humans.
A. dolphins as capable of recognizing themselves in mirrorsan ability that is often considered a
sign of self-awarenessand to grasp spontaneously
B. dolphins ability to recognize themselves in mirrorsan ability that is often considered as a sign
of self-awarenessand of spontaneously grasping
C. dolphins to be capable of recognizing themselves in mirrorsan ability that is often considered a
sign of self-awarenessand to grasp spontaneously
D. that dolphins have the ability of recognizing themselves in mirrorsan ability that is often
considered as a sign of self-awarenessand spontaneously grasping
E. that dolphins are capable of recognizing themselves in mirrorsan ability that is often
considered a sign of self-awarenessand of spontaneously grasping

181. The results of two recent unrelated studies support the idea that dolphins may share
certain cognitive abilities with humans and great apes; the studies indicate dolphins as
capable of recognizing themselves in mirrorsan ability that is often considered a
sign of self-awarenessand to grasp spontaneously the mood or intention of humans .

A. indicate requires a 'that' after it

B. and C. Same as A

D. 'considered as' and 'ability of' are both incorrect idioms

E. Correct

180. Charles Lindbergh, for his attempt at a solo transatlantic flight, was very reluctant to have any
extra weight on his plane, he therefore refused to carry even a pound of mail, despite being
offered $1,000 to do so.
A. Charles Lindbergh, for his attempt at a solo transatlantic flight, was very reluctant to have any
extra weight on his plane, he therefore
B. When Charles Lindbergh was attempting his solo transatlantic flight, being very reluctant to have
any extra weight on his plane, he
C. Since he was very reluctant to carry any extra weight on his plane when he was attempting his
solo transatlantic flight, so Charles Lindbergh
D. Being very reluctant to carry any extra weight on his plane when he attempted his solo
transatlantic flight was the reason that Charles Lindbergh
E. Very reluctant to have any extra weight on his plane when he attempted his solo transatlantic
flight, Charles Lindbergh

180. Charles Lindbergh, for his attempt at a solo transatlantic flight, was very reluctant to
have any extra weight on his plane, he therefore refused to carry even a pound of
mail, despite being offered $1,000 to do so

A. run on - 2 independent clauses separated by comma

B. 'being very reluctant ..' does not modify Charles Lindbergh correctly.

C. ' Since he was ..' needs to modify 'Charles Lindbergh'; therefore needs the name immediately after the comma.

D. 'Being very reluctant' cannot be referred to as a reason.

E. Correct

178. In California today, Hispanics under the age of eighteen account for more than 43
percent, compared with a decade ago, when it was about 35 percent

A. Comparing 'Hispanics under 18' to a decade - incorrect.

B. 'it' does not have a clear antecedent

C. Correct

D. Same as B

E. Same as B

178. In California today, Hispanics under the age of eighteen account for more than 43 percent,
compared with a decade ago, when it was about 35 percent.
A. In California today, Hispanics under the age of eighteen account for more than 43 percent,
compared with a decade ago, when it was about 35 percent.
B. Of the Californians under the age of eighteen, today more than 43 percent of them are Hispanic,
compared with a decade ago, when it was about 35 percent.
C. Today, more than 43 percent of Californians under the age of eighteen are Hispanic, compared
with about 35 percent a decade ago.
D. Today, compared to a decade ago, Californians who are Hispanics under the age of eighteen
account for more than 43 percent, whereas it was about 35 percent.
E. Today, Hispanics under the age of eighteen in California account for more than 43 percent,
unlike a decade ago, when it was about 35 percent.

BG wrote:Why is A wrong?
Why is E wrong?

both of these choices are wrong for the same reason: neither of them specifies what we're taking percentages of.
you need to say "43% of californians", or "43% of the population of california", or 43% of... something.

In C,
"More than 43% of Californians under the age of eighteen are Hispanic" has different meaning with" Hispanics under the age of eighteen account for
more than 43%". Which one is correct?

the first is correct, because it actually means something. see above.


also, as remarked above, you know that the first one is correct, because it's in the correct answer.

--

if you append "...of california's population" onto the end of the second one, so that it actually becomes meaningful, then it says something quite
different from the meaning of the first one. post back if you don't see the difference.

"Compared" follows "Hispanic" immediately, does it mean "compared" modify "Hispanic"?

first: you know it can't, because that's the correct answer. if "hispanic" were erroneously compared to a percentage, then this couldn't be the correct
answer.
apparently, "compared with"/"compared to" can be used to modify the statistic in the preceding clause, even if that statistic doesn't come right before
the comma. this is a lesson that you can draw by yourself from looking at this problem, simply by noting what appears in the correct answer.

In my view, "compared" should modify "43%".

well, sure. it must, for the sentence to make any sense at all.
so clearly the lesson you should learn here is that this particular type of modifier is more flexible than you had thought. you should
obviously not conclude that the correct answer contains an ungrammatical modifier.
remember: correct answers are correct.

"are Hispanic" , "are Hispanics", "is Hispanic" or " is Hispanics". Which one is correct?

oh boy.
actually, ALL of these can be correct, in the proper context, although the last is a bit awkward.

the first two, of course, must go with something plural. in this case, that would be "43% of californians".
they both make sense, though; the only difference is that "hispanic" is an adjective while "hispanics" is a plural noun. the difference is akin to the
distinction between "43% of the pills seized were yellow" and "43% of the pills seized were yellow pills".

the last two must go with something singular, which, in this case, would be "43% of the population".
they both make sense, but "is Hispanics" is too awkward in my opinion; it would be better written as "consists of hispanics".

172. In 2000, a mere two dozen products accounted for half the increase in spending on prescription
drugs, a phenomenon that is explained not just because of more expensive drugs but by the fact
that doctors are writing many more prescriptions for higher-cost drugs.
A. a phenomenon that is explained not just because of more expensive drugs but by the fact that
doctors are writing
B. a phenomenon that is explained not just by the fact that drugs are becoming more expensive but
also by the fact that doctors are writing
C. a phenomenon occurring not just because of drugs that are becoming more expensive but because
of doctors having also written
D. which occurred not just because drugs are becoming more expensive but doctors are also writing
E. which occurred not just because of more expensive drugs but because doctors have also written

172. In 2000, a mere two dozen products accounted for half the increase in spending on
prescription drugs, a phenomenon that is explained not just because of more
expensive drugs but by the fact that doctors are writing many more prescriptions for
higher-cost drugs

A. ' Not by X but by Y' is the correct idiom

B. Correct

C. Changes meaning - implies that the occurrence is due to drugs and doctors.

D. 'which' refers to drugs - incorrect

E. Same as D

168. The Quechuans believed that all things participated in both the material level and the mystical
level of reality, and many individual Quechuans claimed to have contact with it directly with an
ichana (dream) experience.
A. contact with it directly with
B. direct contact with it by way of
C. contact with the last directly through
D. direct contact with the latter by means of
E. contact directly with the mystical level due to

168. The Quechuans believed that all things participated in both the material level and the
mystical level of reality, and many individual Quechuans claimed to have contact
with it directly with an ichana (dream) experience.

A. 'it ' has an ambiguous reference

B. Same as A

C. 'contact directly' means something different from 'direct contact'.

D. Correct

E. 'Due to' implies cause - changing the meaning. The contact is through ichana not because of it.

165. Twenty-two feet long and 10 feet in diameter, the AM-1 is one of the many new satellites that is
a part of 15 years effort of subjecting the interactions of Earths atmosphere, oceans, and land
surfaces to detailed scrutiny from space.
A. satellites that is a part of 15 years effort of subjecting the interactions of Earths atmosphere,
oceans, and land surfaces
B. satellites, which is a part of a 15-year effort to subject how Earths atmosphere, oceans, and land
surfaces interact
C. satellites, part of 15 years effort of subjecting how Earths atmosphere, oceans, and land surfaces
are interacting
D. satellites that are part of an effort for 15 years that has subjected the interactions of Earths
atmosphere, oceans, and land surfaces
E. satellites that are part of a 15-year effort to subject the interactions of Earths atmosphere, ocean,
and land surfacesjust

think about the meaning.

the sentence is clearly intended to say that (a) this effort comprises, among other things, a bunch of satellites, and that (b) am-1 is one of them.

think about itif you think the sentence should say "is a part", then it would be pointless and wasteful to write "one of the..." in the first place.
in that case, the sentence would just say "am-1 is a new satellite that is part of xxxxx".

165. Twenty-two feet long and 10 feet in diameter, the AM-1 is one of the many new
satellites that is a part of 15 years effort of subjecting the interactions of Earths
atmosphere, oceans, and land surfaces to detailed scrutiny from space

A. 'that is' cannot refer to plural satellites

B. 'which is' cannot be used to refer to satellites

C. 'of subjecting how' is an incorrect idiom

D. 'has subjected' tense implies that the action is complete; therefore incorrect

E. Correct
163. Although the restaurant company has recently added many new restaurants across the country
and its sales have increased dramatically, its sales at restaurants open for more than a year have
declined.
A. the restaurant company has recently added many new restaurants across the country and its sales
have increased dramatically, its
B. the restaurant company has recently added many new restaurants across the country and its sales
increased dramatically, its
C. many new restaurants have recently been opened across the country and its sales increased
dramatically, the restaurant companys
D. having recently added many new restaurants across the country and with its sales increasing
dramatically, the restaurant companys
E. recently adding many new restaurants across the country and having its sales increase
dramatically, the restaurant companys

163. Although the restaurant company has recently added many new restaurants across the
country and its sales have increased dramatically, its sales at restaurants open for
more than a year have declined.

A. Correct

B. No Parallelism - 'has added' needs 'have increased'

C. 'Its' has no antecedent

D. 'Although having ..' needs to modify 'The restaurant company'

E. Same as D

look at the sentence from this point of view:

A new hair-growth drug is being sold for three times the price, ....................... the drugs maker charges for another product with the same
active ingredient.

so we need a word that clearly connects two parts, modifying price.


the correct answer here is that (C)

the price that the ....

A new hair-growth drug is being sold for three times the price, per milligram, _______ the drugs maker charges for another product with the same
active ingredient.

A.) as - if we put 'as' in the blank then we need another clause for comparing the price of hair-growth drug. A new drug is sold for 3 times the
price, as the maker charges for another product.

A new hair-growth drug is being sold for three times the price, per milligram, as the drugs maker charges for another product with the same active
ingredient.

A.) as

B.) than

C.) that

D.) of what

E.) at which

In Greek theology the supreme being was Esaugetu Emissee (Master of Breath), who dwelt in an upper realm in which the sky was the floor, and
who had the power to give and to take away the breath of life.

A. in which the sky was the floor, and who had the power to give and to take

B. where the sky was the floor, having the power to give and to take

Suggests the upper realm has a place where the sky becomes the floor as opposed to the upper realm being characterised by this quality.

C. whose floor was the sky, and who has the power of giving and of taking

No need of present "has". The sentence is in past.


D. in which the sky was the floor, with the power of giving and taking

the prepo phrase "with the power.." modifies floor giving an illogical meaning

E. whose floor was the sky, having the power to give and take

the sky didn't have the powers in question

upper realm is not a place - so we cannot use "where" . in which sounds more appropriate.
we also need a parallel construction (and who) to define the other things the supreme being could do

Had is being used here to mean, possessed, owned, a simple past tense of have. (It is not a past participle used in past perfect tense here.) The
whole text is used in past tense and we require a simple past tense once again in the second arm; hence had

158. Minivans carry as many as seven passengers and, compared with most sport utility
vehicles, cost less, get better gas mileage, allow passengers to get in and out more
easily, and have a smoother ride

A. Correct

B. Incomplete sentence with no primary verb. (remove 'which ..' . Leaves sentence with only 'Minivan.')

C. Run on. 2 independent clauses separated by comma

D. Changes meaning - implies only Minivans with 7 passengers do the following

E. No Parallelism - 'cost is lower' and 'they get better' is not parallel

158. Minivans carry as many as seven passengers and, compared with most sport utility vehicles, cost
less, get better gas mileage, allow passengers to get in and out more easily, and have a smoother
ride.
A. Minivans carry as many as seven passengers and, compared with most sport utility vehicles, cost
less,
B. Minivans, which carry as many as seven passengers, compared with most sport utility vehicles,
they cost less,
C. Minivans carry as many as seven passengers, in comparison with most sport utility vehicles, and
have a lower cost, they
D. Minivans, carrying as many as seven passengers, compared with most sport utility vehicles, cost
less,
E. Minivans, which carry as many as seven passengers, compared with most sport utility vehicles
the cost is lower, and they

2) are the following three clauses properly linked together, if not then what is wrong - those in department stores, do very little impulse shopping, do
not buy a pair of skis and a boomerang when they come in for a basketball, but leave with only a basketball

1) there's no such construction as "A, B, but C". i.e., "but" can't link three things in the way that "and" or "or" could.

2) it makes no sense to put all 3 of these things in parallel, because they don't belong in parallel; the last two are an explanation of the first one. i.e., the
last two things are an explanation of the idea that these shoppers "do very little impulse shoppin

157. Shoppers in sporting goods stores, unlike in department stores, do very little impulse
shopping, not buying a pair of skis and a boomerang when they come in for a
basketball, but they leave with a basketball only .

A. Comparing shoppers to stores is incorrect

B. Same as A

C. Incorrect sentence structure. do X, Do Y , but Z - incorrect - 'and' is missing

D. Correct
E. Same as A

157. Shoppers in sporting goods stores, unlike in department stores, do very little impulse shopping,
not buying a pair of skis and a boomerang when they come in for a basketball, but they leave
with a basketball only.
A. in department stores, do very little impulse shopping, not buying a pair of skis and a boomerang
when they come in for a basketball, but they leave with a basketball only
B. in department stores, shop impulsively very little; someone who comes in for a basketball will
leave with a basketball only and not also buy a pair of skis and a boomerang as well
C. those in department stores, do very little impulse shopping, do not buy a pair of skis and a
boomerang when they come in for a basketball, but leave with only a basketball
D. those in department stores, do very little impulse shopping; someone who comes in for a
basketball will leave with a basketball only and not buy a pair of skis and a boomerang as well
E. department stores, shop impulsively very little; someone will not buy a pair of skis and a
boomerang when they come in for a basketball but will leave with only a basketball

156. Almost a decade after New York State passed laws to protect patients by reducing the grueling
hours worked by medical residents, twelve hospitals have been investigated by state medical
officials, finding that all twelve consistently break the laws, many residents work longer than 24
hours straight, and that more than half the surgical residents work more than 95 hours a week.
A. twelve hospitals have been investigated by state medical officials, finding that all twelve
consistently break the laws, many residents work longer than 24 hours straight, and that more
than half the surgical residents work
B. an investigation by state medical officials of twelve hospitals have found all twelve consistently
breaking the laws, that many residents work longer than 24 hours straight, with more than half
the surgical residents working
C. an investigation of twelve hospitals by state medical officials has found that all twelve
consistently break the laws, that many residents work longer than 24 hours straight, and that
more than half the surgical residents work
D. twelve hospitals were investigated by state medical officials who found all twelve breaking the
laws, with many residents working longer than 24 hours straight, and more than half the surgical
residents work
E. an investigation by state medical officials has found that, of twelve hospitals, all twelve
consistently break the laws, that many residents work longer than 24 hours straight, with more
than half the surgical residents working

156. Almost a decade after New York State passed laws to protect patients by reducing the
grueling hours worked by medical residents, twelve hospitals have been investigated
by state medical officials, finding that all twelve consistently break the laws, many
residents work longer than 24 hours straight, and that more than half the surgical
residents work more than 95 hours a week

A. The comma must be followed by 'an investigation' - the investigation happened after 12 years not the hospitals

B. 'an investigation' is singular - 'have' is incorrect

C. Correct

D. Same as A

E. 'and' is missing. Found that .. , that, with . Incorrect structure

151. On account of a law passed in 1993, making it a crime punishable by imprisonment that a United
States citizen hold gold in the form of bullion or coins, immigrants found that on arrival in the
United States they had to surrender all of the gold they had brought with them.
A. On account of a law passed in 1993, making it a crime punishable by imprisonment that a United
States citizen hold
B. With a law passed in 1933 that makes it a crime punishable by imprisonment that a United States
citizen hold
C. A law passed in 1933 that made it a crime punishable by imprisonment for a United States citizen
holding
D. Because of a law passed in 1933 making it a crime punishable by imprisonment for a United
States citizen to hold
E. Due to a law being passed in 1933 that makes it a crime punishable by imprisonment for a United
States citizen to hold151. On account of a law passed in 1993, making it a crime punishable by imprisonment
that a United States citizen hold gold in the form of bullion or coins, immigrants
found that on arrival in the United States they had to surrender all of the gold they
had brought with them.

A. Incorrect idiom 'crime that'. 'crime for' is needed

B. 'With ..' does not imply the desired cause-effect meaning


C. 'holding' is the wrong tense. 'to hold' is required. Makes an incomplete sentence

D. Correct

E. 1993 is the past - therefore 'being passed' is the wrong tense. Also, 'Because of' is preferred to 'due to'

C is wrong because of idiom issue.

crime punishable by imprisonment for a United States citizen holding -- is incorrect use of idiom.

It should be

crime punishable by imprisonment for a United States citizen to hold

The crime "to hold" such things is punishable.

There is one more problem with C, although I would like the instructors to correct me if I m wrong

"holding gold in the form of bullion or coins" just after the noun modifies the noun and therefore the sentence should stand without the adjectival
modifier. If we remove the modifier in 'C' it reads --

A law passed in 1933 that made it a crime punishable by imprisonment for a United States citizen, ....

It again doesnt make any sense.

not a grammar issue, but it's a modifier that doesn't make sense.

if you have that kind of modifier ("A law..."), then it should provide a description of the noun that follows it.
e.g.,
A famous and influential boxer, Manny Pacquiao has plans for a career in Philippine politics after he retires from boxing.
here, the modifier "a famous and influential boxer" modifies "manny pacquiao", so it's fine.

here, though, that usage would suggest that immigrants themselves (= the subject following the comma) are "a law". that is nonsense and thus
incorrect.

--

one other thing:


you referred to part of a sentence as a "fragment".
this description doesn't make sense -- "fragment" is only used for something that's supposed to be a whole sentence but is, in fact, only part of one.
Pueden hacerle

ah yes, the dreaded "because of X VERBing..." construction.

this construction is incorrect if the causal agent (i.e., the thing that the "because" is attributed to) is the VERB.

for instance,

* we laughed all day because of tony walking into the girls' bathroom --> incorrect, because the verb (walking into the bathroom) is the reason why we
were laughing all day.
the correct version of this sentence would read we laughed all day because of tony's walking into the girls' bathroom. in other words, if the causal
agent is the action (which appears in -ing form), then it must be preceded by a possessive.

* estelle was afraid to move because of a spider sitting on the wall --> correct, because estelle is afraid because of the spider itself, not because of the
spider's sitting on the wall.

this sentence should be read as an instance of the latter: it's the law itself that caused the immigrants' problems.

there you go.

goelmohit2002 wrote:Can't we kick out E...because we cannot replace due to by "caused by" here ?

Replacing Due to with "caused by" in E makes no sense....As per my understanding....if in a sentence we can replace due to with caused by...then only
usage of due to is correct...

Please correct me if my understanding is wrong...


absolutely correct. you can always replace "due to" with "caused by".

technically, then, this statement would be declaring that immigrants themselves were caused by the law.

therefore, yes, (e) is wrong for this reason

well, there are TWO essential modifiers in that sentence, both of which are traditionally placed after the noun. (both of them are participial modifiers -
one a present participle, one a past participle)
* passed in 1933
* making it a crime...
since we can't place both of these modifiers directly after the noun, we have to place one of them after the other.
since "passed in 1933" is the shorter of the two, we elect to place that one after the noun.**

--

**this is often the "rule" that's used for the placement of two items that have the same grammatical priority - i.e., you don't know which one to place
first, since they're both things that go in the same place.

for instance:
i dedicated a song to my father
i dedicated to my father a song that recounted all the lessons he taught me in life

both of these are correct constructions; if you reverse the placement in the second one, it becomes too confusing / difficult to read.

The OA is D indeed... I picked A didn't see any idioms as it wasn't the focus of my h/w... and I am not good with idioms anyways...

I wanted to briefly disscuss the CONDITIONALS and the TENSES that practically give the wrong answers away... you might know it already, but
review is always helpful (feel free to add what you know on this matter)!
_____________________________________________________________
Conditional Sentences are known as Conditional Clauses or "If" Clauses. They are used to express that the action in the main clause (without if) can
only take place if a certain condition (in the clause with if) is fulfilled.

There are 3 Types of those:

Type 1
It is possible and also very likely that the condition will be fulfilled. We don't know for sure whether the condition actually will be fulfilled or not,
but the conditions seems rather realistic so we think it is likely to happen.

Formula: if + Simple Present, will-Future

Example: If I go to Ireland, I will visit Dublin.

Type 2
It is possible but very unlikely, that the condition will be fulfilled. Conditional Sentences Type II refer to situations in the present. An action could
happen if the present situation were different. I don't really expect the situation to change, however. I just imagine what would happen if

Formula: if + Simple Past, Conditional I (= would + Infinitive)

Example: If I went to Ireland, I would visit Dublin. (I felt this one sounded kind of awkward, but I guess gramatically correct)

Type 3
It is impossible that the condition will be fulfilled because it refers to the past. Conditional Sentences Type III refer to situations in the past. An
action could have happened in the past if a certain condition had been fulfilled. Things were different then, however. We just imagine, what would
have happened if the situation had been fulfilled.

Formula: if + Past Perfect, Conditional II (= would + have + Past Participle)

Example: If I had gone to Ireland, I would have visited Dublin.

REMEMBER:
1)"WOULD/WILL" never appears in the "IF" clause and you can easily eliminate choices based on that fact!!!
2) If the tense is PRESENT SIMPLE, then, the condition is likely to be either PAST SIMPLE or FUTURE SIMPLE, NOT anything else, meaning the tenses
should be parallel (If PAST PERFECT -> Then PRESENT PERFECT) etc
3) Do NOT go by "what sounds right"... YOU WILL get it wrong... So if this subject is your weak area (like it was mine) never go by this rule!
4) If I WERE blah..., the blah... many know this rule (i didn't, it thought it was SVA )

if you want practice quizzes, here is the link:


http://www.englisch-hilfen.de/en/exercises_list/if.htm

please add more...


DISCLAIMER: some of you know that I am doing GMAX, but this is NOT their material... this is my own efforts of Google search to get more info on
the subject!
72. Many environmentalists, and some economists, say that free trade encourages industry to relocate to countries
with ineffective or poorly enforced antipollution laws, mostly in the developing world, and that, in order to
maintain competitiveness, rich nations have joined this downward slide toward more lax attitudes about
pollution.
A. that, in order to maintain competitiveness, rich nations have joined this downward slide toward more lax attitudes
about pollution
B. that, for maintaining competitiveness, rich nations join in this downward slide toward more lax attitudes about
pollution
C. that rich nations join this downward slide toward more lax attitudes about pollution because of wanting to
maintain competitiveness
D. that in rich nations, joining this downward slide toward more lax attitudes about pollution is a result of wanting to
maintain competition
E. that wanting to maintain competition is making rich nations join in this downward slide toward an attitude about
pollution that is more lax
B: to maintain is the right idiom (to show intention)
C: nations join means present simple tense shows eternal truth not a recent phenomenon. Because of wanting
is worse than Because they want
D, E are plain BAD. The major trouble is WANTING to maintain.
here are a couple of problems with choice b:
- 'for maintaining competitiveness' is bad idiomatic usage: it's an incorrect way to express intent or purpose. (the
construction in choice a is surprisingly wordy - in order to could be replaced with just to, with no loss of meaning -
and might not make it onto the real exam.)
- 'rich nations join in this downward slide': fatal change in meaning. this wording implies that the rich nations are
colluding with each other, metaphorically holding hands, deciding together to become more lax about pollution.
(choice a expresses the correct meaning, which is that the slide is happening and that rich nations are individually
joining it.)
- the simple present tense join implies that there is some sort of timeless truth about the statement being made.
that's the wrong meaning: the sentence is meant to say that the rich nations have begun to do this as a result of
current trends. if you don't understand the difference, consider the following analogous examples:
private universities have admitted more females than males --> meaning: this is an ongoing or cumulative trend,
continuing into the present from some designated starting point
private universities admit more females than males --> meaning: this is a fundamental truth about the way private
universities operate

Though now eaten in large quantities around the world and harmless, the tomato is a member of the generally toxic nightshade family, including
belladonna, and was once thought to be poisonous itself as a result.
A) Though now eaten in large quantities around the world and harmless, the tomato is a member of the generally toxic nightshade family, including
belladonna, and was once thought to be poisonous itself as a result.
Modifier phrase + main clause + noun modifier . Noun modifier or adjective almost always follow touch rule so
nightshade family cannot be subject because it is already object of main clause so subject is missing for modifier so
it is dangling modifier.
B) The tomato, though now eaten in large quantities around the world and harmless, is a member of the generally toxic nightshade family, which
includes belladonna, and it was therefore once thought to be poisonous itself.
No clear referent of it
C) Once thought to be poisonous itself, the tomato is harmless and now eaten in large quantities around the world, and is a member of the
generally toxic nightshade family, including belladonna.

Tomato cannot think itself as poisonous.


D) Though now eaten in large quantities around the world and known to be harmless, the tomato was once considered poisonous because it is a
member of the generally toxic nightshade family, which includes belladonna.
Here Though now eaten in large quantities around the world and known to be harmless modifying tomato correctly.
E) A member of the generally toxic nightshade family, including belladonna, the tomato was once considered poisonous even though it is harmless
and now eaten in large quantities around the world.

Including Belladonna is modifying Tomato here. You can see that.Please remove first modifier as we add modifier
to describe a particular noun but it not essential part of sentence unless it is essential modifier.
The remaining sentence is Including belladonna, the tomato was once considered. So you can clearly see that
including belladonna is modifying Tomato and that is wrong in meaning.

I hope my explanation helps.

57. Though now eaten in large quantities around the world and harmless, the tomato is a member of the generally
toxic nightshade family, including belladonna, and was once thought to be poisonous itself as a result.
A. Though now eaten in large quantities around the world and harmless, the tomato is a member of the generally
toxic nightshade family, including belladonna, and was once thought to be poisonous itself as a result.
B. The tomato, though now eaten in large quantities around the world and harmless, is a member of the generally
toxic nightshade family, which includes belladonna, and it was therefore once thought to be poisonous itself.
C. Once thought to be poisonous itself, the tomato is harmless and now eaten in large quantities around the world,
and is a member of the generally toxic nightshade family, including belladonna.
D. Though now eaten in large quantities around the world and known to be harmless, the tomato was once
considered poisonous because it is a member of the generally toxic nightshade family, which includes belladonna.
E. A member of the generally toxic nightshade family, including belladonna, the tomato was once considered
poisonous even though it is harmless and now eaten in large quantities around the world.
The original sentence contains a lot of clauses in a confusing order. We need to find an answer choice that rephrases
the sentence in a clear and concise manner. Also, the phrase "though now eaten in large quantities around
the world and harmless" contains two elements that are not parallel. Moreover, "including belladonna" is
incorrect left dangling. It should be "which includes belladonna." Finally, "itself" could refer either to "tomato" or
to "belladonna". (A) This choice is incorrect as it repeats the original sentence. (B) The pronoun "it" is
ambiguous as it could refer either to "the nightshade family" or "belladonna" or "tomato." Additionally, the
phrase "though now eaten in large quantities around the world and harmless" contains two elements that are not
parallel. (C) The phrase "including belladonna" does not properly modify anything. (D) CORRECT. The
opening phrase contains the two parallel elements "eaten in large quantities . . ." and "known to be
harmless." The phrase "which includes belladonna" correctly modifies the "nightshade family." Finally, the
pronoun "it" unambiguously refers to the tomato. (E) The phrase "including belladonna" does not properly modify

50. A wildlife expert predicts that the reintroduction of the caribou into northern Minnesota would fail if the density
of the timber wolf population in that region is more numerous than one wolf for every 39 square miles.
A. would fail if the density of the timber wolf population in that region is more numerous
B. would fail provided the density of the timber wolf population in that region is more
C. should fail if the timber wolf density in that region was greater
D. will fail if the density of the timber wolf population in that region is greater
E. will fail if the timber wolf density in that region were more numerous
A prediction is made about a future event, which
can be restated using the construction y will
happen if x happens fi rst (an alternate form is if x
happens, y will happen). Th e subjunctive verb form
would expresses remoteness and thus casts some
doubt on the likelihood of x. Th e verb form will
fail expresses the likelihood x with greater
certainty. Density is not a countable quantity, so it
cannot be modifi ed by more numerous, which is
used solely for countable quantities; greater is
correct.
A Would fail expresses doubt about the future
outcome; density should be modifi ed by
greater.
B Th is construction requires will fail, not
would fail; if is preferred to provided; density
should be modifi ed by greater.
C Will fail, not should fail, is required; timber
wolf density does not clearly refer to the
population; the tense of the fi nal verb is
incorrect.
D Correct. Th e verb form will fail better
expresses the certainty of the prediction;
density is appropriately modifi ed by greater.
E Timber wolf density does not clearly refer to
the population; the tense and number of the
fi nal verb are incorrect; density cannot be
modifi ed by numerous.
Th e correct answer is D.

OG 12 q 73 Tue Jun 16, 2009 7:49 am

Quote
Elapsed Time: 00:00
start
lap
stop
Combining enormous physical strength with higher intelligence,the neanderthals appear as equipped for facing any obstacle the environment could put
in their path,but their relatively sudden disappearnce during the paleolithic era indicates that an inability to adapt to some environmental change led o
their extinction.

A)appear as equipped for facing any obstacle the environment could put in their path
B)appear to have been equipped to face any obstacle the environment could put in their path,
C)appear as equipped to face any obstacle the environment could put in their paths,
D)appeared as equipped to face any obstacle the environment could put in their path,
E)appeared to have been equipped for facing any obstacle the environment could put in their path,
OA:B

Need free GMAT or MBA advice from an expert? Register for Beat The GMAT now and post your question in these forums!

RobinmrthaRising GMAT Star


Follow user
View User Profile
Joined

01 Apr 2009
Posted:

91 messages
Thanked:

10 times
Wed Jun 17, 2009 3:10 am
Quote
Appear as is incorrect...
It should be appear to...
E is wordy and awkward...
So B is correct

pink_08Rising GMAT Star


Follow user
View User Profile
Joined

31 Dec 2008
Posted:

87 messages
Mon Aug 03, 2009 4:41 pm
Quote
Could anyone give an explanation why (B) is the correct answer.

Specifically, why " have been equipped " is correct usage. have been is present perfect which dictates any action started in the past but continues in
the present. Since Neanderthal are extinct, how is this usage right ?

Also appear is in the present tense, Is it used bcos of the idiom "appear to " (or) because the modifier at the start of the sentence is present tense.

B)appear to have been equipped to face any obstacle the environment could put in their path

pandeyvineet24GMAT Destroyer!
Follow user
View User Profile
Joined

02 Jun 2008
Posted:

527 messages
Thanked:

16 times
Target GMAT Score:

720
Mon Aug 03, 2009 5:08 pm
Quote
Why B is the correct answer here. B should not be correct here simply because it uses present perfect.

I picked D
Thanked by: agarwalmanoj2000

pink_08Rising GMAT Star


Follow user
View User Profile
Joined

31 Dec 2008
Posted:

87 messages
Mon Aug 03, 2009 5:51 pm
Quote
OG answer is B.

GMAT/MBA EXPERT

lunarpowerGMAT Instructor
Follow user
View User Profile
Joined

03 Mar 2008
Posted:

3377 messages
Followed by:

1356 members
Thanked:

2167 times
GMAT Score:

800
Tue Aug 04, 2009 1:41 am
Quote
this isn't a present perfect construction, because it's actually an infinitive.
i.e., it's not "they have been equipped"; it's "they appear TO HAVE been equipped".

in any case, you should probably just memorize this construction as a one-off idiomatic structure. if you say "they seemed/appeared to...", then, no
matter how remote the event is (in time), you use this construction.

for instance:
it seems that the students cheated on the exam
(normal past tense in this construction)
but...
the students seem to have cheated on the exam
(not here)

i don't really have a good explanation for this, other than "i'm a writer with a firm command of formal english, and i know that it is so" and "you should
just think of it as an idiom". sorry i can't do better than that.

in any case, though, you DO have to use an infinitive after "appear" or "seem" in this sort of construction. and if you think about it, this is as past-
tense as an infinitive can get. so that's why you have to use it.

_________________
Ron is a Director of Curriculum Development at Manhattan GMAT. He has been teaching various standardized tests for 20 years.

--

Pueden hacerle preguntas a Ron en castellano


Potete chiedere domande a Ron in italiano
On peut poser des questions Ron en franais
Voit esitt kysymyksi Ron:lle mys suomeksi

--

Quand on se sent bien dans un vtement, tout peut arriver. Un bon vtement, c'est un passeport pour le bonheur.

Yves Saint-Laurent

--
Learn more about ron
Thanked by: Janice Chia

Free Manhattan Prep online events - The first class of every online Manhattan Prep course is free. Classes start every week.

viju9162GMAT Destroyer!
Follow user
View User Profile
Joined

11 Jun 2007
Posted:

434 messages
Thanked:

6 times
Target GMAT Score:

750+
GMAT Score:

600
Tue Aug 04, 2009 2:29 am
Quote
In B, " to have been" means they are equipped from past till present ( today), right?.

should "been" be used here?

_________________
"Native of" is used for a individual while "Native to" is used for a large group

GMAT/MBA EXPERT

lunarpowerGMAT Instructor
Follow user
View User Profile
Joined

03 Mar 2008
Posted:

3377 messages
Followed by:

1356 members
Thanked:

2167 times
GMAT Score:

800
Tue Aug 04, 2009 2:45 am
Quote
viju9162 wrote:
In B, " to have been" means they are equipped from past till present ( today), right?.

should "been" be used here?


read my post, directly above yours. in this context, no, it doesn't mean that. (this is NOT a present perfect construction, because it's an infinitive
construction. note my example above, in which "the students cheated" is certainly not an event that continues into the present.)

_________________
Ron is a Director of Curriculum Development at Manhattan GMAT. He has been teaching various standardized tests for 20 years.

--

Pueden hacerle preguntas a Ron en castellano


Potete chiedere domande a Ron in italiano
On peut poser des questions Ron en franais
Voit esitt kysymyksi Ron:lle mys suomeksi

--

Quand on se sent bien dans un vtement, tout peut arriver. Un bon vtement, c'est un passeport pour le bonheur.

Yves Saint-Laurent

--

Learn more about ron

Free Manhattan Prep online events - The first class of every online Manhattan Prep course is free. Classes start every week.

uptowngirl92GMAT Destroyer!
Follow user
View User Profile
Joined

19 Apr 2009
Posted:

447 messages
Thanked:

5 times
GMAT Score:

670
Thu Oct 01, 2009 5:49 am
Quote
Did'nt understand it at all!!
Ron,could u please help us get the OA by the POE method??

hypermeganetJust gettin' started!


Follow user
View User Profile
Joined

30 Sep 2009
Posted:

24 messages
Thanked:

1 times
Thu Oct 01, 2009 1:13 pm
Quote
shilpi84 wrote:
Combining enormous physical strength with higher intelligence,the neanderthals appear as equipped for facing any obstacle the environment could put
in their path,but their relatively sudden disappearnce during the paleolithic era indicates that an inability to adapt to some environmental change led o
their extinction.

A)appear as equipped for facing any obstacle the environment could put in their path
B)appear to have been equipped to face any obstacle the environment could put in their path,
C)appear as equipped to face any obstacle the environment could put in their paths,
D)appeared as equipped to face any obstacle the environment could put in their path,
E)appeared to have been equipped for facing any obstacle the environment could put in their path,

OA:B
I'll try and help:

"Appear as equipped" would only be correct if the neanderthals' status was being compared to something else. For example: "the neanderthals
appear as equipped to fight the cold as they do the tigers."

Since we know their status is not being compared, we can eliminate A, C, D.


So we're stuck between B and E. Let's look at E:

E)appeared to have been equipped for facing any obstacle the environment could put in their path,

"appeared to have been equipped" doesn't seem wrong at first. It's wrong due to the structure of the infinitive clause but who cares? That's the hard
part to figure out. The EASY part of the sentence that is flawed is in the second half:

for facing any obstacle the environment could put in their path,

"for facing" is the present participle and the entire sentence is in the past tense. You can eliminate this one right away.

So, even without knowing why B is correct, you can select it.

But for edification purposes:

B is correct because it uses a singular "appear", uses the past participle, and uses infinitive after the past participle.
Thanked by: uptowngirl92

GMAT/MBA EXPERT

lunarpowerGMAT Instructor
Follow user
View User Profile
Joined

03 Mar 2008
Posted:

3377 messages
Followed by:

1356 members
Thanked:

2167 times
GMAT Score:

800
Mon Oct 05, 2009 2:53 am
Quote
uptowngirl92 wrote:
Did'nt understand it at all!!
Ron,could u please help us get the OA by the POE method??
you can actually eliminate your way down to the correct answer by using 2 idioms.

(1) "appear as ADJ" is unidiomatic; the correct version is "appear to be ADJ", or, if someone/something was ADJ in the past, "appear to have been
ADJ".
that gets you down to (b) and (e).

(2) "equipped for VERBing" is unidiomatic; the correct version is "equipped to VERB".
that gets you to the correct answer.

there are further, more subtle, considerations that are addressed in my previous post, but these idioms are really all that you need to use POE.

_________________
Ron is a Director of Curriculum Development at Manhattan GMAT. He has been teaching various standardized tests for 20 years.

--

Pueden hacerle preguntas a Ron en castellano


Potete chiedere domande a Ron in italiano
On peut poser des questions Ron en franais
Voit esitt kysymyksi Ron:lle mys suomeksi

--

Quand on se sent bien dans un vtement, tout peut arriver. Un bon vtement, c'est un passeport pour le bonheur.

Yves Saint-Laurent
--

Learn more about ron


Thanked by: jaggies, Maripili87, kamlesh14u, wanderlust

Free Manhattan Prep online events - The first class of every online Manhattan Prep course is free. Classes start every week.

hk_4uRising GMAT Star


Follow user
View User Profile
Joined

22 Oct 2007
Posted:

79 messages
Sun Dec 13, 2009 2:18 am
Quote
lunarpower wrote:
this isn't a present perfect construction, because it's actually an infinitive.
i.e., it's not "they have been equipped"; it's "they appear TO HAVE been equipped".

in any case, you should probably just memorize this construction as a one-off idiomatic structure. if you say "they seemed/appeared to...", then, no
matter how remote the event is (in time), you use this construction.

for instance:
it seems that the students cheated on the exam
(normal past tense in this construction)
but...
the students seem to have cheated on the exam
(not here)
Hi Ron

I have memorized this as you have suggested. However I am not able to understand what you mean to say by the example above

Further , one question still remains , which construction is better in context

appear to have been equipped


appeared to have been equipped

Basically I wanted to know which is the correct verb tense here

GMAT/MBA EXPERT

lunarpowerGMAT Instructor
Follow user
View User Profile
Joined

03 Mar 2008
Posted:

3377 messages
Followed by:

1356 members
Thanked:

2167 times
GMAT Score:

800
Mon Dec 14, 2009 2:59 am
Quote
hk_4u wrote:
Further , one question still remains , which construction is better in context
appear to have been equipped
appeared to have been equipped

Basically I wanted to know which is the correct verb tense here


by default, "appear" is in the present tense, since we're talking about the way these things appear to present-day observers.

if there were a context that would place "appear" in the past, then you could use the past tense. for instance: to nineteenth-century observers, they
appeared to have been equipped...
but, absent such a context, you should go with the present, for simplicity (the default assumption is that this "appears" to be the case to present
observers).

--

in this case, you've also got parallelism between "appear" and "indicates", two descriptions that both describe observations made in the present.
since "indicates" is not underlined, that determines the tense context. so your other observation, which is made at the same time, should be parallel to
it.

_________________
Ron is a Director of Curriculum Development at Manhattan GMAT. He has been teaching various standardized tests for 20 years.

--

Pueden hacerle preguntas a Ron en castellano


Potete chiedere domande a Ron in italiano
On peut poser des questions Ron en franais
Voit esitt kysymyksi Ron:lle mys suomeksi

--

Quand on se sent bien dans un vtement, tout peut arriver. Un bon vtement, c'est un passeport pour le bonheur.
. Although she had been known as an effective legislator first in the Texas Senate and later in the United States
House of Representatives, not until Barbara Jordans participation in the hearings on the impeachment of
President Richard Nixon in 1974 was she made a nationally recognized figure, as it was televised nationwide.
29
A. later in the United States House of Representatives, not until Barbara Jordans participation in the hearings on the
impeachment of President Richard Nixon in 1974 was she made a nationally recognized figure, as it was
B. later in the United States House of Representatives, Barbara Jordan did not become a nationally recognized figure
until 1974, when she participated in the hearings on the impeachment of President Richard Nixon, which were
C. later in the United States House of Representatives, it was not until 1974 that Barbara Jordan became a nationally
recognized figure, with her participation in the hearings on the impeachment of President Richard Nixon, which
was
D. then also later in the United States House of Representatives, not until 1974 did Barbara Jordan become a
nationally recognized figure, as she participated in the hearings on the impeachment of President Richard Nixon,
being
E. then also later in the United States House of Representatives, Barbara Jordan did not become a nationally
recognized figure until 1974, when she participated in the hearings on the impeachment of President Richard
Nixon, which was
MODIFIER although SHE so after the comma, we must have the NAME Barbara Jordan.
Hearings WERE televised So B is right.
"which" is fine in (b).
the gmat tends to write sentences in which "which" stands for the ELIGIBLE noun that's closest to the comma.
by "eligible", i mean that the noun has to AGREE IN TERMS OF SINGULAR/PLURAL with the FOLLOWING VERB.
here's an example:
the box of nails, which is on the counter, is to be used on this project.
in this case, "which" CANNOT refer to "nails", since the verb "is" is singular. therefore, the nearest eligible noun is
"box (of nails)". so, "which" unambiguously stands for that.
in our observation, the gmat has been VERY good about this.
whenever i've seen a "which" that refers to "X + preposition + Y" rather than just Y, it has ALWAYS been the case
that X was singular and Y was plural (or X was plural and Y was singular), and the verb had a form that matched
X and didn't match Y.
in this sentence, "which were..." is plural and so can't refer to nixon. therefore it must refer to the hearings

47. Even though Clovis points, spear points with longitudinal grooves chipped onto their faces, have been found all
over North America, they are named for the New Mexico site where they were first discovered in 1932.
A. Even though Clovis points, spear points with longitudinal grooves chipped onto their faces, have been found all
over North America, they are named for the New Mexico site where they were first discovered in 1932.
B. Although named for the New Mexico site where first discovered in 1932, Clovis points are spear points of
longitudinal grooves chipped onto their faces and have been found all over North America.
C. Named for the New Mexico site where they have been first discovered in 1932, Clovis points, spear points of
longitudinal grooves chipped onto the faces, have been found all over North America.
D. Spear points with longitudinal grooves that are chipped onto the faces, Clovis points, even though named for the
New Mexico site where first discovered in 1932, but were found all over North America.
E. While Clovis points are spear points whose faces have longitudinal grooves chipped into them, they have been
found all over North America, and named for the New Mexico site where they have been first discovered in 1932.
Even though, although, and while introduce clauses
that appear to be logically incompatible but in
fact are not. In this sentence, the apparent
incompatibility that must be clearly expressed is
that although the spear points are named for a
particular place in New Mexico, they are in fact
found throughout North America. Because their
discovery took place in 1932 and is not ongoing,
the correct verb tense is simple past, not present
perfect.
A Correct. Th e even though clause expresses
clearly that the seeming incompatibility is
between where the spear points have been
found (all over North America) and the
naming of the spear points for a single site
in New Mexico.
B Th e sentence structure indicates that the
expected incompatibility is between the
geographically based name of the points and
their physical properties, which makes no
sense; where discovered is missing a subject
the correct form is where they were fi rst
discovered.
C Have been fi rst discovered is the wrong tense,
since the discovery is a discrete event
completed in the past.
D Th e sequence of information in this sentence
is confusing; even though and but both
introduce information that is contrary to
expectation, so to use them both to describe
a single apparent contradiction is redundant
and nonsensical.
E While introduces a description of Clovis
points and suggests that this appears
incompatible with their appearance all over
North America, which makes no sense; have
been fi rst discovered is the wrong tense.
Th e correct answer is A.

46. Heavy commitment by an executive to a course of action, especially if it has worked well in the past, makes it
likely to miss signs of incipient trouble or misinterpret them when they do appear.
A. Heavy commitment by an executive to a course of action, especially if it has worked well in the past, makes it
likely to miss signs of incipient trouble or misinterpret them when they do appear.
B. An executive who is heavily committed to a course of action, especially one that worked well in the past, makes
missing signs of incipient trouble or misinterpreting ones likely when they do appear.
C. An executive who is heavily committed to a course of action is likely to miss or misinterpret signs of incipient
trouble when they do appear, especially if it has worked well in the past.
D. Executives being heavily committed to a course of action, especially if it has worked well in the past, makes them
likely to miss signs of incipient trouble or misinterpreting them when they do appear.
E. Being heavily committed to a course of action, especially one that has worked well in the past, is likely to make an
executive miss signs of incipient trouble or misinterpret them when they do appear.
Th is sentence explains that an executive who is
blindly committed to a proven course of action is
likely to overlook or misinterpret indicators that
the plan may no longer be working. Th e sentence
needs to make clear who may misinterpret these
indicators.
A Th e passive construction causes the sentence
to be wordy and confusing; the reference for
it is ambiguous, leaving the reader with
questions about who or what is likely to miss
27
these signs.
B Th e sentence structure indicates that the
executive, not his or her strategy, causes signs
to be overlooked; the modifi er when they do
appear is misplaced.
C Th e reference for the pronoun it is unclear
because many nouns have intervened
between the appearance of the logical
referent (course of action) and it.
D Misinterpreting should be an infi nitive verb
form to parallel miss; the phrasing
throughout the sentence is wordy and
awkward.
E Correct. Th e grammatical structure of this
sentence and the appropriate placement of
modifi ers expresses the meaning clearly and
concisely.
Th e correct answer is E.
when you should eliminate "being"?
the answer to that question is, ROUGHLY, that you should avoid "being" when expressing the IDENTITY or
CHARACTERISTICS of some individual or thing. this is because "being" is usually unnecessary in such cases; there
are simpler modifiers (such as appositives) that, while absolutely impossible to use in spoken language, are better
in written language.
example:
being a cigar aficionado, john has strong opinions on when to use single-guillotine cigar cutters rather than
double-guillotine cutters. --> bad.
a cigar aficionado, john has strong opinions on when to use single-guillotine cigar cutters rather than doubleguillotine
cutters. --> good. notice that we can simply omit the "being" here.
you don't want to omit "being" here, because it's not expressing identity: in the context of (e), it's a necessary
verbal. (nice litmus test: try omitting it and see whether the sentence is still viable, perhaps with minor
modifications. here, it isn't.)
so, to sum up:
if "being" expresses IDENTITY or CHARACTERISTICS, then kill it.
otherwise, evaluate it on the same merits as you would any other verb.
A. Heavy commitment by an executive to a course of action, especially if it has worked well in the past, makes it
likely to miss signs of incipient trouble or misinterpret them when they do appear.
much more importantly:
* makes it likely to miss...
this doesn't work.
technically, this would mean that "it" - an unspecified entity - is likely to miss the signs.
if you use the "it is ADJ..." construction, and the verb has a specific subject, you MUST include that subject in the
construction. it is likely that the executive will miss...
Quote:
B. An executive who is heavily committed to a course of action ... makes missing signs of incipient trouble or
misinterpreting ones likely when they do appear.
once that modifier is eliminated, notice that you have a sentence that says that the executive him/herself makes
missing the signs likely.
"misinterpreting ones" is also wrong. this should be "them", not "ones".
28
C. especially if it has worked well in the past must be written immediately after course of action
D. Notice POSSESSIVE executives we cannot use THEM

43. Fossils of a whale that beached on an African shore more than a million years ago and was subsequently
butchered by hominids have been recovered by paleontologists.
A. that beached on an African shore more than a million years ago and was subsequently butchered by hominids
have
B. that beached on an African shore more than a million years ago and then was subsequently butchered by
hominids has
C. that beached on an African shore more than a million years ago, which was subsequently butchered by hominids,
has
D. having been beached on an African shore more than a million years ago and subsequently butchered by hominids,
have
E. having beached on an African shore more than a million years ago and then subsequently were butchered by
hominids have
26
Fossils HAVE B, C out
D.
Having been beached is passive action something like having been imprisoned (somebody else imprisoned
him/her) this cannot be true for beaching
E. WERE butchered is wrong (the whale was butchered, not fossils). Also, then and subsequently can not be used
together.

25
A Correct. Although the original sentence is
complicated, the parallelism of its structure
and phrasing allows its meaning to be clear
and its expression eff ective.
B Verbs saw and did indicate action completed
in the past; the simple past tense is not
appropriate in either case; the adjective
frequent cannot modify the verb; awkward
and muddy.
C Made indicates past action, but the
present tense is logically required; as well as
gesturing violates the parallelism of the two
subordinate (that) clauses; choppy and
unclear.
D Having seen is not parallel to have been;
made and did do not show ongoing action;
frequent incorrectly modifies the verb; as well
as gesturing destroys the parallelism of the
two subordinate (that) clauses; awkward and
unclear.
E Replacing the verb make with the infinitive
to make results in an ungrammatical
construction that fails to complete the
sentence.
Th e correct answer is A.
42. In the Louisiana Purchase of 1803, the United States acquired 828,000 square miles for about four cents an
acre, which more than doubled the countrys size and that brought its western border within reach of the Pacific
Ocean.
A. In the Louisiana Purchase of 1803, the United States acquired 828,000 square miles for about four cents an acre,
which more than doubled the countrys size and that brought
B. For about four cents an acre the United States acquired, in the Louisiana Purchase of 1803, 828,000 square miles,
more than doubling the countrys size and it brought
C. With the Louisiana Purchase in 1803, the United States acquired 828,000 square miles for about four cents an
acre, more than doubling its size and bringing
D. The United States, in the Louisiana Purchase of 1803, for about four cents an acre, acquired 828,000 square
miles, more than doubling the countrys size, bringing
E. Acquiring 828,000 square miles in the Louisiana Purchase of 1803, the United States bought it for about four
cents an acre, more than doubling the countrys size and bringing
Cause and Effect: so ING forms are required for all the effects.
A out
B out
D. "in the louisiana purchase" is placed AFTER "the united states". this placement implies that the united states itself
is actually contained within the louisiana purchase.
* "for about four cents an acre" is placed in a place that is unclear.
E has a bigger issue of IT referring to an adjective (Louisiana)

41. Researchers have found that individuals who have been blind from birth, and who thus have never seen anyone
gesture, nevertheless make hand motions when speaking just as frequently and in virtually the same way as
sighted people do, and that they will gesture even when conversing with another blind person.
A. who thus have never seen anyone gesture, nevertheless make hand motions when speaking just as frequently
and in virtually the same way as sighted people do, and that they will gesture
B. who thus never saw anyone gesturing, nevertheless make hand motions when speaking just as frequent and in
virtually the same way as sighted people did, and that they will gesture
C. who thus have never seen anyone gesture, nevertheless made hand motions when speaking just as frequently
and in virtually the same way as sighted people do, as well as gesturing
D. thus never having seen anyone gesture, nevertheless made hand motions when speaking just as frequent and in
virtually the same way as sighted people did, as well as gesturing
E. thus never having seen anyone gesture, nevertheless to make hand motions when speaking just as frequently
and in virtually the same way as sighted people do, and to gesture
Th e researchers have found (1) that individuals
make hand motionsas sighted people do and (2)
that they will gesturewith another blind person. In
the original sentence, the two findings are
reported in two parallel subordinate clauses
introduced by that. Th e verb tenses are logical and
parallel: who have been blind and who have never
seen indicate a condition that began in the past
and continues in the present; make and do refer to
present actions. Th e verb make (hand motions) is
correctly modified by the adverb frequently to
show how the action of the verb is carried out.
Th e emphatic future will gesture is properly used
here with even to emphasize the extreme or the
unexpected.
25
A Correct. Although the original sentence is
complicated, the parallelism of its structure
and phrasing allows its meaning to be clear
and its expression eff ective.
B Verbs saw and did indicate action completed
in the past; the simple past tense is not
appropriate in either case; the adjective
frequent cannot modify the verb; awkward
and muddy.
C Made indicates past action, but the
present tense is logically required; as well as
gesturing violates the parallelism of the two
subordinate (that) clauses; choppy and
unclear.
D Having seen is not parallel to have been;
made and did do not show ongoing action;
frequent incorrectly modifies the verb; as well
as gesturing destroys the parallelism of the
two subordinate (that) clauses; awkward and
unclear.
E Replacing the verb make with the infinitive
to make results in an ungrammatical
construction that fails to complete the
sentence.
Th e correct answer is A.

37. The results of two recent unrelated studies support the idea that dolphins may share certain cognitive abilities
with humans and great apes; the studies indicate dolphins as capable of recognizing themselves in mirrorsan
ability that is often considered a sign of self-awarenessand to grasp spontaneously the mood or intention of
humans.
21
A. dolphins as capable of recognizing themselves in mirrorsan ability that is often considered a sign of selfawareness
and to grasp spontaneously
B. dolphins ability to recognize themselves in mirrorsan ability that is often considered as a sign of selfawareness
and of spontaneously grasping
C. dolphins to be capable of recognizing themselves in mirrorsan ability that is often considered a sign of selfawareness
and to grasp spontaneously
D. that dolphins have the ability of recognizing themselves in mirrorsan ability that is often considered as a sign of
self-awarenessand spontaneously grasping
E. that dolphins are capable of recognizing themselves in mirrorsan ability that is often considered a sign of selfawareness
and of spontaneously grasping
A. Indicates THAT is the preferred construction. Capable of recognizing and to grasp are not parallel.
B. Ability to recognize is not parallel to of spontaneously grasping
C. Indicates THAT is the preferred

34. Minivans carry as many as seven passengers and, compared with most sport utility vehicles, cost less, get better
gas mileage, allow passengers to get in and out more easily, and have a smoother ride.
A. Minivans carry as many as seven passengers and, compared with most sport utility vehicles, cost less,
B. Minivans, which carry as many as seven passengers, compared with most sport utility vehicles, they cost less,
C. Minivans carry as many as seven passengers, in comparison with most sport utility vehicles, and have a lower
cost, they
D. Minivans, carrying as many as seven passengers, compared with most sport utility vehicles, cost less,
E. Minivans, which carry as many as seven passengers, compared with most sport utility vehicles the cost is lower,
and they
In D, the modifier "...carrying..." seems to signify that the latter traits are only true of minivans when those minivans
are actually carrying the requisite number of passengers. if they aren't, then all of a sudden those things aren't true
anymore.
choice (a) correctly uses the conjunction "and", which implies that both things are true but don't necessarily have to
do with each other (i.e., the # of passengers is independent of the other claims).
D has another mistake, because in D:
Minivans, carrying as many as seven passengers, compared with most sport utility vehicles, cost less,
The compared with most sport utility vehicles modify the nearer passengers , but the original intention is to
modify the MinivansSo D is wrong
Minivans carry as many as seven passengers and, compared with most sport utility vehicles, cost less, get better gas
mileage, allow passengers to get in and out more easily, and have a smoother ride.
Does compared with most sport utility vehicles modify Minivans? Becauseand, ahead of compared
, so compared can only modify theMinivans rather than modify the passengers?
Yes, the "and" helps to clarify. While the general "rule" is that noun modifiers have to touch the nouns they modify,
this is more of an art than a science. There are a few exceptions, and this is one. The other main exception is when
you have multiple modifiers for the same noun: you cannot put them all next to the same noun, so you just line them
up instead. For example, we can say: "a way of painting that became popular in the 1950's." It's not the "painting"
that became popular, but rather the "way." The word "way" doesn't make much sense on its own, however, so we
have to say "of painting" first before we modify it.
A student asked:
I still have a question of the parallelism in (A).
Minivans carry as many as seven passengers and, compared with most sport utility vehicles, cost less, get better gas
mileage, allowpassengers to get in and out more easily, and have a smoother ride.
if we drop "compared with most sport utility vehicles" ...so reduce to
Minivans V1 as many as seven passengers and V2 ,V3 ,V4, and V5.
Is there anything wrong in this kind of parallelism? Is the first "and" necessary here?
Isn't it a run-on sentence? (between V3, V4)
I used to think that S + V1, V2, V3, V4 ,and V5 is correct.
plz help me clarify~
19
Answer:
well, remember that parallelism is really of 2 types: GRAMMATICAL and LOGICAL.
in your case, v1 though v5 all happen to be in the same tense - present tense, because we're talking about
unchanging features of minivans - but ONLY v2 through v5 are LOGICALLY parallel.
v1 describes the van's passenger seating capacity, which is NOT presented as a point of superiority over sport utility
vehicles, while v2 through v5 specifically describe advantages of minivans over sport utility vehicles.
therefore, the sentence is written with v2 through v5 in list form, but, because v1 isn't in the same category as those
others, it's (rightfully) excluded from that list.
the SC intended to compare only 4 features:
cost less, get better gas mileage, allow passengers to get in and out more easily, and have a smoother ride.
beween Minivans and sport utility vehicles,
excluding the feature of
as many as seven passengers.
IF I place the comparison at the front, 5 features will be compared.
Question:
Is possible to have 2 "and" in a list???
"Minivans carry ....AND....cost less, get better..., allow....AND have....
I discarded A for this reason, but it is wrong!
Answer:
these two "and"s are not both in the list.
the list begins with "cost less".
analogy:
i went to the grocery store and bought food, got money from the ATM, and sampled dishes from the deli.
this is fine, since the list (3 things that i did at the store) STARTS with "bought food". "went to the store" is not part
of the list.
these choices are run-on sentences: each contains a sentence that is written with two subjects.
eliminating modifiers from choice (b) yields "Minivans, they cost less..."
eliminating modifiers from choice (c) yields "Minivans, they get better gas mileage..."
eliminating modifiers from choice (e) yields "Minivans, the cost is lower..."

34. Minivans carry as many as seven passengers and, compared with most sport utility vehicles, cost less, get better
gas mileage, allow passengers to get in and out more easily, and have a smoother ride.
A. Minivans carry as many as seven passengers and, compared with most sport utility vehicles, cost less,
B. Minivans, which carry as many as seven passengers, compared with most sport utility vehicles, they cost less,
C. Minivans carry as many as seven passengers, in comparison with most sport utility vehicles, and have a lower
cost, they
D. Minivans, carrying as many as seven passengers, compared with most sport utility vehicles, cost less,
E. Minivans, which carry as many as seven passengers, compared with most sport utility vehicles the cost is lower,
and they
In D, the modifier "...carrying..." seems to signify that the latter traits are only true of minivans when those minivans
are actually carrying the requisite number of passengers. if they aren't, then all of a sudden those things aren't true
anymore.
choice (a) correctly uses the conjunction "and", which implies that both things are true but don't necessarily have to
do with each other (i.e., the # of passengers is independent of the other claims).
D has another mistake, because in D:
Minivans, carrying as many as seven passengers, compared with most sport utility vehicles, cost less,
The compared with most sport utility vehicles modify the nearer passengers , but the original intention is to
modify the MinivansSo D is wrong
Minivans carry as many as seven passengers and, compared with most sport utility vehicles, cost less, get better gas
mileage, allow passengers to get in and out more easily, and have a smoother ride.
Does compared with most sport utility vehicles modify Minivans? Becauseand, ahead of compared
, so compared can only modify theMinivans rather than modify the passengers?
Yes, the "and" helps to clarify. While the general "rule" is that noun modifiers have to touch the nouns they modify,
this is more of an art than a science. There are a few exceptions, and this is one. The other main exception is when
you have multiple modifiers for the same noun: you cannot put them all next to the same noun, so you just line them
up instead. For example, we can say: "a way of painting that became popular in the 1950's." It's not the "painting"
that became popular, but rather the "way." The word "way" doesn't make much sense on its own, however, so we
have to say "of painting" first before we modify it.
A student asked:
I still have a question of the parallelism in (A).
Minivans carry as many as seven passengers and, compared with most sport utility vehicles, cost less, get better gas
mileage, allowpassengers to get in and out more easily, and have a smoother ride.
if we drop "compared with most sport utility vehicles" ...so reduce to
Minivans V1 as many as seven passengers and V2 ,V3 ,V4, and V5.
Is there anything wrong in this kind of parallelism? Is the first "and" necessary here?
Isn't it a run-on sentence? (between V3, V4)
I used to think that S + V1, V2, V3, V4 ,and V5 is correct.
plz help me clarify~
19
Answer:
well, remember that parallelism is really of 2 types: GRAMMATICAL and LOGICAL.
in your case, v1 though v5 all happen to be in the same tense - present tense, because we're talking about
unchanging features of minivans - but ONLY v2 through v5 are LOGICALLY parallel.
v1 describes the van's passenger seating capacity, which is NOT presented as a point of superiority over sport utility
vehicles, while v2 through v5 specifically describe advantages of minivans over sport utility vehicles.
therefore, the sentence is written with v2 through v5 in list form, but, because v1 isn't in the same category as those
others, it's (rightfully) excluded from that list.
the SC intended to compare only 4 features:
cost less, get better gas mileage, allow passengers to get in and out more easily, and have a smoother ride.
beween Minivans and sport utility vehicles,
excluding the feature of
as many as seven passengers.
IF I place the comparison at the front, 5 features will be compared.
Question:
Is possible to have 2 "and" in a list???
"Minivans carry ....AND....cost less, get better..., allow....AND have....
I discarded A for this reason, but it is wrong!
Answer:
these two "and"s are not both in the list.
the list begins with "cost less".
analogy:
i went to the grocery store and bought food, got money from the ATM, and sampled dishes from the deli.
this is fine, since the list (3 things that i did at the store) STARTS with "bought food". "went to the store" is not part
of the list.
these choices are run-on sentences: each contains a sentence that is written with two subjects.
eliminating modifiers from choice (b) yields "Minivans, they cost less..."
eliminating modifiers from choice (c) yields "Minivans, they get better gas mileage..."

32. Possible long-term consequences of the greenhouse effect, the gradual warming of the Earths climate, may
include melting the polar ice caps and a rising sea level.
A. may include melting the polar ice caps and a rising sea level
B. may include the melting of polar ice caps and the rising sea level
C. may include polar ice caps that are melting and sea levels that are rising
D. include melting the polar ice caps and sea levels that are rising
E. include melting of the polar ice caps and a rise in sea level
possible is already mention in the sentence, you dont need 'may'..its redundant.
so A,B and C are out.
E maintain parallelism as well.
B is out because MAY is redundant.
B doesn't maintain parallelism.
the melting of polar ice caps and the rising sea level
Melting :- Gerund
Rising : Adjective.
E is parallel because of
melting of PIC (Gerund equivalent to noun) || the rise in sea level :- NOUN.

Re: Appositive Vs. Absolute phrase


Thu Dec 16, 2010 1:04 pm

This is from my notes (which i made reading somewhere on net) :

Appositives rename noun phrases and are usually placed beside what they rename .
The following example shows a noun phrase in apposition to another:
The lady, our president, spoke out against racism.

"Our president" renames the subject "the lady" and so is


in apposition to it.

Absolute phrases are made of nouns or pronouns followed by a participle and any modifiers of the noun or pronoun. Absolute phrases contain a subject
(unlike participial phrases), and no predicate. They serve to modify an entire sentence : A group of words that modifies an independent clause as a
whole.

Joan looked nervous, her fears creeping up on her.


noun/subject: her fears
participle: creeping
modifier: up on her
absolute phrase: her fears creeping up on her

Tom paled when he came home, his mother standing in the


doorway.
noun/subject: his mother
participle: standing
modifier: in the doorway
absolute phrase: his mother standing in the doorway

I am sure, you can now answer your question yourself.

Than
21. Because of the enormous research and development expenditures required to survive in the electronics industry,
an industry marked by rapid innovation and volatile demand, such firms tend to be very large.
12
A. to survive
B. of firms to survive
C. for surviving
D. for survival
E. for firms' survival
The subject of the main clause (such firms) presumes a prior reference to the firms in question. Furthermore,
the logical subject of to survive and the logical complement of required should be made explicit. All three
demands are met by B, the best choice. Choices A, C, and D, with no reference to the firms in question, meet
none of these demands. In choice E, the illogical and awkward use of a prepositional phrase (for firms' survival)
buries the needed initial reference to firms in a possessive modifier.

23. Despite protests from some waste-disposal companies, state health officials have ordered the levels of bacteria
in seawater at popular beaches to be measured and that the results be published.
A. the levels of bacteria in seawater at popular beaches to be measured and that the results be
B. that seawater at popular beaches should be measured for their levels of bacteria, with the results being
C. the measure of levels of bacteria in seawater at popular beaches and the results to be
D. seawater measured at popular beaches for levels of bacteria, with their results
E. that the levels of bacteria in seawater at popular beaches be measured and the results
In this sentence, English idiom requires one of two paradigms: x ordered y to be zed or x ordered that y be z
'ed. Choice E, the best answer, employs the second of these paradigms. Choice A mixes the two paradigms
(levels... to be measured and that the results be published), producing a sentence that lacks parallelism. C
and D use neither paradigm and are thus unidiomatic. Also, in D, the pronoun their has no logical and
grammatical antecedent. Choice B unidiomatically employs the verb should (not in either paradigm); also, the
pronoun their does not agree in number with seawater, its most logical antecedent.

24. Manifestations of Islamic political militancy in the first period of religious reformism were the rise of the
Wahhabis in Arabia, the Sanusi in Cyrenaica, the Fulani in Nigeria, the Mahdi in the Sudan, and the victory of the
Usuli mujtahids in Shiite Iran and Iraq.
A. Manifestations of Islamic political militancy in the first period of religious reformism were the rise of the Wahhabis
in Arabia, the Sanusi in Cyrenaica, the Fulani in Nigeria, the Mahdi in the Sudan, and
B. Manifestations of Islamic political militancy in the first period of religious reformism were shown in the rise of the
Wahhabis in Arabia, the Sanusi in Cyrenaica, the Fulani in Nigeria, the Mahdi in the Sudan, and also
C. In the first period of religious reformism, manifestations of Islamic political militancy were the rise of the
Wahhabis in Arabia, of the Sanusi in Cyrenaica, the Fulani in Nigeria, the Mahdi in the Sudan, and
D. In the first period of religious reformism, manifestations of Islamic political militancy were shown in the rise of the
Wahhabis in Arabia, the Sanusi in Cyrenaica, the Fulani in Nigeria, the Mahdi in the Sudan, and
E. In the first period of religious reformism, Islamic political militancy was manifested in the rise of the Wahhabis in
Arabia, the Sanusi in Cyrenaica, the Fulani in Nigeria, and the Mahdi in the Sudan, and in
E, the best choice, uses parallel phrases for the two major coordinate members (in the rise of... and in the
victory of ...) and also for the series listed in the first of these (s in t, u in v, w in x, and y in z). E's placement of
the In... reformism phrase at the beginning of the sentence is direct and efficient. Choices A, B, C, and D omit
and before the Mahdi, the last element in the first series; thus, they incorrectly merge the second major member
(the victory of) into the series listed under the first member (the rise of). Furthermore, in A and B the in...
reformism phrase has been awkwardly set between the subject and verb of the sentence

12. Not one of the potential investors is expected to make an offer to buy First Interstate Bank until a merger
agreement is signed that includes a provision for penalties if the deal were not to be concluded.
A. is expected to make an offer to buy First Interstate Bank until a merger agreement is signed that includes a
provision for penalties if the deal were
B. is expected to make an offer for buying First Interstate Bank until they sign a merger agreement including a
provision for penalties if the deal was
C. is expected to make an offer to buy First Interstate Bank until a merger agreement be signed by them with a
provision for penalties if the deal were
D. are expected to make an offer for buying First Interstate Bank until it signs a merger agreement with a provision
for penalties included if the deal was
E. are expected to be making an offer to buy First Interstate Bank until they sign a merger agreement including a
provision for penalties if the deal were
NOT ONE singular D/E out
If the deal WERE not was (hypothetical)
C be cannot be used as there is no THAT before
A is best
sanj wrote:but in this similar sentence "A higher interest rate is only one of the factors, albeit an important one, that keepsthe housing market from
spiraling out of control, like it did earlier in the decade." plural verb 'keep' is correct

Punjab is the only one of the 33 contiguous states that still has a sizeable wheat production. so here also 'have' should be correct

no, you're missing the crucial difference between these two examples - namely, the presence or absence of the word the. that makes all the difference.

remember, if you're going to consider a prepositional phrase disposable, then the sentence WITHOUT the prepositional phrase has to make sense. let's
try that with the first one:
a higher interest rate is only one ... that keeps the housing market from spiraling out of control.
that doesn't make grammatical sense without the before 'only one'. therefore, we MUST consider all those words at the end to be part of the
prepositional phrase starting with 'of the factors', so that 'factors' is the subject of the verb 'keep'.
now let's try it with the second one:
punjab is the only one ... that still has a sizable wheat production.
ok, that works - perfectly logical, even if we eliminate the prepositional phrase. therefore, 'only one' is the subject of 'has'.

so, as you can see, the presence/absence of 'the' makes all the difference in the world.

Re: Minnesota is the only one of the


Thu May 13, 2010 6:59 am

ranjeet1975 wrote:what is wrong with C

i think the easiest criterion on which to eliminate choice (c) is the fact that its parallelism is inferior to the parallelism exhibited in choice (e).

note that choice (e) contains two parallel "where" clauses. in general, you should always opt for a choice with superior parallelism over a
choice with inferior parallelism.[/i]

also, the use of the comma in choice (c) isn't appropriate.


[b]there should not be a comma in the construction "X and Y" unless "X" and "Y" are independent clauses -- i.e., each is a complete
sentence on its own, with "and" serving as a coordinating conjunction.

and the use of 'there is' is right in E?

(e) is an officially correct answer, so, yes.

everything in the officially correct answers is always correct, 100% of the time. even if it would be considered ugly by many, or even most, other sources.
Pueden hacerle preguntas a Ron en castellano
Potete fare domande a Ron in italiano
On peut poser des questions Ron en franais
Voit esitt kysymyksi Ron:lle mys suomeksi

Un bon vtement, c'est un passeport pour le bonheur.


Yves Saint-Laurent
sangeethmani
Students

Posts: 22
Joined: Tue Oct 27, 2009 8:03 am

Re: Minnesota is the only one of the


Thu Sep 16, 2010 7:31 pm

What is the diffrence between D and E.. Why is E right?

Is D incorrect because an article follows where?


RonPurewal
ManhattanGMAT Staff

Posts: 16527
Joined: Tue Aug 14, 2007 8:23 am

Re: Minnesota is the only one of the


Tue Oct 05, 2010 5:44 am

sangeethmani wrote:What is the diffrence between D and E.. Why is E right?

Is D incorrect because an article follows where?


(d) and (e) are both grammatically correct, but (d) is out because it contains a significant change in meaning.

if two or more sentences are grammatically correct, then you must pick the one whose meaning adheres most closely to the
original meaning.
although the original is grammatically incorrect, its intended meaning is clear:
* minnesota is the only state where there is a sizable wolf population;
AND
* minnesota is the only state where the wolf is still the worst enemy of the sheep.

(e) preserves this meaning, while (d) does not.


the semicolon in (d) separates its meaning into two unrelated sentences; i.e., the part of (d) that follows the semicolon suggests that the wolf is still the
archenemy of the sheep in general. this is almost exactly the opposite of the intended meaning, which is that minnesota is the only state where that's
still true!
Pueden hacerle preguntas a Ron en castellano
Potete fare domande a Ron in italiano
On peut poser des questions Ron en franais
Voit esitt kysymyksi Ron:lle mys suomeksi

Un bon vtement, c'est un passepo

13. Minnesota is the only one of the contiguous forty-eight states that still has a sizable wolf population, and where
this predator remains the archenemy of cattle and sheep.
A. that still has a sizable wolf population, and where
9
B. that still has a sizable wolf population, where
C. that still has a sizable population of wolves, and where
D. where the population of wolves is still sizable;
E. where there is still a sizable population of wolves and where
if it says the only one of..., then it's singular.
if it says only one of... (without the), then it's plural.
in the phrase "wolf population", wolf is actually an adjective, not a noun. you can't use a pronoun to stand for an
adjective.
you can't say "such a wolf" or "this wolf" either; those constructions imply that you actually have some particular wolf
in mind.
BASIC DOUBT:
in this similar sentence "A higher interest rate is only one of the factors, albeit an important one, that keeps the
housing market from spiraling out of control, like it did earlier in the decade." plural verb 'keep' is correct
Punjab is the only one of the 33 contiguous states that still has a sizeable wheat production. so here also 'have'
should be correct
no, you're missing the crucial difference between these two examples - namely, the presence or absence of the
word the. that makes all the difference.
remember, if you're going to consider a prepositional phrase disposable, thenthe sentence WITHOUT the prepositional
phrase has to make sense. let's try that with the first one:
a higher interest rate is only one ... that keeps the housing market from spiraling out of control.
that doesn't make grammatical sense without the before 'only one'. therefore, we MUST consider all those words at
the end to be part of the prepositional phrase starting with 'of the factors', so that 'factors' is the subject of the verb
'keep'.
now let's try it with the second one:
punjab is the only one ... that still has a sizable wheat production.
ok, that works - perfectly logical, even if we eliminate the prepositional phrase. therefore, 'only one' is the subject of
'has'.
so, as you can see, the presence/absence of 'the' makes all the difference in the world.
1) s-v agreement
a,b, and c violate s-v agreement(states... has) has should be replaced by 'have'
2) between d and e,
E holds up parallel structure
another error in D) is after a semicolon. Although the sentence after the semicolon is complete sentence. it suggests
absolutely independent meanings.
when you use a semicolon between two sentences, the first one is more general statement, and the second one is
more specific.
for example,
Mitzi keeps in good shape; Mitzi run 7 miles, swimming, or weight-lifting.
the first sentence is more generic, the second one tell you that how Mitzi keeps in shape by these workouts.
Let's revert to our D) after the semicolon, it is absolutely independent from the first sentence.
In choices A and C, the construction that still has ..., and where modifies Minnesota with clauses that are not
grammatically parallel. In choice B, the omission of and illogically makes the where ... clause modify wolf
population rather than Minnesota--that is, choice B says in effect that the wolf population is where the wolf
remains the archenemy of cattle and sheep. Choice D is grammatically constructed, but it lacks a conjunction that
10
establishes a logical relation between the clauses; since Minnesota as a grammatical subject is separated from the
clause following the semicolon, the statement there need not even pertain to Minnesota. In E, the best choice, the
18. New theories propose that catastrophic impacts of asteroids and comets may have caused reversals in the
Earth's magnetic field, the onset of ice ages, splitting apart continents 80 million years ago, and great volcanic
eruptions.
A. splitting apart continents
B. the splitting apart of continents
C. split apart continents
D. continents split apart
E. continents that were split apart
The word splitting must function as a noun to parallel the other items in the noun series of which it is part:
reversals, onset, and eruptions. In B, the best choice, the definite article the clearly signifies that splitting is to
be taken as a noun. In A, splitting introduces a verb phrase that breaks the parallelism of the noun series. In C,
the verb split is similarly disruptive. Choice D, grammatically vague, resembles C if split is a verb and E if split
is an adjective. In E, continents illogically replaces the splitting in the series: although the impacts in question
may have caused continents to split, they did not cause those continents that were split apart 80 million years
ago to materialize.

anita_083 wrote:

70. The growth of the railroads led to the abolition of local times, which was determined by when the sun reached the observer's meridian

and differing from city to city, and to the establishment of regional times.

(A)which was determined by when the sun reached the observer's meridian and differing

(B)which was determined by when the sun reached the observer's meridian and which differed

(C)which were determined by when the sun reached the observer's meridian and differing

(D)determined by when the sun reached the observer's meridian and differed

(E)determined by when the sun reached the observer's meridian and differing

Thanks!

I thought Paul would have solved this question for us.

However, I will explain this to the best of my ability.

When we have a noun + of + object construction, WHICH refers to the NOUN, not the object. Reading the original question, you can see that the
middle clause refers to "local times." As such, you cannot use which to subordinate the sentence.

Now we are left with D & E. As Erin on TestMagic said, there is a clear difference between differed and differing.

Differed = PASSIVE voice / action is being done on the subject


Differing = ACTIVE / subject performs the action

"determined by when..." is a participial phrase modifying the object of the preposition. Therefore, we see that the "subject" is TIMES. TIMES is
being acted upon by the sun. Therefore DIFFERED.

TIME is differing from city to city. It is "performing the action." Therefore, we use ACTIVE voice.

Therefore, the answer is E.

RonPurewal wrote:
Hei wrote:Hi Stacey, you say that C is not right because "were determined" is not parallel with "(were) differing". Is it because of the omitted "were"?

nope, it runs deeper than that.

you're describing a state that simply existed back then, so, if you use 'were determined', you've got to use the simple past tense again: you'd say 'differed'
in this case.

the construction 'were verbing' is limited to situations in which something else happened during the verb-ing.
example: while the man was chatting with his girlfriend, his cell phone broke the calm with its strident ringtone.

Dear Ron,
I understood your point in the verb-ing example. Appreciate if you could answer my questions -

The growth led to X, which were A and B, and Y.

1. Can I have X and Y as -

X - which were determined by tests


Y - painted in blue color

Thus the complete sentence would be -

The growth led to X, which were determined by tests and painted in blue color, and Y

2. In the above sentence both X and Y are working verbs. X and Y are not participles because they are preceded by "were". Right ?

Appreciate you valuable help and time here.

regards,
Ankit
RonPurewal
ManhattanGMAT Staff

Posts: 16527
Joined: Tue Aug 14, 2007 8:23 am

Re: Re:
Mon May 09, 2011 2:13 am

saxenankit wrote:The growth led to X, which were A and B, and Y.

1. Can I have X and Y as -

X - which were determined by tests


Y - painted in blue color

Thus the complete sentence would be -

The growth led to X, which were determined by tests and painted in blue color, and Y

2. In the above sentence both X and Y are working verbs. X and Y are not participles because they are preceded by "were". Right ?

Appreciate you valuable help and time here.

regards,
Ankit

i think you're having some problems with your A/B/X/Y here -- i'm a little confused. it seems that the X and Y in your discussion are actually the A and
B in your original sentence on top; correct me if i'm wrong.

what do you mean by "working verbs"? i'm not familiar with that terminology.

in any case, when you look at something like "were determined", you can look at this in two ways that are not really different:
1) it's a passive-voice verb
2) it's a form of "to be" (were) + a past participle (determined).
note that these are not different, because (2) is actually the definition of the passive voice: i.e., the passive voice is defined as a verb form of "to be" plus
a past participle.

but, no, "determined" and "painted" are not themselves verbs in this case.

i can understand the confusion -- for these verbs, the participle and the past tense look identical (both are "determined").
for cases like these, here's a test:
if a form works like other past participles -- such as "driven", "given", "done", etc. -- then it's a participle.
if a form works like other past tense verbs -- such as "drove", "gave", "did", etc. -- then it's a participle.

here, "X were driven and given" would work, and "X were drove and gave" would be incorrect. therefore, these are participles, not past tense verbs.
Pueden hacerle preguntas a Ron en castellano
Potete fare domande a Ron in italiano
On peut poser des questions Ron en franais
Voit esitt kysymyksi Ron:lle mys suomeksi

Un bon vtement, c'est un passeport pour le bonheur.


Yves Saint-Laurent

First discovered more than 30 years ago, (-modifier-) Lina's sunbird (-subject-), a four-and-a-half-inch animal found in
the Philippines and that resembles a hummingbird (-modifier-),has(-verb-) shimmering metallic colors on its head ;a brilliant
orange patch, bordered with red tufts, in the center of its breast; and a red eye. (-list of things-)

You are asked to complete the second modifier:

A. found in the Philippines and that resembles


Not parallel : "found" (modifier) "resembles" (verb)
D. that is found in the Philippines and it resembles
Not parallel, and moreover in a parallel list like this one, if the subject is the same, repeating the subject (and IT resembles) is considered
redundant.

B. found in the Philippines and that, resembling


E. that is found in the Philippines, resembling
Those are not parallel as well "found/is found" and "resembling". ( and the comma + ing modifier is not used correctly)

C is parallel, and reminds us that the GMAT considers those modifiers perfectly parallel:
found in the Philippines and resembling
Another Official Question about this concept: scientists-have-recently-discovered-what-could-be-the-9394.html
_________________

2. First discovered more than 30 years ago, Lina's sunbird, a four-and-a-half-inch animal found in the Philippines and
that resembles a hummingbird, has shimmering metallic colors on its head; a brilliant orange patch, bordered with
red tufts, in the center of its breast; and a red eye.
A. found in the Philippines and that resembles
B. found in the Philippines and that, resembling
C. found in the Philippines and resembling
D. that is found in the Philippines and it resembles
E. that is found in the Philippines, resembling
the core of the sentence is
a bird has colors.

both 'found in the philippines'** and 'resembling a hummingbird' are modifiers. they use somewhat different specific verb formsbut they both modify
the same thing, and so they're parallel.

--

**digression: note the spelling of 'philippines': one L, two P's.

17. Despite an expensive publicity campaign, ticket sales for the new play were poor enough that
it closed only after two weeks.
poor enough that it closed only after two weeks
poor enough that it was closed after only two weeks
so poor that it closed only after two weeks
so poor that it was closed after only two weeks
so poor that only after two weeks it closed

17.
The original sentence contains the words "poor enough that it closed..." However,
"[Adjective] enough that [Clause]" is an incorrect idiomatic form. The proper idiom
with a clause is "so [Adjective] that [Clause]." In this case, the sentence should read
"so poor that..."
In addition, the placement of "only" is incorrect. "Only" should be placed immediately
before the word it modifies. In this case, "only" modifies "two weeks," so it should be
placed immediately before "two weeks."
Note that the correct answer does change the verb from active to passive voice ("it
was closed"), which is generally slightly less preferable. However, every other
answer choice is demonstrably wrong.
Moreover, the very slight difference in meaning between "it closed" and "it was
closed" is minor enough to allow without tripping the alarm of "altered intent."
Also, note that every choice contains the word "it" (which refers to "the new play," the
topic if not the grammatical subject of the sentence). Thus, pronouns are not an
issue in this problem.
(A) This choice is incorrect as it repeats the original sentence.
(B) The idiom "X enough to Y" should be replaced with "so X that Y," and the word
"only" should be directly in front of the time phrase it modifies, "two weeks."
(C) The word "only" should be directly in front of the time phrase it modifies: "two
weeks."
(D) CORRECT. This proper idiom "so X that Y" is used, and the word "only" comes
directly in front of the time phrase it modifies, "two weeks." The passive voice
expression "it was closed" is perfectly acceptable.
(E) As in choice (C), the word "only" should be directly in front of the time phrase it
modifies: "two weeks."

13. According to a recent study, hand sanitizers require a 60 percent minimum alcohol
concentration for the killing of most harmful bacteria and viruses.
require a 60 percent minimum alcohol concentration for the killing of
require that there be a 60 percent minimum alcohol concentration to kill
require that a 60 percent minimum alcohol concentration be present to kill
require a 60 percent minimum alcohol concentration to kill
require that there be a 60 percent minimum alcohol concentration for the killing of

13.
The phrase "require a 60 percent minimum alcohol concentration for the killing of" is
wordy and unidiomatic. The proper idiom takes the more concise form "require X to
Y."
(A) This choice is incorrect as it repeats the original sentence.
(B) This choice incorrectly uses the unidiomatic and wordy form "require that
there be X to Y." The proper idiom is "require X to Y."
(C) This choice incorrectly uses the unidiomatic and wordy form "require that X
be Z to Y." The proper idiom is "require X to Y."
(D) CORRECT. The choice uses the proper idiom "require X to Y" where X is
the noun phrase "a 60 percent minimum alcohol concentration" and Y is the
verb infinitive "kill."
(E) This choice incorrectly uses the unidiomatic and wordy form "require that
there be X for Y." The proper idiom is "require X to Y.

10. The original sentence contains several errors. First, the phrase remembered
because of is unidiomatic; the correct idiomatic construction is
remembered for. Second, "like" is incorrect because the Guggenheim is a
specific example and the use of "such as" would be more appropriate to
introduce examples. Third, the sentence incorrectly suggests that "the
Guggenheim Museum" is a "design." More properly, the sentence should
discuss the design "for" the museum.
(A) This choice is incorrect as it repeats the original sentence.
(B) CORRECT. "Because of" is replaced by "for." "Like" is replaced by "such
as." And the use of "the one for" makes clear that the design is for the museum
and is not the museum itself.
(C) This choice does not correct "because of." Moreover, it incorrectly implies
that the museum itself is a design. However, the use of "such as" in place of
"like" is correct.
(D) This choice does not correct "because of." Moreover, the relative
pronoun that lacks a clear antecedent. It would be correct to say "The
design for the house, like that for the factory, is beautiful," for example.
But such a parallel structure does not exist in this sentence. However, the
use of "such as" in place of "like" is correct.
(E) This choice does not correct "like." Moreover, the possessive construction
"Guggenheim Museum's" is awkward and unidiomatic. However, the use of
"for" rather than "because of" is correct.

10. Though Frank Lloyd Wright is best remembered today because of bold designs like the
Guggenheim Museum in New York City, most of his buildings were intended to blend into their
surroundings.
because of bold designs like the Guggenheim Museum
for bold designs such as the one for the Guggenheim Museum
because of bold designs such as the Guggenheim Museum
because of bold designs such as that for the Guggenheim Museum
for bold designs like the Guggenheim Museum's
4. Public-access cable television was created in the 1970s as a means to derive public benefit
from the laying of private television cables on public land.
from the laying of private television cables on public land
from laying private television cables on the public land
by the laying of private television cables on the public's land
from private television cables being laid on public land
by laying private television cables on land that was public

4.
The original sentence contains the correct idiom "to derive X from Y," where X and Y
are both nouns. In this sentence, the use of the noun phrase "the laying of private
television" maintains consistency and parallelism with the noun benefit mentioned
earlier in the sentence.
(A) CORRECT. This choice is correct as it repeats the original sentence.
(B) The verb "to derive from" requires noun objects. Here, "laying" is not a noun.
Moreover, "the public land" is not idiomatic; it should be "public land."
(C) The correct idiom is "to derive X from Y," not "to derive X by Y." Moreover, "the
public's land" is unnecessary when "public land" would suffice.
(D) "To derive from" is correct in this choice. The wordy and unidiomatic construction
"private television cables being laid on public land" fails to provide a noun that would
ensure consistency with the noun benefit mentioned earlier in the sentence.
(E) The correct idiom is "to derive X from Y," not "to derive X by Y." Moreover, "land
that was public" is wordy; "public land" is more concise.

1. Having more than the usual numbers of fingers or toes on the hands or feet is termed
polydactyly.
Having more than the usual numbers of fingers or toes
Having had more than the usual number of fingers or toes
Having more than the usual number of fingers or toes
To have more than the usual number of fingers or toes
To have more than the usual numbers of fingers or toes

1.
The original sentence incorrectly uses the phrase "numbers of" instead of the correct
expression "number of."
(A) This choice is incorrect as it repeats the original sentence.
(B) This choice correctly replaces "numbers of" with "number of." However, the
present perfect tense verb "having had" is incorrectly used. The present
perfect tense is used to indicate an event that started in the past and remains
true in the present. Since this sentence simply defines the term "polydactyly,"
the present perfect tense is inappropriate. Instead, in order to maintain parallel
structure, the phrase "is termed polydactyly" must be preceded by a noun
phrase; the word "having" is a gerund, a verb that acts as a noun, and is
therefore appropriate to open that phrase.
(C) CORRECT. This choice correctly replaces "numbers of" with "number of."
(D) This choice correctly replaces "numbers of" with "number of." However, the
phrase beginning with the infinitive form "to have" is not parallel with the
phrase "is termed polydactyly." To maintain parallel structure the phrase "is
termed polydactyly" must be preceded by a noun phrase; the word "having" is
a gerund, a verb that acts as a noun, and is therefore appropriate to open that
phrase.
(E) This choice incorrectly uses the phrase "numbers of" instead of the correct
expression "number of." Moreover, the phrase beginning with the infinitive form
"to have" is not parallel with the phrase "is termed polydactyly." To maintain
parallel structure the phrase "is termed polydactyly" must be preceded by a
noun phrase; the word "having" is a gerund, a verb that acts as a noun, and is
therefore appropriate to open that phrase.

3. Like other performance indicators, the growth rate of a start-up business is generally related
to the amount of time and resources dedicated to the endeavor.
Like other performance indicators, the growth rate of a start-up business is generally related
to the amount of time and
Just like other performance indicators, the growth rate of a start-up business is generally
related to the amount of time and
Like other performance indicators are, the growth rate of a start-up business is generally
related to the amount of time and number of
As other performance indicators are, the growth rate of a start-up business is generally related
to the amount of time and
As other performance indicators are, the growth rate of a start-up business is generally related
to the amount of time and number of

3.
While like correctly compares the two nouns indicators and growth rate, amount
of incorrectly modifies the countable noun resources. Amount of is applied to both
components of the compound noun: amount of time and (amount of) resources.
Amount of can be used to modify uncountable nouns, such as time, but not
countable nouns, such as resources. Number of should be used to modify
countable nouns.
(A) This choice is incorrect as it repeats the original sentence.
(B) Amount of incorrectly modifies the countable noun resources. Amount of is
applied to both components of the compound noun: amount of time and (amount of)
resources. Amount of can be used to modify uncountable nouns, such as time,
but not countable nouns, such as resources. Number of should be used to modify
countable nouns. Further, just like is unnecessarily wordy. The more concise like is
preferred.
(C) While number of correctly modifies the countable noun resources, like
incorrectly compares two verb phrases: like otherindicators are, the growth
rateis. Like can be used to compare nouns, but not verb phrases. As should be
used to compare verb phrases.
(D) As is correctly used to compare two verb phrases: "as other...indicators are, the
growth rate...is." However, amount of incorrectly modifies the countable noun
resources. Amount of is applied to both components of the compound noun:
amount of time and (amount of) resources. Amount of can be used to modify
uncountable nouns, such as time, but not countable nouns, such as resources.
Number of should be used to modify countable nouns.
(E) CORRECT. As correctly compares two verb phrases: as otherindicators are,
the growth rateis. Further, number of correctly modifies the countable noun
resources.

4. A recent study of the United States tort system indicates that in 2004, accident, productliability,
and other tort costs totaled $260 billion, twice as many as 1990.
twice as many as 1990
twice as much as 1990
twice the amount spent in 1990
a number double those of 1990s
a number double that of 1990s

4.
The word many is used to modify countable items, for example many apples. For
uncountable things, the modifier much should be used, as in much money. Here,
the quantity word refers to the "tort costs." It would be correct to say that the
costs of x are as much as the costs of y. It could also be correct to say that
costs of x are as many as the costs of y, but only if referring to the number of
types of costs (e.g. fixed, variable, tax-deductible, etc.), rather than the amount
of the expenses. Here, since it is the amount that is compared, as much as should
replace as many as in the original sentence. Additionally, tort costs are illogically
compared to the year 1990, rather than to tort costs in 1990.
(A) This choice is incorrect as it repeats the original sentence.

(B) This choice correctly uses as much as to compare the amount of the
costs. However, the comparison is still incomplete: tort costs are illogically
compared to the year 1990, rather than to tort costs in 1990.
(C) CORRECT. The phrase twice the amount spent in 1990 modifies $260
billion, correctly conveying the idea that in 1990 the amount spent was $130
billion.
(D) The plural pronoun those cannot refer to the singular a number, which is
the logical antecedent.
Those could refer to tort costs, but if that is the case, then a
comparison is illogically drawn between $260 billion and tort costs. The
logical comparison would be between $260 billion and the amount of the
tort costs in 1990. The construction is wordy and awkward, with the
- 70 -
possessive of 1990s having no following noun to clearly possess.
(not fully followed)

(E) The construction is wordy and awkward, with the possessive of


1990s having no following noun to clearly possess.
7. The administration has increased the number of fines for mining safety violations as part of
their campaign to protect miners.
has increased the number of fines for mining safety violations as part of their
have increased the number of fines for mining safety violations as part of their
has increased the number of fines for mining safety violations as part of its
has increased the amount of fines for mining safety violations as part of its
have increased the amount of fines for mining safety violations as part of their

7. The original sentence contains one error; the plural their can not refer to the
singular administration.
(A) This choice is incorrect as it repeats the original sentence.
(B) This choice repeats the original error; the plural their can not refer to the
singular administration. Furthermore, the plural have increased does not agree
with administration.
(C) CORRECT. Choice C correctly employs the singular has increased and its;
both agree with the singular administration.
(D) This choice correctly uses the singular has increased and its; both agree with
the singular administration. However, the use of amount is incorrect. Amount
is used for uncountable quantities. Fines are countable, and so number
should be used.
(E) Choice E incorrectly uses amount. Amount is used for uncountable quantities.
Fines are countable, and so number should be used. Also, the plural have
increased and the plural their cannot refer to the singular administration.

2. By choosing glass apartments towering a hundred feet over brownstone units designed for
earlier generations, seemingly younger-than-ever moneyed professionals have embraced a
modern design ethic that accentuated their luxury-laden lives.
By choosing glass apartments towering a hundred feet over brownstone units designed for
earlier generations, seemingly younger-than-ever moneyed professionals have embraced a
modern design ethic that accentuated
By choosing glass apartments towering a hundred feet over brownstone units designed for
earlier generations, seeming younger-than-ever moneyed professionals have embraced a
modern design ethic that accentuates
In choosing glass apartments in hundred-foot towers instead of brownstone units designed for
earlier generations, seemingly younger-than-ever moneyed professionals have embraced a
modern design ethic that accentuates
In choosing glass apartments in hundred-foot towers instead of brownstone units designed for
earlier generations, seemingly younger-than-ever moneyed professionals have embraced a
modern design ethic that accentuated
In choosing glass apartments towering a hundred feet over brownstone units designed for
earlier generations, seeming younger-than-ever moneyed professionals have embraced a
modern design ethic, accentuating

2.
The original sentence contains two problems. First, towering a hundred feet
over brownstone units is unclear and implies the absurd meaning that the glass
apartments are located directly over brownstone units in different buildings. Second,
the verb "accentuated" should be in the present tense, since the earlier use of the
present perfect tense ("have embraced") implies that the embracing is still happening,
and therefore that the ethic accentuates the lives in the general present. Incidentally,
in this context, "by choosing" and "in choosing" have nearly identical meanings; as a
result, this split is immaterial.
(A) Incorrect, as it repeats the original sentence.
(B) This choice repeats the errors from the choice (A) and adds another. The
adjective seeming is incorrect, since adjectives modify nouns; it is not
seeming professionals, but seemingly younger... professionals. An adverb
must be used to describe an adjective.
(C) CORRECT. Using in hundred-foot towers instead of rather than towering
a hundred feet over makes the intended meaning clearer. Also, the verb
"accentuates" is in the proper tense (present).
(D) The verb "accentuated" should not be in the past tense, as noted above.
(E) In this choice, towering a hundred feet over brownstone units is unclear
and implies the absurd meaning that the glass apartments are located directly
over brownstone units in different buildings. The adjective seeming is
incorrect, as noted earlier in choice B. An adverb must be used to describe an
adjective. Finally, the participle "accentuating" should arguably be
replaced with the relative clause "that accentuates"; following a comma,

10. The bowerbirds of Australia derive their name from the fact that the males build elaborate
bowers of sticks and twigs to attract females, decorating them with flowers and other
vegetation in a display of courtship.
the fact that the males build elaborate bowers of sticks and twigs to attract females,
decorating them with flowers and other vegetation
the elaborate bowers of sticks and twigs that the males build and decorate with flowers and
other vegetation in order to attract females
the elaborate bowers of sticks and twigs, decorated with flowers and other vegetation that the
males use to attract females
the fact that the males build elaborate bowers of sticks and twigs, having decorated them with
flowers and other vegetation, to attract females
the elaborate bowers of sticks and twigs that are built by the males and decorated with
flowers and other vegetation to attract females

The original sentence contains the pronoun "them" but it is not grammatically clear
whether the pronoun's antecedent is "bowers of sticks and twigs" or
"females." Logically, we know that the antecedent is "bowers", so we need to find a
replacement that makes this clear. Moreover, the bowerbird does not derive its name
from the fact that it builds bowers, but from the bowers themselves.
(A) This choice is incorrect as it is the same as the original sentence.
(B) CORRECT. This choice rewrites the sentence to make it clear that the name
derives from the bowers and not from the fact of building them, and it also eliminates
the pronoun "them" and instead refers to "structures" to make the relationship clear.
(C) This choice does not make it clear that the males build the bowers and decorate
them. Instead, it seems to suggest that the bowers exist on their own and that the
male uses only the flowers and vegetation to attract females.
(D) This choice uses the phrase "having decorated them" improperly. It is not
necessary to use "having" in this context because the sentence describes an ongoing
event, not one that occurred in the past.
(E) This choice is in the passive voice, which is not preferable to active voice when a
grammatical active version (such as B) is also offered. Moreover, the placement of
the modifier "that are built by the males" incorrectly implies the sticks and
twigs are built by the males. Also the phrase "and decorated with flowers and other
vegetation to attract females" seems to further imply that the sticks and twigs are also
decorated with flowers...

17. Since the last election, the lobbying effort initiated by environmental organizations,
homeowners, and small business owners have increased awareness of pending environmental
legislation.
have increased awareness of pending environmental legislation
have increased awareness about pending legislation dealing with the environment
has increased awareness about pending environmental legislation
has increased awareness of pending environmental legislation
has increased awareness of environmental legislation that is still pending

17.
The original sentence contains an incorrect subject-verb relationship. While it may
seem the subject of this sentence is environmental organizations, homeowners, and
small business owners, the actual subject is the singular lobbying effort. Therefore,
the plural verb have is incorrect.
(A) This choice is incorrect as it repeats the original sentence.
(B) The subject-verb relationship is incorrect. While it may seem the subject of this
sentence is environmental organizations, homeowners, and small business owners,
the actual subject is the singular lobbying effort. Therefore, the plural verb have is
incorrect. Additionally, this choice contains the incorrect idiom awareness about
instead of awareness of. The end of this sentence is unnecessarily wordy: pending
legislation dealing with the environment. A more concise wording is preferable.
-9-
Awareness of is the right idiom not awareness about.
(C) While the subject-verb relationship issue is corrected by using the verb has to
agree in number with the singular subject lobbying effort, the incorrect idiom
awareness about is used instead of awareness of.
(D) CORRECT. The verb has agrees in number with the singular subject lobbying
effort and the correct form of the idiom, awareness of, is used. The end of the
sentence, pending environmental legislation, is clear and concise.
(E) The end of this choice, environmental legislation that is still pending, is
unnecessarily wordy.

16. Since 1989, after the Berlin Wall had been demolished, one of the most problematic ethnic
groups in the reunified Germany, in cultural and economic assimilation terms, were the former
East Germans, who have had to acclimate to an entirely different political system.
after the Berlin Wall had been demolished, one of the most problematic ethnic groups in the
reunified German, in cultural and economic assimilation terms, were the former East Germans
after the Berlin Wall was demolished, one of the most problematic ethnic groups in the
reunified Germany, in cultural terms as well as those of economic assimilation, were the
former East Germans
when the Berlin Wall was demolished, one of the reunified Germany's most problematic ethnic
groups, in terms of cultural and economic assimilation, was the former East Germans
when the Berlin Wall was demolished, one of the most problematic ethnic groups in the
reunified Germany, in terms of cultural and economic assimilation, has been the former East
Germans
after the Berlin Wall had been demolished, one of the most problematic ethnic groups in the
reunified Germany, in both terms of cultural and economic assimilation, have been the former
East Germans

16. The subject of the sentence is "one of the most problematic ethnic groups", a
singular noun. The verb, however, is "were," which is plural. We need to find a
choice that uses a singular verb instead.
(A) This choice is the same as the original sentence.
(B) This choice does not correct the subject-verb issue; it still uses "were" to refer to
"group."
(C) This choice uses the singular "was," but the simple past is not the appropriate
tense here because of the ongoing nature of the problem. Moreover, "the reunified
Germany's most problematic ethnic groups" is an awkward construction.
(D) CORRECT. This choice uses the singular "has been," which is also in the present
perfect tense, indicating the ongoing nature of the problem. Moreover, "in terms of
cultural and economic assimilation" is a more idiomatic and elegant phrasing than that
of the original.
(E) This choice does not correct the subject-verb issue; it uses "have been" to refer to
"group."

12. Before the Civil War, Harriet Tubman, along with other former slaves and white abolitionists,
helped create what had become known as the Underground Railroad, and were responsible for
leading hundreds, if not thousands, of slaves to freedom.
had become known as the Underground Railroad, and were
would become known as the Underground Railroad, and were
had become known as the Underground Railroad, and was
has been becoming known as the Underground Railroad, and was
would become known as the Underground Railroad, and was

The original sentence contains two flaws. First, "what had become known as the
Underground Railroad" is incorrectly in the past perfect tense ("had become"). The
past perfect is used to describe a past event that occurred before another past event.
In this case, however, the "Underground Railroad" did not become known as such (or
known as anything at all) until after it was created, and there is no past action that
occurs afterward that would justify the use of the past perfect tense. Second, the
subject of the sentence is "Harriet Tubman" only; phrases such as "along with,"
"accompanied by," and "as well as" do not create plural subjects (only "and" allows for
the formation of a plural subject). Yet, the verb used in the original sentence is "were,"
which suggests a plural subject. Instead, the correct verb form "was" is necessary in
order to agree with the singular subject "Harriet Tubman."
(A) This choice is incorrect as it repeats the original sentence.
(B) This choice corrects the verb tense by replacing the past perfect with the
conditional "would become known" (used to express the future from the point of view
of the past: "I said yesterday that I would go to the store today.") However, it does not
correct the subject-verb agreement problem (retaining "were").
(C) This choice does not correct the verb tense issue, retaining the incorrect past
perfect tense. It does correct the subject-verb agreement by replacing "were" with
"was."
(D) This choice uses the incorrect and awkward verb phrase "has been becoming,"
which incorrectly suggests that the labeling of the Underground Railroad continues to
the present day. However, it does correct the subject-verb agreement by replacing
"were" with "was."

2. A higher interest rate is only one of the factors, albeit an important one, that keeps the
housing market from spiraling out of control, like it did earlier in the decade.
keeps the housing market from spiraling out of control, like it did
keep the housing market from spiraling out of control, as it did
keeps the housing market from spiraling out of control, as it did
keep the housing market from spiraling out of control, like
keep the housing market from spiraling out of control, like it did

The original sentence states that "a higher interest rate is only one of the
factors...that keeps..." The clause "that keeps the housing market from
spiraling out of control" is describing the word "factors." Since "factors" is plural
noun and "keeps" agrees with singular nouns, we need to find a sentence that
replaces "keeps" with "keep". Also, "like it did earlier in the decade" is
incorrect. When comparing clauses (i.e., a phrase containing a subject and
a verb), we must use "as" instead of "like." For example, "She sings like her
mother" and "She sings as her mother does" are both correct, but "She sings
like her mother does" is not.
(A) This choice is incorrect as it repeats the original sentence.
(B) CORRECT. This choice remedies the verb agreement issue by using the
singular "keep." It also correctly changes the comparison word from "like" to
"as."
(C) This subject-verb issue with "keeps" remains.
(D) The subject-verb issue is remedied, however, the comparison is a faulty
one. "Like" is used to compare two nouns and here the second part of the
comparison is "earlier in the decade." The phrase "it did" is needed here to
logically complete the comparison, in which case, "as" must be used to draw
the comparison.
(E) The subject-verb issue is remedied, however, the word "as" should be
used instead of "like

15. According to anthropologists, the use of human language covers a wide spectrum from
practical communication between people engaged in the same task to establishing hierarchy
within a social group.
practical communication between people engaged in the same task to establishing hierarchy
communicating practically between people engaging in the same task to the establishment of
hierarchy
practical communication between people engaged in the same task to the establishing of
hierarchy
communicating practically between people engaging in the same task to hierarchy established
practical communication

lowering housing prices instead of raise them."


15.
The original sentence contains the construction "from X to Y," which requires
parallelism between X and Y. In this case, X is the regular noun phrase "practical
communication," but Y is the gerund "establishing". (A gerund is an "-ing" form of a
verb acting as a noun, such as in the sentence "Swimming is fun.") We need to find a
choice that puts both X and Y in the same grammatical form. Note that the difference
- 31 -
between "people engaged in the same task" and "people engaging in the same task"
is minimal. Both forms are valid.
(A) This choice is incorrect as it repeats the original sentence.
(B) This answer choice changes X to a gerund and Y to a regular noun phrase. X and
Y are still not parallel.
(C) Adding the word "the" in front of Y here doesn't change the fact that the regular
noun phrase is not parallel to the gerund.
(D) This answer choice changes X to a gerund but Y to "hierarchy established," which
is not parallel to X.
(E) CORRECT. This choice correctly changes Y to a regular noun phrase "the
establishment of hierarchy," so that this phrase is now parallel to X, "practical
communication."

11. Undergoing a gastric bypass is to commit to a new lifestyle not only because they will no
longer be permitted to eat large portions of food at one sitting, nor will they be allowed to
consume foods high in sugar or fat.
Undergoing a gastric bypass is to commit to a new lifestyle not only because they will no
longer be permitted to eat large portions of food at one sitting, nor will they
Undergoing a gastric bypass is committing to a new lifestyle not only because they will no
longer be permitted to eat large portions of food at one sitting, but also they will not
Undergoing a gastric bypass is to commit to a new lifestyle because the patient not only will
no longer be permitted to eat large portions of food at one sitting but also they will not
To undergo a gastric bypass is to commit to a new lifestyle not only because the patient will
no longer be permitted to eat large portions of food at one sitting, nor will they
To undergo a gastric bypass is to commit to a new lifestyle because the patient not only will
no longer be permitted to eat large portions of food at one sitting but also will not

11.
The comparison in this sentence between undergoing a gastric bypass and to
commit to a new lifestyle is not parallel. To be parallel, the comparison should either
be undergoing is committing or to undergo is to commit. Furthermore, neither
instance of the pronoun they has a clear antecedent, since there is no noun to which
either they could refer. Finally, the not only X nor Y construction is unidiomatic; the
proper construction is not only X but also Y.
(A) This choice is incorrect as it repeats the original sentence.
(B) Neither instance of the pronoun they in this choice has a clear antecedent.
(C) In this choice, undergoing is not parallel with to commit. Additionally, the plural
pronoun they is incorrectly used to refer to the singular noun the patient.
Moreover, the "not only X but also Y" construction requires X and Y to be parallel, but
"will no longer be permitted" is not parallel to "they will not be allowed."
(D) This choice incorrectly uses the unidiomatic not only X nor Y. Additionally, the
plural pronoun they incorrectly refers to the singular noun the patient.
(E) CORRECT. This choice properly compares to undergo to to commit, correcting
the initial error in parallelism. Additionally, the idiomatic not only X but also Y is used
correctly.

2.
This sentence contains three parallel elements: the unusual confluence, an
unpredictable backdrop, and the camaraderie. Additionally, the introductory
modifying phrase originally developed by ancient Hawaiians correctly modifies the
noun surfing.
(A) CORRECT. This choice properly follows rules of parallel construction and uses
the introductory phrase to correctly modify the noun surfing.
(B) The introductory modifying phrase originally developed by ancient Hawaiians
incorrectly describes surfings appeal rather than surfing itself. Additionally, the
pronoun its has no clear antecedent to refer to.
(C) The introductory modifying phrase originally developed by ancient Hawaiians
incorrectly describes surfings appeal rather than surfing itself. Furthermore, the
third element of the sentence, developing camaraderie among people, is not parallel
to the other two elements.
(D) This choice lacks clarity of meaning by stating a backdrop that is unpredictable
and that is, by turns, gracefully and serenely violent and formidable. The original
intent of the sentence is to use the adjectives graceful, serene, violent, and
formidable to describe surfings unpredictable backdrop; this choice improperly
separates this into two distinct ideas. Additionally, this choice incorrectly changes the
adjectives graceful and serene to adverbs gracefully and serenely; thus, the
adverbs incorrectly modify the adjectives violent and formidable rather than the
noun backdrop.
(E) The plural pronoun their incorrectly refers to the plural noun people, making it
seem that people, rather than surfing, contain the three parallel elements found in the
sentence.

2. Originally developed by ancient Hawaiians, surfing appeals to people due to the sports
unusual confluence of adrenaline, skill, and high paced maneuvering, an unpredictable
backdrop that is, by turns, graceful and serene, violent and formidable, and the camaraderie
that often develops among people in their common quest to conquer nature.
surfing appeals to people due to the sports unusual confluence of adrenaline, skill, and high
paced maneuvering, an unpredictable backdrop that is, by turns, graceful and serene, violent
and formidable, and the camaraderie that often develops
surfings appeal is its unusual confluence of adrenaline, skill, and high paced maneuvering, an
unpredictable backdrop that is, by turns, graceful and serene, violent and formidable, and the
camaraderie that often develops
surfings appeal to people is due to the sports unusual confluence of adrenaline, skill, and high
paced maneuvering, an unpredictable backdrop that is, by turns, graceful and serene, violent
and formidable, and developing camaraderie
surfing appeals to people due to the sports unusual confluence of adrenaline, skill, and high
paced maneuvering, a backdrop that is unpredictable and that is, by turns, gracefully and
serenely violent and formidable, and the camaraderie that often develops
surfing appeals to people due to their unusual confluence of adrenaline, skill, and high paced
maneuvering, an unpredictable backdrop that is, by turns, graceful and serene, violent and
formidable, and the camaraderie that often develops

3. The Federal Reserve announcement said that growth had accelerated after slowing in the
second quarter and that the policy makers remain concerned about the prospects of inflation,
even though there are few signs of higher energy prices driving up the cost of other goods so
far.
that growth had accelerated after slowing in the second quarter and that the policy makers
remain concerned about the prospects of inflation, even though there are few
growth had accelerated after slowing in the second quarter and that the policy makers remain
concerned about the prospects of inflation, even though there are few
that growth had accelerated after slowing in the second quarter and the policy makers remain
concerned about the prospects of inflation, even though there are little
growth had accelerated after slowing in the second quarter and the policy makers remain
concerned about the prospects of inflation, even though there are little
that growth accelerated after slowing in the second quarter and that the policy makers remain
concerned about the prospects of inflation, even though there are few

3.
The original is correct. The equivalent elements, that growth had accelerated... and
that the policy makers remain..., are parallel clauses beginning with "that." The verb
tenses are also correct; the action that occurred in the most distant past (had picked
up pace) uses the past perfect; the later past event (statement also said) uses the
simple past. Also, few is correct, since it modifies the quantifiable noun "signs."
(A) CORRECT. This choice is correct as it repeats the original sentence.
- 25 -
(B) This choice is not parallel; "that" is required before growth had accelerated... to
make it parallel to that the policy makers remain..."
(C) This choice is not parallel; that growth had accelerated... is not parallel to the
policy makers remain... Moreover, little is incorrect; since signs are countable,
few is required.
(D) This choice is parallel, but the syntax is incorrect. "That" can be colloquially
omitted after the verb "said," but not in formal writing. Also, "little" should be
replaced by "few."
(E) This choice is parallel, but incorrectly uses the simple past, accelerated. This
action is the earlier past event, as it must have occurred before the Fed commented
on the trend. The earlier of multiple past events must use the past perfect.

22. Descending approximately 4,000 years ago from the African wildcat, it has been an
exceedingly short time for the domestic cat with respect to genetic evolution and it scarcely
seems sufficient to allow the marked physical changes that transformed the animal.
Descending approximately 4,000 years ago from the African wildcat, it has been an
exceedingly short time for the domestic cat with respect to genetic evolution and it scarcely
seems sufficient to allow the marked physical changes that transformed the animal.
The domestic cat descended from the African wildcat approximately 4,000 years ago, which is
an exceedingly short time for the domestic cat's genetic evolution and scarcely sufficient for
the marked physical changes that transformed the animal.
Descending from the African wildcat approximately 4,000 years ago, the domestic cat has had
an exceedingly short time for its genetic evolution and has been scarcely sufficient for the
marked physical changes in the animal.
Having descended from the African wildcat approximately 4,000 years ago, the domestic cat
has had an exceedingly short time for its genetic evolution that has scarcely been sufficient for
the marked physical changes that transformed the animal.
The domestic cat descended from the African wildcat approximately 4,000 years ago, an
exceedingly recent divergence with respect to genetic evolution and one which scarcely seems
sufficient to allow the marked physical changes in the animal.

22.
The original sentence begins with a misplaced modifier. It is the domestic cat that
descended from the wildcat. We need to find a choice that expresses this
correctly.
(A) This choice is the same as the original sentence.
(B) The original modifier issue has been corrected. However, the phrase
"which is an exceedingly short time" has no referent ("4,000 years ago" is not a
time span but a specific moment).
(C) The original modifier issue has been corrected. However, the phrase "has
been scarcely sufficient..." incorrectly refers to the domestic cat.
(D) The original modifier issue has been corrected. However, the phrase "that
has scarcely been sufficient..." incorrectly modifies "genetic evolution". Also
"the marked physical changes that transformed the animal" is redundant.
Compare to E: "the marked physical changes in the animal," a much tighter
way of conveying the same information.
(E) CORRECT. This choice correctly rearranges the opening modifier to place
the words "the domestic cat" immediately next to the modifier "descended from
the African wildcat."

21. The physicist Richard Feynman presented a comprehensive introduction to modern physics
designed for undergraduate students in a two-year course.
The physicist Richard Feynman presented a comprehensive introduction to modern physics
designed for undergraduate students in a two-year course.
For undergraduate students, the physicist Richard Feynman presented a two-year course,
being a comprehensive introduction to modern physics.
A comprehensive introduction was in a two-year course by the physicist Richard Feynman
presenting to undergraduate students an introduction to modern physics.
Presenting a comprehensive introduction, the physicist Richard Feynman introduced modern
physics in a two-year course designed for undergraduate students.
In a two-year course designed for undergraduate students, the physicist Richard Feynman
presented a comprehensive introduction to modern physics.

21.
The original sentence suggests that Feynmans introduction covered "physics
designed for undergraduate students." This is nonsensical; rather the course is
designed for undergraduate students and covers the general topic of physics.
(A) This choice is incorrect as it repeats the original sentence.
(B) Beginning the sentence with for undergraduate students is awkward and
unclear. The verb phrase being a comprehensive introduction following the
comma seems illogically to modify the physicist Richard Feynman. With the use
of the unnecessary being, this creates the awkward suggestion that the
physicist was a comprehensive introduction. (Remember that "being" is almost
always wrong on the GMAT.)
(C) The sentences meaning is unclear due to the use of many prepositional
phrases with no punctuation: In a two-year course followed by by the physicist
- 23 -
Richard Feynman and later, to undergraduate students and to modern
physics. Also, the subject of this passive sentence is a comprehensive
introduction. It would make more sense for Feynman to be the subject, since he
was actively doing something: presenting. Finally, the use of "presenting" with
the passive construction introduces a verb tense error; Feynman is not currently
"presenting" the course, rather, the course was presented by Feynman.
(D) The use of both introduction and introduced is redundant: it suggests that
Feynman introduced a comprehensive introduction.
(E) CORRECT. The placement of the prepositional phrase in a two-year course
designed for undergraduate students at the beginning of the sentence clarifies
the meaning: a physics course was designed for the students. The construction of
the rest of the sentence is straightforward: the subject (the person doing the
action) the physicist Richard Feynman, the verb (what he actually did)
presented, and the object (what he presented) a comprehensive introduction to
modern physics.

20. Classical guitar was neither prestigious nor was often played in concert halls until it was
revived by Andres Segovia in the mid-twentieth century, having been won over by the
instrument's sound despite its relative obscurity.
Classical guitar was neither prestigious nor was often played in concert halls until it was
revived by Andres Segovia in the mid-twentieth century, having been won over by the
instrument's sound despite its relative obscurity.
Classical guitar was neither prestigious nor played often in concert halls until it was revived by
Andres Segovia in the mid-twentieth century, having been won over by the instrument's sound
despite its relative obscurity.
Classical guitar was not prestigious and was not often played in concert halls until Andres
Segovia revived it in the mid-twentieth century, after he was won over by the sound despite
the instrument's relative obscurity.
Classical guitar did not have prestige nor was it performed often in concert halls until its
revival by Andres Segovia, who in the mid-twentieth century was won over by the instrument's
sound despite its relative obscurity.
Classical guitar was neither prestigious nor was often played in concert halls until Andres
Segovia revived it in the mid-twentieth century, when he was won over by the sound of the
relatively obscure instrument.

20.
In the original sentence, "was" does not need to be repeated after "nor." Moreover,
"having been won over..." incorrectly modifies "classical guitar" (the subject of the
preceding clause) instead of Segovia.
(A) This choice is the same as the original sentence.
(B) This choice does not correct the modifier issue.
(C) CORRECT. This choice corrects the "nor" issue as well as the modifier
issue. Now it is clear that it was Segovia who was won over by the
instrument's sound.
(D) This choice is incorrect because the phrase "classical guitar did not have
prestige nor was it performed..." is both unidiomatic ("not ... nor" is incorrect)
and unparallel ("did not have....nor was it performed").
(E) This choice is incorrect because it repeats "was" after "nor" and because it
implies that Segovia was won over by the sound of the instrument in the midtwentieth
century, while the original sentence makes clear that this happened
at some earlier point.

19. Pests had destroyed grape, celery, chili pepper crops, sugar beet and walnut in the region, but
in the 1880s, more effective pest-control methods saved the citrus industry.
Pests had destroyed grape, celery, chili pepper crops, sugar beet and walnut in the region, but
in the 1880s, more effective pest-control methods saved the citrus industry.
Pests had destroyed grape, celery, chili pepper, sugar beet and walnut crops in the region, but
in the 1880s, more effective pest-control methods saved the citrus industry.
Pests had destroyed grape, celery, chili pepper, sugar beet and walnut crops in the region, but
more effective pest-control methods that were introduced in the 1880s saved the citrus
industry.
In the 1880s, pests destroyed grape, celery, chili pepper, sugar beet and walnut crops in the
region and more effective pest-control methods saved the citrus industry.
In the 1880s, more effective pest-control methods saved the citrus industry from what was
destroying grape, celery, chili pepper, sugar beet and walnut crops in the region

19.
This sentence has poor parallelism, due to the placement of the word crops. It
seems to indicate that there were three types of crops (grape, celery and chili pepper)
that had been destroyed, but that sugar beet and walnut had been destroyed entirely,
rather than just the crops of those plants.
Pests had destroyed grape, celery, chili pepper crops, sugar beet and walnut
in the region, but in the 1880s, more effective pest-control methods saved the
citrus industry.
(A) This choice is incorrect as it repeats the original sentence. OK.
Pests had destroyed grape, celery, chili pepper, sugar beet and walnut crops
in the region, but in the 1880s, more effective pest-control methods saved the
citrus industry.
(B) CORRECT. The word crop is placed correctly after the list of crop types.
The modifier in the 1880s correctly modifies the last phrase in the sentence,
indicating only that the citrus industry was saved in the 1880s. By using the
past perfect had destroyed, this sentence indicates that the other crops had
been destroyed at some time prior to the 1880s. The later past event uses the
simple past tense, whereas the earlier past event uses the past perfect tense.
This time line of events matches the meaning in the original sentence.
Pests had destroyed grape, celery, chili pepper, sugar beet and walnut crops
in the region, but more effective pest-control methods that were introduced in
the 1880s saved the citrus industry.
The citrus industry was saved in the 1880s. As per the sentence above,
we know that the methods were introduced in the 1880s but have no idea
when the citrus industry was saved.
(C) The modifying phrase that were introduced in the 1880s refers to the
methods immediately preceding the phrase. This alters the meaning of the
sentence, since in the 1880s no longer modifies saved the citrus industry.
The simple past tense indicates saved happened sometime in the past, but
not necessarily in that particular decade.
The modifier uses the relative pronoun that, but that should only
introduce essential modifiers. Which is a better choice here, since the
modifying phrase is non-essential.
- 22 -
In the 1880s, pests destroyed grape, celery, chili pepper, sugar beet and
walnut crops in the region and more effective pest-control methods saved the
citrus industry.
(D) The placement of the modifier In the 1880s and the use of two simple
past verbs destroyed and saved, indicate that both occurred in that decade.
The original sentence indicates only that the citrus industry was saved in the
1880s, and by using the past perfect had destroyed indicates that the other
crops had been destroyed at some time prior to the 1880s. The original
sentence used the word but to indicate a contrast, yet this sentence alters the
meaning by using and instead.
In the 1880s, more effective pest-control methods saved the citrus industry
from what was destroying grape, celery, chili pepper, sugar beet and walnut
crops in the region.
(E) The use of the phrase what was destroying is an awkward way to refer to
pests. Also, the placement of the modifier In the 1880s and the use of the
past progressive was destroying indicates that the destruction was ongoing in
that decade. The original sentence indicates only that the citrus industry was
saved in the 1880s, and by using the past perfect had destroyed indicates
that the other crops had been destroyed at some time prior to the 1880s.

15. The author Herman Melville and the poet Walt Whitman are icons of American literature,
greatly beloved by generations past and present.
The author Herman Melville and the poet Walt Whitman are icons
Herman Melville the author and Walt Whitman the poet are icons
The author named Herman Melville and the poet named Walt Whitman are great icons
The author, Herman Melville, and the poet, Walt Whitman, are icons
Herman Melville, the author, and Walt Whitman, the poet, had been icons

15.
The original sentence is correct. The modifiers Herman Melville and Walt Whitman
are restrictive they are necessary to restrict the scope of the words author and
poet respectively and hence the use of comma pairs to set off the modifiers is not
appropriate here. In addition, the context of the sentence implies that the men
continue to be icons of American literature since they are beloved by generations
both past and present; hence the use of the present tense are is appropriate.
(A) CORRECT. The original sentence is correct as written.
(B) The modifiers the author and the poet for Herman Melville and Walt
Whitman respectively are non-restrictive they are not necessary to identify the
subjects and only serve to add information and hence should be set off with comma
pairs (e.g., Herman Melville, the author, and Walt Whitman, the poet, .
(C) The phrases The author named Herman Melville and the poet named Walt
Whitman are unnecessarily wordy. In addition, an icon of something has implied
greatness; hence, the phrase great icon is redundant.
(D) The restrictive modifiers Herman Melville and Walt Whitman are improperly set
off by comma pairs.
(E). The modifiers the author and the poet are non-restrictive and properly set off
with comma pairs. The tense of the verb had been is not appropriate since it is
implied by the context of this sentence that the men continue to be icons of American
literature.

Please read (and follow!) the forum rules. Your subject line should be the first 5-8 words of the problem.

"before" places the "independence" portion of the sentence relative to the following clause. It does not place the "who relinquished power" portion of
the sentence. We still have two events: (1) had been a colony, and (2) the people who relinquished power only after a long struggle. That's our past
perfect / simple past mix in this case.

This is not the same thing as saying, for example:


Before she went to the store, she stopped by her mom's house.
Now, the before is actually linking the only two events we have here.

Also, for that before/after rule. It's not saying that you cannot have past perfect - only that it's not required any more. Typically, if the meaning of the
sentence is "this one thing happened RIGHT before this other thing - very close in time" then I don't generally use past perfect. If they are still widely
separated, though, then I may use it.
Stacey Koprince
Instructor
Director of Online Community
ManhattanGMAT

11. Before its independence in 1947, Britain ruled India as a colony and they would relinquish
power only after a long struggle by the native people.
Before its independence in 1947, Britain ruled India as a colony and they would relinquish
power
Before independence in 1947, Britain had ruled India as a colony and relinquished power
Before its independence in 1947, India was ruled by Britain as a colony and they relinquished
power
Before independence in 1947, India had been ruled as a colony by Britain, which relinquished
power
Before independence in 1947, India had been a colony of the British, who relinquished power

11.
The original sentence begins with a modifier ("Before its independence") that clearly
describes India, though the subject of the main clause is Britain. Moreover, "ruled
India as a colony" is wordy and the verb "ruled" is in the simple past when it would be
better in the past perfect (two past actions, one of which was earlier). Finally, "they"
has no grammatical antecedent and "would" is not a proper tense here (the simple
past is required).
(A) This choice is incorrect as it repeats the original sentence.
(B) Britain should not be the recipient of the modifier "Before its independence."
(C) The pronoun "they" has no logical antecedent. Logically it probably refers to the
British, but the British do not appear in the sentence. Also the past perfect tense
would have been preferable here (had been ruled) since the ruling occurred before
the relinquishing of the power.
(D) The phrase "ruled as a colony by Britain" is awkward and unclear. The placement
of the modifer "by Britain" makes it unclear that the ruling is being done by Britain.
(E) CORRECT. This correctly places India as the recipient of the opening modifier.
The past perfect is utilized to indicate that different times in the past. Notice that the
word "ruled" has been removed from this answer choice, however, this did not result
in a change of meaning. To be a colony of the British is to be ruled by the British.
The exclusion of the pronoun its in the beginning of the sentence (see answer
choices A and C) is incidental.

n Sep 07, 2008 1:00 pm

Since there are so many questions in the thread above, I took the liberty of numbering them.
Question 1
When an -ing word follows a noun without a comma between, the -ing word modifies the immediately preceding noun. So, yes, I agree that "lurking"
describes the crocodiles, effectively telling us which crocodiles were encountered by the vacationers.

Question 2
As for the "with eyes and noses peeking out" prepositional phrase, it depends on the context. Prepositional phrases are flexible, as they can be noun or
adverbial modifiers (and sometimes an odd blend of the two, in my opinion).

As a noun modifier: The native crocodiles with eyes and noses peeking out are scary. (i.e. WHICH crocodiles?)
As an adverbial modifier: Vacationers encounter native crocodiles lurking in the shadows, with eyes and noses peeking out. (i.e. HOW were the
crocodiles lurking?)

Adverbial modifiers can modify verbs, or other adjectives. So, while "lurking" is not a verb here, it is a modifier, which must be modified by an adverb or
adverbial phrase.

Notice that in the adverbial modifier example, the phrase could also be considered to modify the noun subject. You can temporarily remove the
"lurking" modifier to see this:

Vacationers encounter native crocodiles, with eyes and noses peeking out. (i.e. WHICH crocodiles? The one with eyes and noses peeking out.)

This is why I say the prepositional phrase can act as a blend of noun and adverbial modifier--in this case, both interpretations make sense.

You suggested there may be ambiguity: Maybe the "with eyes and noses peeking out" describes which vacationers or how the vacationers encountered
the crocodiles. There are two reasons this is not a problem:
(1) Context: We are told the vacationers are on boat tours, not in the murky water. The interpretation "vacationers with eyes and noses peeking out
from the surface of the murky water" just doesnt make as much sense.
(2) Placement: The modifier follows "crocodiles lurking," whereas "vacationers encountered" is physically separated from the prepositional phrase by
"crocodiles lurking," which is more logical anyway.

Finally, I would point out that the modifier split was primarily about "whose" vs. "with." "Whose" is clearly wrong, as "shallows" precedes the comma
instead of "crocodiles," which is the noun we want to modify. Whatever reservations you had about what the "with" phrase modifies, the preceding
language was the same in (D) and (E). In the real test, remember that you wont always like the phrasing of the correct choice, but you wont go wrong if
you avoid any with definite errors.

Question 3
Basically, they all mean the same thing, but use different sentence structure. Any of them could be correct in isolation, but the "whose" relative clause is
only correct if it follows "crocodiles" in the overall sentence.

with eyes and noses peeking out


--Prepositional phrase (with). Prepositional phrases can modify nouns, verbs, or other modifiers. See Question 2 above.
--"-ing" noun modifier (peeking out) modifies eyes and noses.

with eyes and noses that are peeking out


--Prepositional phrase (with). Prepositional phrases can modify nouns, verbs, or other modifiers. See Question 2 above.
--Relative clause (that), which modifies the immediately preceding noun (eyes and noses).
--Verb in a tense (are peeking) within the relative clause. This verb must match in number the subject of the relative clause (eyes and noses)

whose eyes and noses are peeking out


--Relative clause (whose), which modifies the immediately preceding noun. Relative clauses can ONLY modify nouns.
--Both a subject (eyes and noses) and a verb in a tense (are peeking) are within the relative clause. Standard subject-verb rules would apply: Plural
subject goes with plural verb.

Question 4
I dont think they are entirely similar. In the first, "peeking" follows and modifies a noun. In the second, "disappointing" follows "made," which could be
a verb (i.e. she made X). If "made" isnt supposed to be a verb here, I guess it is supposed to modify "recordings" (i.e. Which recordings? The ones she
made.) If so, that modifier would be better as a relative clause ("recordings THAT she made...") as that would allow "made" to be a verb, and the overall
phrase to be a modifier.
Emily Sledge
Instructor
ManhattanGMAT

10. Many daring vacationers who participate in guided boat tours on the Tarcoles River encounter
native crocodiles lurking in the shallows, whose eyes and noses are peaking out from the
surface of the murky water.
encounter native crocodiles lurking in the shallows, whose eyes and noses are peaking out
encountered native crocodiles lurking in the shallows, whose eyes and noses peak out
had encountered native crocodiles lurking in the shallows, whose eyes and noses peak out
encounter native crocodiles lurking in the shallows, with eyes and noses peaking out
encounter native crocodiles lurking in the shallows, with eyes and noses that are peaking out

0.
In the original sentence, the modifier whose eyes and noses are peaking out
incorrectly refers to shallows. When used to introduce a noun modifier, whose
always refers to the immediately preceding noun. In this case, the author intends to
refer to the crocodiles, not the shallows.
(A) This choice is incorrect as it repeats the original sentence.
(B) The modifier whose eyes and noses peak out incorrectly refers to shallows.
When used to introduce a noun modifier, whose always refers to the immediately
preceding noun. In this case, the author intends to refer to the crocodiles, not the
shallows. Further, the past tense encountered is inconsistent with the present
tense participate. When there is no compelling reason to change tenses,
consistency is preferred. Also, the past tense encountered seems to imply that
these encounters have already happened. However, it is clear from the original
- 16 -
sentence that the encounters are ongoing occurrences for vacationers who
participate in guided boat tours.
(C) The modifier whose eyes and noses peak out incorrectly refers to shallows.
When used to introduce a noun modifier, whose always refers to the immediately
preceding noun. In this case, the author intends to refer to the crocodiles, not the
shallows. Further, the past perfect had encountered is used incorrectly. The past
perfect tense should only be used to specify the first of two past events. Here, there
are no past events.
(D) CORRECT. The adverbial modifier with eyes and noses peaking out correctly
modifies the verb "lurking." As this example shows, adverbial modifiers do not
need to be placed adjacent to the verbs they modify. Further, the present tense
encounter is consistent with the present tense participate.
(E) While the adverbial modifier with eyes and noses that are peaking out correctly
modifies the verb "lurking," this phrasing is unnecessarily wordy. The more concise
with eyes and noses peaking out is preferred.

14. Based on recent box office receipts, the public's appetite for documentary films, like nonfiction
books, seems to be on the rise.
like nonfiction books
as nonfiction books
as its interest in nonfiction books
like their interest in nonfiction books
like its interest in nonfiction books

4.
The original sentence contains a faulty comparison. Nonfiction books is either
illogically compared to the publics appetite, or improperly used to suggest that
"nonfiction books" are examples of documentary films. The proper comparison
should be between the public's "appetite" for x and its "appetite" for y.
- 39 -
(A) This choice is incorrect as it repeats the original sentence.
(B) In this choice, "nonfiction books" is illogically compared to the public's "appetite."
The proper comparison should be between the public's "appetite" for x and its
"appetite" for y. Moreover, the use of the comparison word "as" is incorrect. "As" is
used to compare verb phrases, not nouns; in this case, two nouns ("appetite" and
"interest") are compared so the comparison word "like" should be used instead.
(C) This choice logically compares the public's "appetite" for documentary films to its
"interest" in nonfiction books. However, the use of the comparison word "as" is
incorrect. "As" is used to compare verb phrases, not nouns; in this case, two nouns
("appetite" and "interest") are compared so the comparison word "like" should be
used instead.
(D) This choice logically compares the public's "appetite" for documentary films to its
"interest" in nonfiction books. However, this choice incorrectly uses the plural pronoun
"their" to refer to the singular noun "the public."
(E) CORRECT. This choice logically compares the public's "appetite" for documentary
films to its "interest" in nonfiction books.

13. Antigenic shift refers to the combination of two different strains of influenza; in contrast,
antigenic drift refers to the natural mutation of a single strain of influenza.
influenza; in contrast, antigenic drift refers to the natural mutation of a single strain of
influenza
influenza, different than the natural mutation of a single strain, known as antigenic drift
influenza, in contrast to the natural mutation of a single strain, known as antigenic drift
influenza, different than antigenic drift, which refers to the natural mutation of a single strain
of influenza
influenza; in contrast to antigenic drift, which refers to the natural mutation of a single strain
of influenza

13.
The original sentence correctly contrasts "antigenic shift" and "antigenic drift" in a
parallel format. In addition, the connection punctuation, a semi-colon, is used
correctly to connect two complete sentences.
(A) CORRECT. The sentence is correct as written.
(B) This choice uses the incorrect comparison phrase "different than"; the correct
phrase is "different from." In addition, the comparison "antigenic shift refers to..." is
not parallel to "the natural mutation of... known as antigenic drift." Finally, the simple
comma between "influenza" and "different" provides an inadequate transition between
the two parts of the sentence; the addition of a conjunction such as "and is" (e.g., "...
influenza, [and is] different ...") is necessary here.
(C) The comparison "antigenic shift refers to..." is not parallel to "the natural mutation
of... known as antigenic drift."
(D) This choice uses the incorrect comparison phrase "different than"; the correct
phrase is "different from." In addition, the simple comma between "influenza" and
"different" provides an inadequate transition between the two parts of the sentence;
the addition of a conjunction such as "and is" (e.g., "... influenza, [and is] different ...")
is necessary here.
(E) This choice creates a sentence fragment by incorrectly using a semi-colon when
the second half of the sentence ("in contrast to antigenic drift...") is not a complete
sentence.

16. Although William Pereira first gained national recognition for his movie set designs, including
those for the 1942 film Reap the Wild Wind, future generations will remember him as the
architect of the Transamerica Tower, the Malibu campus of Pepperdine University, and the city
of Irvine.
including those for the 1942 film Reap the Wild Wind, future generations will
like those for the 1942 film Reap the Wild Wind, future generations
like that for the 1942 film Reap the Wild Wind, future generations will
including that for the 1942 film Reap the Wild Wind, future generations will
including those for the 1942 film Reap the Wild Wind, future generations

16.
(A) CORRECT. The original sentence is correct as written.
Future generations . Will remember
(B) The word like is used incorrectly to introduce an example. Using like alters the
meaning of the sentence, implying that William Pereiras designs were simply similar
to the designs for Reap the Wind. It is preferable to use the word including. The
present tense remember is incorrectly used with the subject future generations.
The original sentence was correct to use the future tense will remember.
(C) The word like is used incorrectly to introduce an example. Using like alters the
meaning of the sentence, implying that William Pereiras designs were simply similar
to the designs for Reap the Wind. It is preferable to use the word including. The
antecedent of the pronoun that is the plural movie set designs, so the plural
pronoun those should have been used.
(D) The antecedent of the pronoun that is the plural movie set designs, so the
- 48 -
plural pronoun those should have been used.
(E) The present tense remember is incorrectly used with the subject future
generations. The original sentence was correct to use the future tense will
remember.

15. Law firms and other professional services groups, academic institutions, and research divisions
often have informal talent marketplaces where senior employees strive to identify the best
employees junior to them and the junior employees compete for the assignments that they
find most attractive.
where senior employees strive to identify the best employees junior to them and the junior
employees compete for the assignments that they find most attractive
in which senior employees strive and identify the best junior employees and the junior
employees compete for the assignments that they find most attractive
where senior employees strive to identify the best junior employees and the best junior
employees compete for the most attractive assignments
that enable senior employees to strive and identify the best junior employees and the best
junior employees compete for the most attractive assignments
in which senior employees strive to identify the best junior employees and the best junior
employees compete for the most attractive assignments

15. This question uses the incorrect pronoun "where" to refer to "informal talent
marketplaces." "Where" is used to refer to physical locations, but the "talent
marketplaces" are a metaphorical construct, not an actual geographical location. In
this case, "in which" is the appropriate construction.
The original answer choice also utilizes unnecessary turns of phrase in its use of
best employees junior to them, as well as assignments that they find most
attractive, both of which use excessive language and could be expressed more
succinctly.
(A) This choice is incorrect as it repeats the original sentence.
(B) This choice correctly changes "where" to "in which" but incorrectly uses the
conjunction and between the terms strive and identify, suggesting that the two
terms denote two distinct activities, as opposed to the intended single activity of
identifying the best junior employees. Also, assignments that they find most
attractive are unnecessarily wordy.
(C) This answer repeats the original pronoun error ("where").
(D) This choice incorrectly uses the conjunction and between the terms strive and
identify, suggesting that the two terms denote two distinct activities, as opposed to
the intended single activity of identifying the best junior employees.
(E) CORRECT This choice correctly changes "where" to "in which" and also
rephrases the sentence more concisely ("best junior employees"; "most attractive
assignments").

14. Though once powerful political forces, labor unions have lost much of their influence, which
has resulted in a political climate that some analysts claim to favor management.
which has resulted in a political climate that some analysts claim to favor
resulting in a political climate that some analysts claim favors
which has resulted in a political climate that some analysts claim that favors
resulting in a political climate that some analysts claim to be in favor of
which has resulted in a political climate that has been claimed by some analysts to favor

14.
Claim to/to be.
The original sentence contains several errors. First, the relative pronoun "which"
cannot be used to modify the action of the preceding clause, as it does here ("which
has resulted in..."). Instead, it must be used to modify the immediately preceding noun
only. In this case, that noun is "influence," which cannot be described as resulting in a
"political climate." It is the loss of that influence that has resulted in the "political
climate" described in the sentence, not the influence itself.
Second, "that some analysts claim to favor management" is incorrect. If we
remove "some analysts claim" from the sentence, we are left with "a political
climate that...to favor management." This is incorrect. We need "to favor" to
agree with "a political climate that."
- 47 -
(A) This choice is incorrect as it repeats the original sentence.
(B) CORRECT. This choice corrects the relative pronoun error by replacing "which
has resulted" with "resulting." Moreover, "favors" agrees with "a political climate
that..."
(C) This choice does not correct the relative pronoun error. Moreover, it does not
correct the verb error.
It replaces "to favor" with "that favors," creating the illogical sequence "a
political climate that...that favors."
(D) This choice corrects the relative pronoun error by replacing "which has resulted"
with "resulting." However, it does not correct the verb error.
It replaces "to favor" with "to be in favor of," creating the illogical sequence "a
political climate that...to be in favor of."
(E) This choice does not correct the relative pronoun error. Moreover, it does not
correct the verb error. It does not replace "to favor." Additionally, "has been claimed
by some analysts" is unnecessarily in passive voice and is wordy.

13. Last year, the State Assembly failed to pass a balanced budget because they could not agree
on certain key provisions.
they could not agree on
they could not agree about
it could not agree about
the Assemblymen could not agree about
the Assemblymen could not agree on

13.
The original sentence contains a pronoun error. The pronoun "they" requires a plural
antecedent though there is none in the sentence; "State Assembly" is singular and
"they" cannot be used to refer to the politicians collectively since they were not
mentioned in the sentence.
(A) This choice is incorrect as it repeats the original sentence.
(B) The pronoun "they" is incorrect here because there is no plural antecedent.
Moreover, the phrase "agree about" is unidiomatic; "agree on" should be used
instead.
(C) The singular pronoun "it" correctly refers to the singular noun "the State
Assembly." However, the phrase "agree about" is unidiomatic; "agree on" should be
used instead.
(D) The use of "the Assemblymen" instead of "they" corrects the original pronoun
problem by eliminating the pronoun "they" altogether. However, the phrase "agree
about" is unidiomatic; "agree on" should be used instead.
(E) CORRECT. The use of "the Assemblymen" instead of "they" corrects the original
pronoun problem by eliminating the pronoun "they" altogether

12. One of the important functions of the United Nations is to decide if they should recognize the
legitimacy of a new government that assumed power through violence.
to decide if they should recognize the legitimacy of a new government that
to decide whether to recognize the legitimacy of a new government that
deciding whether to recognize a new government to be legitimate that
to decide if it should recognize the legitimacy of a new government, which
deciding whether they should recognize the legitimacy of a new government that

12.
In the original sentence, the plural pronoun they has no clear antecedent. It is trying
to refer to the United Nations, which is a singular noun. Additionally, on the GMAT, if
is used for a conditional idea, while whether is used for an alternative or possibility.
In this sentence, the United Nations is choosing between two possibilities: either it will
recognize the legitimacy of a new government or it will not. Thus, whether is the
appropriate word.
- 46 -
(A) This choice is incorrect as it repeats the original sentence.
(B) CORRECT. This choice correctly resolves the pronoun issue, and is the only
answer choice that avoids serious problems.
(C) The use of deciding as opposed to the infinitive decide is appropriate, but this
sentence incorrectly separates the modifier that assumed power through violence
from the modified noun government, making this choice awkward and incorrect.
(D) In this choice, if is incorrectly used to distinguish between alternatives.
Furthermore the non-restrictive relative pronoun which is used to introduce a
restrictive clause. Because the clause that assumed power through violence is
essential to the meaning of the sentence, that must introduce the clause.
Conversely, which is used to introduce nonessential elements.
(E) In this choice, the pronoun they has no clear antecedent, since the United
Nations is a singular entity.

9. In their most recent press release, the new management stated that they plan to expand into
the global software market via a series of acquisitions in Asia and Latin America.
their most recent press release, the new management stated that they plan to expand
its most recent press release, the new management stated that they plan to expand
its most recent press release, the new management stated that it plans on expanding
its most recent press release, the new management stated an intention to expand
its most recent press release, the new management stated their intention to expand

9.
The original sentence uses the incorrect plural pronoun their to refer to the singular
noun management. Similarly, the second plural pronoun they is also inconsistent
with its singular antecedent management.
(A) This answer choice is incorrect as it repeats the original sentence.
(B) This answer choice corrects one of the original pronoun errors by changing the
plural pronoun "their" to the singular pronoun "its," but fails to correct the second
pronoun problem, retaining the plural pronoun they, which is inconsistent with the
singular noun management.
(C) This answer choice uses the unidiomatic construction to plan on rather than the
correct idiom to plan to.
(D) CORRECT. This answer choice correctly uses the singular pronoun "its" to refer
to the singular noun "management," and eliminates the use of the incorrect plural
pronoun "they."
(E) In this answer choice, the plural pronoun their" does not agree with the singular
noun management.

5. The lawyers for the patent holder pressed the federal judge to impose an injunction against
the hardware manufacturer, arguing that they should take immediate action in order to
prevent further economic damages against their client.
they should take immediate action in order to prevent further economic damages against their
client
the judge act immediately in order to prevent the client from suffering further damages of an
economic nature
they should act immediately to prevent further economic damages being suffered by the client
the judge act immediately in order to prevent them from suffering economic damages further
immediate action should be taken to prevent their client from suffering further economic
damages

5.
The original sentence is problematic in a number of ways. First, its use of the pronoun
they is incorrect in the first instance. The first they should refer to the federal judge,
who is singular. Hence, the use of the plural pronoun they is incorrect, particularly
when viewed in conjunction with the second pronoun their which correctly refers to
the lawyers. Also, the sentence is unnecessarily wordy, with in order to detracting
from the sentences conciseness. Damages against their client is unidiomatic.
(A) This choice is incorrect as it repeats the original sentence.
(B) This answer choice corrects the pronoun issue. However, this sentence is even
more unnecessarily wordy and awkward, replacing economic damages with
damages of an economic nature. Also, the client is an awkward use of the article
the given that the client refers to the patent holder, which appears in the sentence.
(C) The sentence incorrectly retains the first they pronoun error, though it omits the
use of the second their. Also, the client is an awkward use of the article the given
that the client refers to the patent holder, which appears in the sentence.
(D) This choices use of them incorrectly suggests that the lawyers are to be
protected from economic harm rather than their client, the patent holder.
(E) CORRECT. This sentence remedies the pronoun issue, and uses their correctly
in the second part of the sentence. It is also clear and concise.

19. In the wake of several serious cases of mercury poisoning caused by tainted fish, the state
government ordered that all lakes and streams should be tested for mercury levels.
all lakes and streams should be tested for mercury levels
all lakes and streams should have their mercury levels tested
the mercury levels be tested for all lakes and streams
the mercury levels of all lakes and streams should be tested
the mercury levels of all lakes and streams be tested

19.
The non-underlined portion of the sentence contains the phrase "ordered that," which
requires a clause containing the subjunctive mood. However, the original sentence
uses "should be tested" instead of the subjunctive be tested. In addition, it is illogical
to test a lake or stream "for mercury levels": this implies that one is testing to see
- 58 -
whether mercury levels exist. Instead, one would test the mercury levels of the lake
or stream, measuring the amount of mercury.
(A) This choice is incorrect as it repeats the original sentence.
(B) The active construction all lakes and streams should have their mercury levels
tested is illogical: lakes and streams cannot do such a thing, rather a person would
have to test the mercury levels of the lakes and streams.
(C) This choice correctly uses the subjunctive be tested. However, the placement of
the prepositional phrase for all lakes and streams after be tested is awkward and
can be misinterpreted to mean that the testing is for or on behalf of the lakes and
streams. Instead, one would test the mercury levels of all lakes and streams.
(D) The non-underlined portion of the sentence contains the phrase "ordered that,"
which requires a clause containing the subjunctive mood. However, this choice uses
"should be tested" instead of the subjunctive be tested.
This choice correctly uses the subjunctive be tested, and correctly
(E) CORRECT.
references the testing of mercury levels, replacing the incorrect tested for
mercury
levels.

18.
The original contains a verb mood error. If
adolescence was is improper subjunctive.
Was should not be used in an if clause indicating a hypothetical condition. "If
adolescence were is correct.
(A) This choice is incorrect as it repeats the original sentence.
(B) This choice has a verb mood error. Was should not be used in an if clause
indicating a hypothetical condition; were would be correct. Also, the plural they can
not refer to the singular adolescence.
(C) This choice correctly employs the subjunctive mood by stating if adolescence
were. However, the plural they can not refer to the singular adolescence.
(D) CORRECT. The if clause properly uses the subjunctive mood, if adolescence
were. The singular it refers to the singular adolescence.
(E) This choice correctly employs the subjunctive mood by stating if adolescence
were. However, being one is wordy and awkward; using it is preferable. Being is
virtually always wordy and incorrect

18. A recent and popular self-help book wryly notes that if adolescence was not so painful, it
would have a droll comedic aspect, at least in retrospect.
was not so painful, it
was not so painful, they
were not so painful, they
were not so painful, it
were not so painful, being one

15. Though he had had success broadcasting his controversial radio program on highly regulated
terrestrial airwaves, Howard Stern opted out of terrestrial broadcasting in favor of the less
regulated satellite radio medium.
Though he had had
Though he has had
Even though he had
Having had
Having achieved

The original sentence correctly uses the past perfect form had had to establish a
chronology of two past events; it is clear that Howard Stern had had success (past
perfect) prior to the moment in the past when he opted out (simple past) of terrestrial
broadcasting.
(A) CORRECT. This choice is correct as it repeats the original sentence.
(B) The present perfect form has had incorrectly implies that Howard Stern
continues to have success broadcasting on terrestrial airwaves, even after opting out
of terrestrial broadcasting. The present perfect form is used for events that began in
the past and continue into the present; the past perfect "had had" must be used to
indicate the earliest of multiple past events.
(C) The use of the simple past he had fails to establish a time-ordering of the two
past events; the past perfect form of the verb is needed to indicate that Howard Stern
had had success (past perfect) prior to the moment in the past when he opted out
(simple past) of terrestrial broadcasting. Also, even though is unnecessarily wordy.
The more concise though is preferred.
(D) "Having had" is an accepted alternative past perfect construction and thus is
grammatically correct. However, the use of having had implies that Howard Stern
opted out of terrestrial broadcasting as a result of having had success with
terrestrial broadcasts. The logical meaning of the sentence is that he opted out
despite his previous success, not because of it.
GOOD meaning change twist.
(E) "Having achieved" is an accepted alternative past perfect construction and thus is
grammatically correct. However, the use of having achieved implies that Howard
Stern opted out of terrestrial broadcasting as a result of having achieved success
with terrestrial broadcasts. The logical meaning of the sentence is that he opted out
despite his previous success, not because of it.
GOOD meaning change twist.

Replace the verb "had" with "earned" and you'll see it immediately. We need a past perfect verb in the first half of the sentence, since the second half
happens after the first but still in the past. Past perfect verbs usually include "had" along with a past participle. The confusing thing here is that the past
participle is also the word "had"; but we do need both of them because they serve two different purposes..
Tim Sanders
Manhattan GMAT Instructor
Before i had success as a Manhattan GMAT instructor, i had had success taking the GMAT..

i would prefer to replace the non-past perfect instances of "had" with "experienced" in the sentence above, but the sentence as written works and
hopefully helps you see when each construction would be used..

10. A recent study has cited overcomplexity, increasing worker mobility between companies, and
poor financial planning in estimating that less than half of eligible American workers had
contributed the maximum amount to their employer-offered retirement plans.
A recent study has cited overcomplexity, increasing worker mobility between companies, and
poor financial planning in estimating that less than half of eligible American workers had
contributed the maximum amount to their employer-offered retirement plans.
Overcomplexity, increasing worker mobility between companies, and poor financial planning
have been cited by a recent study that estimated that over half of eligible American workers
do not contribute the maximum amount to their employer-offered retirement plans.
Citing overcomplexity, increasing mobility of workers between companies, and poor financial
planning, less than half of eligible American workers had contributed the maximum amount to
their employer-offered retirement plans, a recent study estimates.
A recent study of American workers, citing overcomplexity, increasing mobility of workers
between companies, and poor financial planning, has estimated that less than half of eligible
American workers had contributed the maximum amount to their employer-offered retirement
plans.
Citing overcomplexity, increasing mobility of workers between companies, and poor financial
planning, a recent study has estimated that less than half of eligible American workers
contribute the maximum amount to their employer-offered retirement plans.
The original sentence incorrectly utilizes the past perfect tense in its use of had
contributed. The
past perfect tense demands that the simple past tense also be used
in the sentence to refer to another action that occurred in the past but after the action
referred to by the past perfect tense. Here, the sentence uses the present perfect
tense has cited and the present participle estimating, but does not use the simple
past tense to refer to another action in the later past.
(A) This choice is incorrect as it repeats the original sentence.
(B) This choice is awkward in its use of the structure Overcomplexity, increasing
mobility . . . , and poor financial planning . . .have been cited as this structure leads
to the use of the passive voice, which is less preferable than the active voice. It is
also problematic in its use of both the past tense estimated and the present perfect
tense have been cited to refer to the recent study; the verb tenses should be
consistent in their treatment of the study.
(C) This choice incorrectly places the modifying phrase Citing overcomplexity,
increasing mobility . . . and poor financial planning adjacent to less than half of
American workers, incorrectly suggesting that it is less than half of American
- 54 -
workers, and not a recent study, that cites these factors as causes for a lack of
contribution to retirement plans. This choice also repeats the original verb tense error
with "had contributed."
(D) This choice repeats the original verb tense error with "had contributed."
(E) CORRECT. This choice correctly uses both the present participle (Citing . . .)
and the present perfect (has estimated) to refer to the recent study, as well as the
present tense contribute in reference to the study findings. The phrase Citing
overcomplexity, increasing mobility . . . , and poor financial planning, is correctly
used here as a modifier for a recent study.
9. In 1860, the Philological Society launched its effort to create a dictionary more comprehensive
than the world had ever seen; although the project would take more than 60 years to
complete, the Oxford English Dictionary had been born.
would take more than 60 years to complete, the Oxford English Dictionary had been
took more than 60 years to complete, the Oxford English Dictionary was
would take more than 60 years to complete, the Oxford English Dictionary was being
would take more than 60 years to complete, the Oxford English Dictionary was
took more than 60 years to complete, the Oxford English Dictionary was about to be

9.
The past perfect ("had been born") is used when there are two past actions and we
want to indicate which one happened first. In the underlined portion of the sentence,
other verb, "would take," is not in the past tense, so we need to use the
however, the
simple past "was born." (Remember that we always use the most simple tenses
allowed; the perfect tenses, and other complicated tenses, are used only when
required by the sentence structure.) The second half of the sentence stands in
contrast to the first half, in which the simple past "launched" is correctly paired with
the past perfect "had seen."
(A) This choice is incorrect as it repeats the original sentence.
(B) This choice changes both the first and second verbs to simple past ("took" and
have two events that took place at
"was born," respectively). In this circumstance, we
different times in the past, which requires use of the past perfect to indicate which
event happened first. The dictionary's "birth" obviously happens before its
completion, so correct usage would be that the "Dictionary had been born."
In B, if it is had been born, it will become CORRECT.
(C) The present participle "being" is used with the progressive tense to indicate a
continuing or ongoing action. Logically, however, the Dictionary's start must have
been at a single point in time, rather than over the course of the book's development.
(D) CORRECT. This
choice correctly uses the simple past "was born." A more
complicated past tense is not required because the other verb "would take," is not in
the past tense.
(E) This choice incorrectly adopts the construction "was about to be born," which
conflicts with the non-underlined portion of the sentence. The first half of the
sentence indicates that the project was "launched" in 1860 in the past tense, making
any reference to the book being "about to be born" at some future point in
time incorrect.This is a complicated one.

Re: born vs. complete, "born" is being used as a metaphor (of course, since a dictionary can't literally be born ). In this case "born" means it was
started. Logically, the start needs to occur before the completion.

Answer B changes the first verb from "would take" to the simple past "took." When using past tense, we default to the simple tense unless a reason
exists to use one of the more complex tenses (in which case we are required to use the more complex tense). In this case, two simple past tenses in the
same sentence indicate that the two events took place essentially simultaneously, but that is not the case. First, the book was "born" and then, some 60
years later, it was completed. This kind of situation requires the past perfect tense (used to indicate which of two past events that took place at different
times occurred first and which occurred second).

For the "future" issue, there are actually two different circumstances here. The first half of the sentence (before the semi-colon) is set in the past tense.
The second half is written from the point of view of the timeframe of the first half. At the point that it was launched, it was also born - these mean the
same thing - but it was not yet completed. From the viewpoint of the time it was launched (in the past), the completion is sometime in the future.

Answer E puts the "birth" in the future and the completion in the past, which is illogical. Answer D correctly puts the "birth" in the past (simultaneous
with launch) and the completion in the future from the point of view of the "birth."

Make sense?

You can construct the sentence that way - but none of the choices give you that option. There is more than one way to construct a sentence and still have
it be grammatically correct.

While "would" is most commonly used for conditional, it is not reflecting conditional tense here. Would can also be used for what's called the "future-
in-past" tense, which is what we have here - the sentence is written from the point of view of the past and talks about an event that is in the future from
that point of view. At the same time, that "future" event is really in the past relative to present time - hence, we use the word "would" instead of "will."

This is an uncommon tense. Most people won't see this on the test, so I wouldn't worry about it too much.
Stacey Koprince
Instructor
Director of Online Community
ManhattanGMAT

7 .
8. The health commissioner said that the government had implemented strict measures to
eradicate the contaminated food and, despite the recent illnesses, it will try to prevent the
outbreak from recurring in the future.
it will try
that it tried
it had tried
it would have tried
that it would try

8.
In the original sentence, the verb "had implemented" is in the past perfect tense,
indicating that this event occurred at some point before the commissioner spoke. The
verb "will try", however, is in the simple future. When the future is indicated from the
Instead, the conditional is
point of view of the past, the simple future is not used.
required. For example, "The man said that he would buy a new car" is preferable to
"The man said that he will buy a new car." We need to find a conditional
verb. Moreover, the pronoun "it" begins a new clause and thus requires repetition of
"that" in order to make clear, using parallel structure, that this new clause is still
something that the commissioner said. For example, "The man said that he would
buy a new car and that he would drive it everywhere" is preferable to "The man said
that he would buy a new car and he would drive it everywhere."
(A) This choice is incorrect as it repeats the original sentence.
(B) This choice does not offer the conditional "would try", though it does offer another
"that". The past tense "tried" is definitely wrong here because the trying
will happen "in the future" according to the original sentence. Thus this choice
changes the meaning unacceptably.
(C) This choice uses the past perfect tense "had tried" where the conditional "would
try" is preferred. An extra "that" is needed to make the two clauses "the government
had..." and "it had tried" parallel.
(D) This is a tempting choice as it fixes the verb tense to the conditional "would."
However, the tense is technically "conditional perfect" (would have tried), which is not
the proper tense. Moreover, an extra "that" is needed to make the two clauses "the
government had..." and "it would try" parallel.
- 53 -
(E) CORRECT. This choice provides the plain conditional tense and another "that".

6. Until Antoine Lavoisier proved otherwise in the eighteenth century, many scientists had
believed that combustion released phlogiston, an imaginary substance whose properties
were not fully understood.
many scientists had believed that combustion released phlogiston, an imaginary substance
whose properties were not fully understood
many scientists believed that phlogiston was an imaginary substance released by combustion
and its properties were not fully understood
phlogiston was an imaginary substance whose properties were not fully understood and which
many scientists had believed was released by combustion
phlogiston, an imaginary substance whose properties were not fully understood, was believed
by scientists to be released by combustion......AVOID PASSIVE ITS A FAULTY CONSTRUCTION
many scientists had believed that phlogiston was released by combustion and was an
imaginary substance whose properties were not fully understood

6.

The past perfect "had believed" is correct because it is the earlier of


two past
actions (the later action is "proved," which is in the simple past tense).
The main
clause is correctly written in active voice. The clause "an imaginary
substance
whose..." correctly modifies "phlogiston."

(A) CORRECT. This choice is correct as it repeats the original sentence.
(B) The simple past "believed" is incorrect because it is the earlier of two past
actions and should be in the past perfect tense instead: "had believed."
Moreover, "was an imaginary substance released by combustion" is
unnecessarily in passive voice.
Finally, the construction "and its properties were not fully understood" is
incorrectly parallel with "phlogiston was released by combustion" rather
than subordinate to it, as in the original sentence; this also creates
ambiguity around the pronoun "its" which could refer to either combustion
or phlogiston.
Here parallelism is not needed.
(C) The placement of "phlogiston" immediately after the opening clause
incorrectly implies that phlogiston had been imaginary until Lavoisier proved
otherwise. Moreover, the sentence is unnecessarily in the passive voice.
(D) This sentence is unnecessarily in the passive voice. Moreover, "was
believed" is incorrectly in the simple past tense. It should instead be in the past
perfect tense ("had been believed"), because it is the earlier of two past
actions.
- 52 -
(E) This sentence correctly uses the past perfect "had believed" but its
construction incorrectly implies that scientists had believed that phlogiston was
imaginary.

Hi kramacha1979,

BOTH have good parallelism and are therefore correct.

Look at parallelism this way--there is a shared sentence opener that has two alternate, parallel endings. The two endings should each make sense when
read following the shared sentence beginning.

...many scientists had believed that phlogiston:


(1) was released by combustion and
(2) was a toxic substance whose properties were not fully understood

...many scientists had believed:


(1) that phlogiston was released by combustion
AND
(2) that it (phlogiston) was a toxic substance whose properties were not fully understood

The first example is slightly preferable, as it is shorter. However, the wordiness/repetition of "that phlogiston" in the second example is acceptable: it's
grammatically correct, and fosters clarity by emphasizing the intended parallelism.
Emily Sledge
Instructor
ManhattanGMAT

EMILY,

Doesnt "an imaginary substance....." seem to modify the entire first clause
I have a problem here understanding as to when to call a part of the sentnce appositive and when to take it as modifier
Pls explain ( to cut short isnt "an imaiginary a modifier modifyig the former part of the sentence )

PLS HELP

Until Antoine Lavoisier proved otherwise in the eighteenth century, many scientists had believed that combustion released phlogiston, an imaginary
substance whose properties were not fully understood.

this sort of thing depends on context. it's usually pretty easy to figure out the context, but you should be aware that these
appositive-
type modifiers can modify EITHER the preceding noun OR the entire preceding
clause/phrase, depending on exactly what that context is .
in the first choice, it's obvious that "an imaginary substance..." is meant to refer to phlogiston. so this is ok.
judging by your post, it seems that you probably understand this context - but that you just didn't know that appositive-type modifiers have this sort of
flexibility. so now you understand.

--

by the way, USUALLY (but not always), the following hold:


* if the appositive type noun is concrete (such as "a substance"), then it will usually refer to the preceding noun.
* if the appsitive type noun is abstract (such as, say, "findings" or "a shortcoming"), then it will usually refer to the entire preceding clause. see #59
and #79 in the purple verbal supplement if you want examples.
Pueden hacerle preguntas a Ron en castellano
Potete fare domande a Ron in italiano
On peut poser des questions Ron en franais
Voit esitt kysymyksi Ron:lle mys suomeksi

Un bon vtement, c'est un passeport pour le bonheur.


Yves Saint-Laurent

2. Because of differences in the humans body's internal pressure and the ocean's, it is essential that a
diver returning to the surface ascends slowly or they will suffer a painful condition known as the
"bends."
Because of differences in the human body's internal pressure and the ocean's, it is essential that a
diver returning to the surface ascends slowly or they
Because of the difference between the human body's internal pressure and the ocean's, it is essential
that a diver returning to the surface ascend slowly or they
Because of the difference in the human body's internal pressure and that of the ocean, it is essential
that a diver returning to the surface ascends slowly or he
Because of differences in the human body's internal pressure and that of the ocean, it is essential that
a diver returning to the surface ascend slowly or they
Because of the difference between the internal pressure of the human body and that of the ocean, it is
essential that a diver returning to the surface ascend slowly or he

"differences in" is not properly


2. The original sentence contains several errors. First,

used to compare two explicit things. For example, "The difference in color is
more
important than the difference in size" is correct, but "The difference in
the color of the
apple and the color of the pear..." is not correct. Instead, it should be
"The difference
between the color of the apple and the color of the pear..." Second, after the
In this case,
construction "it is essential that", one must use the subjunctive mood.

"ascends" (which is indicative, not subjunctive) should be "ascend". Third, the


plural
pronoun "they" is used to refer to "a diver", which is singular. (A) This choice is
incorrect as it repeats the original sentence. (B) This choice incorrectly uses the plural
pronoun "they" to refer to the singular noun "a diver." (C) This choice incorrectly uses
"differences in" where "difference between" is required. Additionally, it uses the
indicative mood "ascends" where the subjunctive mood ("ascend") is required. Finally,
the construction "the human body's internal pressure and that of the ocean" is not
parallel. (D) This choice incorrectly uses "differences in" where "difference between"
is required. Additionally, the plural pronoun "they" incorrectly refers to the singular
- 95 -
noun "a diver." Finally, the construction "the human body's internal pressure and that
of the ocean" is not parallel. (E) CORRECT. The phrase "difference between"
correctly replaces "differences in." Additionally, the verb "ascend" is in the subjunctive
mood. Finally, the singular pronoun "he" correctly refers to the singular "a diver."

9. Despite entering the courthouse with police escort, the lead attorney and his assistant, manhandled by
an aggressive crowd of reporters that bombarded him with questions, was injured seriously enough to
warrant immediate medical attention.
Despite entering the courthouse with police escort, the lead attorney and his assistant, manhandled by
an aggressive crowd of reporters that bombarded him with questions, was injured seriously enough to
warrant immediate medical attention.
Despite the fact that the lead attorney and his assistant entered the courthouse with police escort,
they were manhandled by an aggressive crowd of reporters that bombarded the attorney with
questions and injured him so seriously that he needed immediate medical attention.
Despite their entering the courthouse with police escort, the lead attorney and his assistant were
manhandled by an aggressive crowd of reporters that bombarded him with questions, injuring him so
seriously as to warrant immediate medical attention.
Despite the fact that they entered the courthouse with police escort, the lead attorney and his
assistant, having been manhandled by an aggressive crowd of reporters, was bombarded with
questions and injured seriously enough to warrant immediate medical attention.
Despite entering the courthouse with police escort, the lead attorney and his assistant were
manhandled by an aggressive crowd of reporters that bombarded him with questions and injured him
so seriously as to warrant immediate medical attention.

9. The original sentence contains several errors. First, the subject of the original
sentence is "the lead attorney and his assistant", yet the corresponding verb is "was
injured". The subject and the verb do not agree in number one is plural, the other
singular. Second, "despite" is not properly used with a verb phrase. Instead, it
requires a noun or noun phrase. For example, "Despite eating the apple..." is not
correct, but "Despite his eating the apple..." is correct. Third, "injured seriously
enough to warrant medical attention" is incorrect in this context. "X enough to Y" is
used when the emphasis is on Y. "So X as to Y" is used when the emphasis is on X.
For example, "I am tall enough to touch the ceiling" implies that the focus is on the
fact of being able to touch the ceiling. "So tall as to be able to touch the ceiling"
implies that the focus is on the fact of being tall. Finally, the use of the pronoun "him"
is ambiguous, since it could refer to either the attorney or his assistant. (A) This
choice is incorrect as it repeats the original sentence. (B) CORRECT. It eliminates the
subject-verb agreement issue and ensures that "despite" is followed by a noun ("the
fact"). Additionally, the choice uses the correct expression "so X as to Y" to
emphasize the seriousness of the injury. Finally, the sentence is reworked to avoid
pronoun ambiguity. (C) The pronoun "him" has an ambiguous antecedent, since it
could refer either to the attorney or his assistant. (D) The singular verb "was" does not
agree with the plural subject "the lead attorney and his assistant." Additionally the
phrase "injured seriously enough to warrant immediate medical attention" incorrectly
emphasizes the medical attention over the seriousness of the injury, (E) The pronoun
"him" has an ambiguous antecedent, since it could refer either to the attorney or his
assistant. Additionally, the word "despite" is incorrectly followed by the verb "entering"
instead of a noun or noun phrase.

10. Though margarine was introduced as a supposedly healthful alternative to butter, recent studies
suggest it is as harmful or worse than butter.
it is as harmful or worse than butter
it is just as harmful or even worse than butter
that it is as harmful as or worse than butter
it is as harmful as if not worse than butter
that it is as harmful if not worse than butter10. The original sentence incorrectly omits "that" following the verb "suggest."
Moreover,
it contains the incomplete construction "as . . . or worse than" (missing the second
"as"). (A) This choice is incorrect as it repeats the original sentence. (B) This choice
incorrectly omits "that" following the verb "suggest" and it contains the unnecessary

word "just." Moreover, it contains the incomplete construction "as


. . . or even worse
than" (missing the second "as"). (C) CORRECT. This choice correctly uses the word
Also, it contains the
"that" to introduce a clause that follows the verb "suggest."
complete construction "as . . . as or worse than." (D) This choice incorrectly omits
"that" following the verb "suggest." (E) This choice contains the incomplete
construction "as . . . if not worse than" (missing the second "as").

11. The Diary of Anne Frank tells the true story of a young girl and her family that were hidden during the
Nazi occupation of the Netherlands by a gentile Dutch couple, though they were eventually discovered.
that were hidden during the Nazi occupation of the Netherlands by a gentile Dutch couple, though
they were eventually discovered
that were hidden by a gentile Dutch couple during the Nazi occupation of the Netherlands, though
they were eventually discovered
whom a gentile Dutch couple hid during the Nazi occupation of the Netherlands but were eventually
discovered
who were hidden by a gentile Dutch couple during the Nazi occupation of the Netherlands but were
eventually discovered
who were hidden by a gentile Dutch couple during the Nazi occupation of the Netherlands even though
they were eventually discovered

11. The original sentence uses the relative pronoun "that" where "who" is preferred
because the antecedent is a group of people. Also, the prepositional phrase "by a
gentile Dutch couple" is placed in such a way as to suggest that the occupation was
carried out by the couple. Finally, the pronoun "they" is ambiguous it could refer to
the family or to the couple. (A) This choice is incorrect as it repeats the original
sentence. (B) This choice incorrectly uses the relative pronoun "that" to refer to a
group of people. Additionally, the pronoun "they" is ambiguous - it could refer to the
family or to the couple. (C)
The use of the object pronoun "whom" makes "a girl and
her family" the object of the clause "a gentile Dutch couple hid;" however "a girl and
her family" are the subject of the next clause "were eventually discovered ." This is a
mismatch. (D) CORRECT. It correctly uses the pronoun "who" to refer to a girl and
her family. Additionally, the phrase "by a gentile Dutch couple" is placed immediately
after "who were hidden" to clarify the meaning. Finally, the ambiguous pronoun issue
is avoided altogether. (E) The pronoun "they" is ambiguous - it could refer to the
family or to the couple

12. Carbon monoxide levels in the atmosphere grew by enough of an increased percentage
during the
twentieth century that it began to trap heat radiating from the Earth, and it caused the average
surface temperature to rise.
Carbon monoxide levels in the atmosphere grew by enough of an increased percentage
during the
twentieth century that it began to trap heat radiating from the Earth, and it caused the average
surface temperature to rise.
Carbon monoxide levels in the atmosphere increased by enough of a percentage during the
twentieth
century that they began to trap heat radiating from the Earth, causing the average surface
temperature to rise.
Levels of atmospheric carbon monoxide increased sufficiently during the
twentieth century to begin
trapping heat radiating from the Earth, causing the average surface temperature to
rise.
Atmospheric carbon monoxide levels increased by a sufficient percentage during the twentieth
century
to begin trapping heat radiating from the Earth, which caused the average surface temperature
to rise.
Levels of carbon monoxide in the atmosphere during the twentieth century increased enough
to begin
trapping heat radiating from the Earth, causing the average surface temperature to rise.
12. The original sentence contains several errors. First, "carbon monoxide levels in the
atmosphere" is wordy. Second, "grew by enough of an increased percentage" is
wordy and redundant. Third the singular pronoun "it" incorrectly refers to the plural
"levels". Fourth, the final clause of the sentence--"and it caused the average surface
temperature to rise"--is disjointed from the main clause. (A) This choice is incorrect as
it repeats the original sentence. (B) This choice uses the unnecessarily wordy phrase
"increased by enough of a percentage."
(C) CORRECT. The sentence is made more
concise by rewriting "carbon monoxide levels in the atmosphere" as "Levels of
atmospheric carbon monoxide" and "grew by enough of an increased percentage" as
"increased sufficiently." it" from the sentence
The choice also eliminates the pronoun "

and reworks the final clause--"causing the average surface


temperature to rise"-- as a
modifier, thereby more clearly connecting it to the main
clause. (D) This choice uses
the unnecessarily wordy phrase "increased by a sufficient percentage." It also uses
"which" to refer to the action of the preceding clause, though "which" grammatically
refers only to the immediately preceding noun (in this case, "Earth"). (E) This choice
alters the position of during the twentieth century" and thereby changing the meaning
of the sentence. In this choice "during the twentieth century" modifies the carbon
monoxide levels instead of describing when those levels "increased." This distorts the
meaning by leaving open the possibility that twentieth century carbon monoxide levels
"increased enough" during some other time period (e.g., the 21st century).

15. Starting at age four, Mozart's father began taking him on tours of the capitals of Europe, in order to
demonstrate his musical talents.
Mozart's father began taking him on tours of the capitals of Europe, in order to demonstrate his
musical talents
Mozart's father began taking the boy on tours of the capitals of Europe, to demonstrate his musical
talents
Mozart began accompanying his father on tours of the capitals of Europe, to demonstrate his own
musical talents
Mozart was accompanying his father on tours of Europe's capitals, to demonstrate his musical talents
Mozart's father began taking him on tours of the capitals of Europe, to demonstrate the boy's musical
talents

15. The original sentence begins with a modifier ("starting at age four") that describes
Mozart. Yet, the subject of the main clause is Mozart's father. We need to find a
choice that makes Mozart himself the subject. Moreover, the pronoun "him" has no
grammatical antecedent, since it is meant to refer to Mozart despite the fact that
Mozart is not actually present in the sentence (we have "Mozart's father" instead).
Finally, the pronoun "his" is somewhat ambiguous: does it refer to Mozart (who,
again, is not in the sentence) or to his father? (A) This choice is incorrect as it repeats
the original sentence. (B) "Mozart's father" serves illogically as the subject of the
opening modifier "starting at age four." Additionally, the pronoun "his" is ambiguous in
that it could refer to the boy or the father. (C) CORRECT. "Mozart" is correctly placed
as the subject of the opening modifier "starting at age four." Additionally, the phrase
"his own" clarifies that the musical talents in question are those of the subject, Mozart.

was," which does not match the ongoing


(D) This choice incorrectly uses the verb "

nature of the modifier "starting at age four." Additionally, the pronoun


"his" is
ambiguous in that it could refer to Mozart or his father. (E) "Mozart's father" serves
illogically as the subject of the opening modifier "starting at age four." Additionally, the
pronoun "him" has no grammatical antecedent, since it is meant to refer to Mozart
despite the fact that Mozart is not actually present in the sentence (we have "Mozart's
father" instead).

16. Congress has enacted legislation forbidding state and local governments from raising taxes on
connections that link consumers to the Internet for the next three years.
forbidding state and local governments from raising taxes on connections that link consumers to the
Internet for the next three years.
that forbids state and local governments for the next three years from raising taxes on connections
that link consumers to the Internet.
that for the next three years forbids state and local governments to raise taxes on connections that
link consumers to the Internet.
forbidding for the next three years to state and local governments the raising of taxes on connections
that link consumers to the Internet
that forbids for the next three years state and local governments from raising taxes on connections
that link consumers to the Internet

16. The original sentence contains several errors. First , the preferred idiom is
"forbid X to
do Y" and not "forbid X from doing Y". Second, the placement of the
adverbial
modifier "for the next three years" suggests that it modifies "link." However, the
legislation does not forbid an Internet connection that "links" for the next three years;
it "forbids" for the next three years raising Internet connection taxes. (A) This choice is
incorrect as it repeats the original sentence. (B) This choice incorrectly uses "forbid X

." (C) CORRECT. This choice


from doing Y" instead of the idiomatic "forbid X to do Y

correctly uses the idiom "forbids X to do Y." Additionally "for


the next three years" is
correctly placed next to the verb it modifies, "forbids." (D) This
choice creates an
awkward sentence by using "forbidding to X Y" (where Y is the awkward noun "the
raising of taxes") instead of the idiomatic "forbid X to do Y." (E) This choice incorrectly

uses " forbid X from doing Y" instead of the idiomatic "forbid X to do Y."

17. An economic recession can result from a lowering of employment rates triggered by a drop in
investment, which causes people to cut consumer spending and starts a cycle of layoffs leading back
to even lower employment rates.
a lowering of employment rates triggered by a drop in investment, which causes people to cut
consumer spending and start a cycle of layoffs leading back to even lower employment rates.
a lowering of employment rates triggered by dropping investment, which causes people to cut
consumer spending and starts a cycle of layoffs leading back to even lower employment rates.
falling employment rates triggered by a drop in investment, which cause cutbacks in consumer
spending, starting a cycle of layoffs that lead to even lower employment rates.
falling employment rates that are triggered by a drop in investment, causing people to cut consumer
spending and starting a cycle of layoffs that lead back to even lower employment rates.
falling employment rates that are triggered by a drop in investment, causing cutbacks in consumer
spending and starting a cycle of layoffs leading to even lower employment rates.

17. The original sentence contains a clause beginning with "which" that logically
describes the result of lower employment rates. However, as written, this clause
seems to describe the result of "a drop in investment" because "which" modifies the
noun just before it. We need to find a replacement that makes the causal relationship
clear. Additionally, the phrase "causes people to cut consumer spending" is wordy
and somewhat illogical since the people are the consumers. A more concise way to
say this would be "causes cutbacks in consumer spending." Finally, the use of "back"
is redundant, as it is implied by the word "cycle". (A) This choice is incorrect as it
repeats the original sentence. (B) The use of "which" incorrectly suggests that
"dropping investment" "causes people to cut consumer spending" when, in fact, the
employment rates cause this phenomenon. Additionally, the phrase "causes people to
cut consumer spending" is wordy and the use of "back" is redundant, as it is implied
by the word "cycle". (C) CORRECT. This choice makes clear, through the use of the
- 100 -
plural verb "cause", that the employment rates are responsible for the cutbacks in
spending. Further it uses the concise phrase "cutbacks in consumer spending" and
eliminates the redundant word "back." (D) This choice contains the wordy phrase
"causing people to cut consumer spending" and the redundant phrase "lead back."
Moreover, the words "causing" and "starting" illogically refer back to the economic
recession. In fact, the falling employment rates, not the economic recession, cause
the cutbacks in consumer spending and start the cycle of layoffs. (E) The words
"causing" and "starting" illogically refer back to the economic recession. In fact, the
falling employment rates, not the economic recession, cause the cutbacks in
consumer spending and start the cycle of layoffs.

cording to a survey of graduating medical students conduct


Sat Nov 15, 2008 6:56 pm

GMATPrep 1:

minority graduates are


According to a survey of graduating medical students conducted by the Association of American Medical Colleges,
nearly four times more likely than are other graduates in planning to practice in socioeconomically deprived areas.
(A) minority graduates are nearly four times more likely than are other graduates in planning to practice
(B) minority graduates are nearly four times more likely than other graduates who plan on practicing
(C) minority graduates are nearly four times as likely as other graduates to plan on practicing
(D) it is nearly four times more likely that minority graduates rather than other graduates will plan to practice
(E) it is nearly four times as likely for minority graduates than other graduates to plan to practice

The correct answer is C.


I chose A.
1. Could you please clarify the difference bt "four times more likely than" and "four times as likely as"?
2. Is "plan to" a wrong idiom? is "plan on" the correct idiom?

Thanks so much!

Jamie
Guest

Sun Nov 16, 2008 9:24 am

please help! you skipped my question :-(


thanks,
j
RonPurewal
ManhattanGMAT Staff

Posts: 16635
Joined: Tue Aug 14, 2007 8:23 am

Re: According to a survey of graduating medical students con


Fri Nov 28, 2008 8:19 am

Jamie wrote:1. Could you please clarify the difference bt "four times more likely than" and "four times as likely as"?

replicated from this post. this post addresses more than just your question, but your answer is in there:

heh.

mathematically speaking, there's a difference between '4 times more likely' and '4 times as likely'. Specifically, '4 times more
likely' is actually the same as '5 times as likely', although even experts accidentally conflate the two constructions on occasion.

but this is a verbal question, so let's set the mathematical nitpicking aside; there is no language-based reason to prefer one or the other of these
constructions.

the biggest problem with D is its poor idiomatic construction. you don't say 'it is X times MORE likely that A will happen, RATHER THAN B'; 'more' is
supposed to go with 'THAN', and is incompatible with 'rather than'. the proper construction would be 'it is X times more likely that A
will happen than that B will happen.' better than either of these, though, is the more compact form: 'A is X times more likely to
happen than is B.'

hth.

2. Is "plan to" a wrong idiom? is "plan on" the correct idiom?


no, "plan to" is fine.
likely ... in planning" (incorrect, in (a))
but you missed the idiom that actually _is_ incorrect in this sentence: "
vs. "likely ... to plan" (correct, in (b)).
"likely" must be used with an infinitive. in their usual dastardly way, the gmat writers have camouflaged
this poor idiomatic usage behind not only lots of noise (the words between "likely" and "in"), but also a second, correct idiom (the
one you singled out).
that's tough.

TGC wrote:

Going with the 'cut the fluff' does the sentence makes sense?

More than 300 rivers drain into Siberia's Lake Baikal, which holds 20 percent of the world's fresh water, more than all the North American Great

Lakes combined.

More than 300 rivers drain into Siberia's Lake Baikal more than all the North American Great Lakes combined.

Can anyone enlighten me?

Dear TGC,

I'm happy to respond.

Simply put, you cut the wrong fluff. The structure of this sentence is:
[independent clause][modifier #1[modifier #2]
and "modifier #2" modifies the words "20 percent" in modifier #1. If we eliminate modifier #1, it doesn't make sense, because what modifier #2 is
modifying isn't there!

Does all this make sense?

Mike

_________________

jimmyjamesdonkey wrote:

More than 300 rivers drain into Siberia's Lake Baikal, which holds 20 percent of the world's

fresh water, more than all the North American Great Lakes combined.

(A) More than 300 rivers drain into Siberia's Lake Baikal, which holds 20 percent of the world's

fresh water, more than all the North American Great Lakes combined.

Hello Experts,
Can someone please explain what the modifier "more than all the North American Great Lakes
combined" is modifying? Is it modifying previous modifier or is it modifying the main clause. Of
late, I have been seeing such constructions, and really want to understand how such
constructions work.

Thanks
Himanshu
_________________

+1 Kudos me, Help me unlocking GMAT Club Tests


Aspire009 Re: GMATPrep [#permalink] 19 Sep 2013, 12:19
Current Student
Bookmark
this post
jimmyjamesdonkey wrote:
More than 300 rivers drain into Siberia's Lake Baikal, which holds 20 per
water, more than all the North American Great Lakes combined.

(A) More than 300 rivers drain into Siberia's Lake Baikal, which holds 20
fresh water, more than all the North American Great Lakes combined.

(B) With 20 percent of the world's fresh water, that is more than all the
Lakes combined, Siberia's Lake Baikal has more than 300 rivers that dra

(C) Siberia's Lake Baikal, with more than 300 rivers draining into it, it h
fresh water than all that of the North American Great Lakes combined,
Joined: 23 Jul 2013
Posts: 47 (D) While more than 300 rivers drain into it, Siberia's lake Baikal holds 2
Location: India fresh water, which is more than all the North American Great Lakes com
Concentration: Strategy, Entrepreneurship
Schools: McDonough '16 (WL),Goizueta '16 (D), Merage '16 (D),Boston U '16 (E) More than all the North American Great Lakes combined, Siberia's La
(WL), Ivey '15 (D),Schulich '16 (A), Foster '16 (D),Babson '16 (A), Desautels '16 than 300 rivers draining into it, holds 20 percent of the world's fresh wa
(A),Sauder '16 (I), Cox '16 (D)
GMAT 1: 680 Q49 V34
WE: Programming (Computer Software) A for me
Followers: 0
Kudos [?]: 5 [0], given: 0
FOLLOW
SEND PM
Re: GMATPrep [#permalink] 20 Sep 2013, 10:43

2
This post received
KUDOS

mikemcgarry
Magoosh GMAT Instructor Expert's post

1
This post was
BOOKMARKED
Bookmark
this post
imhimanshu wrote:

Hello Experts,

Can someone please explain what the modifier "more than all the North

Joined: 28 Dec 2011 combined" is modifying? Is it modifying previous modifier or is it modify


Posts: 2535
Followers: 827 late, I have been seeing such constructions, and really want to understa
Kudos [?]: 3393 [2] , given: 38
FOLLOW constructions work.

Thanks

Himanshu

Dear Himanshu,
I'm responding to your pm, and I am happy to help.

More than 300 rivers drain into Siberia's Lake Baikal, which hold
world's fresh water, more than all the North American Great Lak

One way to think about it is: exactly what is "more than all the North
Lakes combined"? The Great Lakes, as lakes, are fresh water, so we a
another quantity of fresh water --- "20 percent of the world's fresh w
that is the object of the the noun-modifying phrase beginning with "mo
surprisingly, the modifier touches the noun, in accordance with the Mod
this blog for more on that rule:
http://magoosh.com/gmat/2013/modifiers- ... orrection/
While there are exceptions to the Modifier Touch Rule, that's an exactly
for the noun modified.

Keep in mind that modifiers are a part of what the GMAT calls "Logica
the SC ----- in other words, to make sense of whether modifiers are use
they are modifying, you have to look beyond the grammar of a sentence
the logic & meaning, which are more primary. See this blog for more
http://magoosh.com/gmat/2013/logical-pr ... orrection/

Does all this make sense?


Mike
_________________

Mike McGarry
Magoosh Test Prep
Chiranjeevee Re: GMATPrep [#permalink] 13 Dec 2013, 10:31
Intern
Bookmark
this post
mikemcgarry wrote:

imhimanshu wrote:

Hello Experts,

Can someone please explain what the modifier "more than all the North

combined" is modifying? Is it modifying previous modifier or is it modify

late, I have been seeing such constructions, and really want to understa

constructions work.
Joined: 28 Jan 2013
Posts: 34
Followers: 0
Kudos [?]: 16 [0], given: 20 Thanks
FOLLOW
SEND PM
Himanshu

Dear Himanshu,

I'm responding to your pm, and I am happy to help.

More than 300 rivers drain into Siberia's Lake Baikal, which hold

world's fresh water, more than all the North American Great Lak

One way to think about it is: exactly what is "more than all the North

Lakes combined"? The Great Lakes, as lakes, are fresh water, so we a

another quantity of fresh water --- "20 percent of the world's fresh w

that is the object of the the noun-modifying phrase beginning with "mo

surprisingly, the modifier touches the noun, in accordance with the Mod

this blog for more on that rule:

http://magoosh.com/gmat/2013/modifiers- ... orrection/

While there are exceptions to the Modifier Touch Rule, that's an exactly

for the noun modified.

Keep in mind that modifiers are a part of what the GMAT calls "Logica

the SC ----- in other words, to make sense of whether modifiers are use

they are modifying, you have to look beyond the grammar of a sentence

the logic & meaning, which are more primary. See this blog for more

http://magoosh.com/gmat/2013/logical-pr ... orrection/

Does all this make sense?

Mike
Re: GMATPrep [#permalink] 13 Dec 2013, 16:18

1
This post received
KUDOS

mikemcgarry
Magoosh GMAT Instructor Expert's post
Bookmark
this post
Chiranjeevee wrote:

Can u please clarify, What is the main verb in option A? Thanks

Dear Chiranjeevee,
I'm happy to help.

Here is version (A):


More than 300 rivers drain into Siberia's Lake Baikal, which hold
world's fresh water, more than all the North American Great Lak
The main subject is "more than 300 rivers", and the main verb is "dra
Joined: 28 Dec 2011
the comma is the independent clause of the sentence. After that are ju
Posts: 2535
Does this make sense?
Followers: 827
Kudos [?]: 3393 [1] , given: 38 Mike
FOLLOW _________________

Mike McGarry
Magoosh Test Prep

n the mid-1920s the Hawthorne Works


Sun May 31, 2009 9:32 am

In the mid-1920s the Hawthorne Works of the Western Electric Company was the scene of anintensive series of experiments that would investigate
changes in working conditions as to theireffects on workers' performance.
(A) that would investigate changes in working conditions as to their effects on workers' performance
(B) investigating the effects that changes in working conditions would have on workers'performance
(C) for investigating what the effects on workers' performance are that changes in workingconditions would cause
(D) that investigated changes in working conditions' effects on workers' performance
(E) to investigate what the effects changes in working conditions would have on workers'performance

AnswerB
i don't know why C and D are wrong.
my opinion
Atheir --->not clear
Ewhat the effects changes--->what the effects that changes

thx in advance
kramacha1979
Students

Posts: 68
RonPurewal
ManhattanGMAT Staff

Posts: 16635
Joined: Tue Aug 14, 2007 8:23 am

Tue Sep 11, 2007 5:52 am

There are 2 problems with A.


- Major problem is "THE other infections." THE is too definitive here, carrying the connotation of "every single one of the other infections."
* THE is also incompatible with "such as":
- Correct: I never read this book, but I read the other books on the shelf.
- Correct: I never read this book, but I read other books on the shelf, such as "Right Hand, Left Hand" and "The Rise and Fall of the Third Reich."
- Incorrect: I never read this book, but I read the other books on the shelf, such as "Right Hand, Left Hand" and "The Rise and Fall of the Third Reich."
- Minor problem is "what they had not..." vs. "something they had not..." The "what" construction is awfully strong, suggesting that this was THE ONE
THING they hadn't thought possible.
* As an analogy, compare the meanings of "I want to do what I love for a living" and "I want to do something I love for a living." The first suggests that
the speaker has one particular field in mind; the second doesn't.
Last edited by RonPurewal on Thu Jan 17, 2008 7:30 pm, edited 1 time in total.

22. Teachers in this country have generally been trained either to approach mathematics like a creative
activity or that they should force students to memorize rules and principles without truly understanding
how to apply them.
to approach mathematics like a creative activity or that they should force students to memorize rules
and principles
to approach mathematics like a creative activity or to force students to memorize rules and principles
to approach mathematics as a creative activity or to force students to memorize rules and principles
that they should approach mathematics as a creative activity or to force students to memorize rules
and principles
that they should approach mathematics like a creative activity or that they should force students to
memorize rules and principles
Choose the correct inference

+1 for C.

The operating word here is "[trained to ]approach mathematics " and not "mathematics". The usage of "as" is to indicate "approach creative
activity in the same way as you approah mathematics.".

Mathematics is like creative activity . (Correct, here we are comparing mathematics to creative
activity).

The dog is trained to treat human being as its friend. (Also correct , though as is between two nouns, the
operating word is "trained to treat" ).

Crick

22. The original sentence incorrectly pairs an infinitive ("to approach") with a clause ("that
they should...") in the construction "either X or Y." Moreover, the use of "like" in the
phrase "to approach mathematics like a creative activity" is incorrect. :"As" should be
used instead. (A) This choice is incorrect as it repeats the original sentence. (B) While
this choice does contain proper parallel structure, it incorrectly uses "like" instead of
"as" in the phrase "to approach mathematics like a creative activity". (C) CORRECT.
The construction "either X or Y" requires parallelism between X and Y. In choice C,
both X and Y are parallel infinitive phrases ("to approach . . ." and "to force . . ."). (D)
This choice incorrectly pairs a clause ("that they should...") with an infinitive ("to
approach") in the construction "either X or Y." (E) While this choice does create a
parallel construction, it awkwardly begins the parallel elements with the words "that
they" instead of the infinitive "to." Moreover, this choice incorrectly uses "like" instead
of "as" in the phrase "to approach mathematics like a creative activity".

26. In response to growing demand for high-end vehicles, the interiors of the newest models of car are so
luxurious that they sell for nearly twice the price of last year's models.
the interiors of the newest models of car are so luxurious that they sell for nearly twice the price of
last year's models
the interiors of the newest models of car are so luxurious that the cars sell for nearly twice the price of
last year's models
auto makers have installed interiors in the newest models of car that are so luxurious that they sell for
nearly twice the price of last year's models
the interior of the newest models of car are so luxurious that they are sold for nearly twice the price of
last year's models
auto makers have installed interiors in the newest models of car that are so luxurious that the cars sell
for nearly twice the price of last year's models

26. The correct answer is E. The original sentence begins with a modifier "In response
to growing demand for high-end vehicles" but this modifier has no logical subject
within the main clause. Moreover, the pronoun "they" is ambiguous, as it could
grammatically refer to either the interiors or to the cars. We know that the intended
antecedent of "they" is the cars, so we need to find a choice that makes this clear.
- 103 -
The only choice that remedies both issues without creating any new ones is E.

27. Historians have long debated whether the spectacular rise and fall of Napoleon in the decades after
the French Revolution was the necessary outcome of the political vacuum having been created by the
toppled monarchy.
was the necessary outcome of the political vacuum having been created by the toppled monarchy
was the necessary outcome of the political vacuum created by toppling the monarchy
were the necessary outcome of the political vacuum that the toppled monarchy created
was the necessary outcome of the political vacuum created from toppling the monarchy
were the necessary outcome of the political vacuum created by the toppling of the monarchy

Were Napoleon's rise and Napolean's fall one outcome or two? Two - a rise is different than a fall, they took place at distinct
times, etc.

Be careful with colloquial language. Yes, technically, we can say things like "bread and butter is my favorite meal " but if you say that,
you're saying you eat them together, as one thing. If you literally were to eat bread and, once finished with the bread, you then
had some butter for dessert, you'd have to make that sentence plural!

Oops, I forgot to address your second question:


The bread and butter sculpture was created by Amy means Amy created the sculpture.
If I wanted to use created from instead, I'd say the sculpture was created from bread and butter. (Not created from Amy, hopefully!) :)
Stacey Koprince
Instructor
Director of Online Community
ManhattanGMAT

The correct answer is E. The original sentence contains a subject-verb


problem: "the
spectacular rise and fall of Napoleon" is a plural noun, but the
corresponding verb
"was" is singular. Moreover, "having been created" is unnecessarily complex.
"Created by" would have been sufficient. Finally, it is not the "toppled
monarchy" that
created the vacuum but rather the fact that the monarchy was toppled.
Choice A
repeats the original sentence. Incorrect. Choice B does not correct the
subject-verb
problem. Incorrect. Choice C corrects the subject-verb problem but not
the "toppled
monarchy" issue. Incorrect. Choice D does not correct the subject-verb
issue.
Incorrect. Choice E corrects all the problems without creating any new
ones. Correct

30. The more people that move to western states, which is an already overburdened ecosystem, the more
pressure on water resources becomes increasingly great, eventually requiring the diversion of major
rivers and the construction of dams.

The more people that move to western states, which is an already overburdened ecosystem, the more
pressure on water resources becomes increasingly great
The more that people move to western states, which is an already overburdened ecosystem, the
greater the pressure is on water resources
With more people moving to western states, an already overburdened ecosystem, the more pressure
on water resources becomes increasingly great
The more that people move to western states, an already overburdened ecosystem, the greater the
pressure on water resources becomes
The more people move to western states, which are already an overburdened ecosystem, the
greater
the pressure on water resources becomes

30. The original sentence uses the verb singular verb "is" with the plural noun
"western
states". Moreover, the phrase "the more people that move to western states"
is not
parallel with "the more pressure on water resources becomes increasingly
great."
These two parts of the idiomatic construction "the more x, the more y" need
to be in
the same form. We need to find a replacement. Choice A is the same as the
original
sentence. Incorrect. Choice B repeats the subject-verb mistake of the
original and
does not rectify the parallelism issue. Incorrect. Choice C makes the
parallelism even
worse by beginning with a phrase "with more people moving to western
states" that is
not capable of sustaining the "the more x, the more y" construction.
Incorrect. Choice
D avoids the subject-verb problem but does not remedy the parallelism issue,
since
"the more that people move" is not parallel with "the greater the pressure on
water
resources becomes". Incorrect. Choice E remedies the subject-verb problem
and
deals with the parallelism issue with the pair of phrases "the more people
move to
western states" and "the greater the pressure on water resources becomes."
Correct.

32. With an emphasis on color and form at the expense of exact duplication of detail, art historians have
suggested that Impressionism had evolved in response to the advent of black-and-white photography,
which allowed precise, albeit monochromatic, pictorial reproduction of a landscape.

With an emphasis on color and form at the expense of exact duplication of detail, art historians have
suggested that Impressionism had evolved
Emphasizing color and form at the expense of exact duplication of detail, it has been suggested by art
historians that Impressionism evolved
Art historians have suggested that Impressionism, with its emphasis on color and form at the expense
of exact duplication of detail, had evolved
Art historians have suggested that Impressionism, with its emphasis on color and form at the expense
of exact duplication of detail, evolved
Impressionism, with its emphasis on color and form at the expense of exact duplication of detail, was
suggested by art historians to have evolved

32. The correct answer is D. The original sentence contains several errors.
First, "with
an emphasis on color and form at the expense of exact duplication of detail"
is a
modifier. In this case, however, it modifies "art historians" instead of the
logically
appropriate "Impressionism." Second, the verb "had evolved" is in the
past perfect
tense when it should be in the simple past, since it is paired with
"have suggested",
which is in present perfect. Choice A is the same as the original sentence.
Incorrect.
Choice B does not remedy the modifier issue, though it does fix the tense
problem.
- 104 -
Incorrect. Choice C remedies the modifier issue, though it does not fix the
tense
problem. Incorrect. Choice D remedies the modifier issue and the tense
problem.
Correct. Choice E remedies the modifier issue, but creates an awkward
passive
construction. Incorrect.

Вам также может понравиться